You are on page 1of 95

1.

Employer- employee relationship

3. Test to determine the existence of employer- employee relationship CHARLIE JAO Petitioner - versus -BCC PRODUCTS SALES INC., and TERRANCE TY, Respondents. Facts: Petitioner maintained that respondent BCC Product Sales Inc. (BCC) and its President, respondent Terrance Ty (Ty), employed him as comptroller starting from September 1995 with a monthly salary of P20,000.00 to handle the financial aspect of BCCs business;1[2] that on October 19,1995, the security guards of BCC, acting upon the instruction of Ty, barred him from entering the premises of BCC where he then worked; that his attempts to report to work in November and December 12, 1995 were frustrated because he continued to be barred from entering the premises of BCC; 2[3] and that he filed a complaint. Although Labor Arbiter Felipe Pati ruled in favor of petitioner on June 24, 1996, 3[6] the NLRC vacated the ruling and remanded the case for further proceedings.4[7] Thereafter, Labor Arbiter Jovencio Ll. Mayor rendered a new decision on September 20, 2001, dismissing petitioners complaint for want of an employer-employee relationship between the parties.5[8] Petitioner appealed the September 20, 2001 decision of Labor Arbiter Mayor. NLRC reversed. CA reversed finding no employer-employee relationship. Issue: Whether petitioner was respondents employee or not (won the employer-employee relationship existed) Ruling: SC affirmed the CA ruling. Petitioner was not an employee of the respondent. (no employer employee relationship exists.) Etched in an unending stream of cases are the four (4) standards in determining the existence of an employer-employee relationship, namely, (a) the manner of selection and engagement of the putative employee; (b) the mode of payment of wages; (c) the presence or absence of power of dismissal; and, (d) the presence or absence of control of the putative employees conduct. Of these powers the power of control over the employees conduct is generally regarded as determinative of the existence of the relationship. Apparently, in the case before us, all these four elements are absent. First, there is no proof that the services of the private respondent were engaged to perform the duties of a comptroller in the petitioner company. There is no proof that the private respondent has undergone a selection procedure as a standard requisite for employment, especially with such a delicate position in the company. Neither is there any proof of his appointment nor is there any showing that the

1 2 3 4 5

Page 1 of 95

parties entered into an employment contract, stipulating thereof that he will receive P20,000.00/month salary as comptroller, before the private respondent commenced with his work as such. Second, as clearly established on record, the private respondent was not included in the petitioner companys payroll during the time of his alleged employment with the former. True, the name of the private respondent Charlie Jao appears in the payroll however it does not prove that he has received his remuneration for his services. Notably, his name was not among the employees who will receive their salaries as represented by the payrolls. Instead, it appears therein as a comptroller who is authorized to approve the same. Suffice it to state that it is rather obscure for a certified public accountant doing the functions of a comptroller from September 1995 up to December 1995 not to receive his salary during the said period. Verily, such scenario does not conform with the usual and ordinary experience of man. Coming now to the most controlling factor, the records indubitably reveal the undisputed fact that the petitioner company did not have nor did not exercise the power of control over the private respondent. It did not prescribe the manner by which the work is to be carried out, or the time by which the private respondent has to report for and leave from work. As already stated, the power of control is such an important factor that other requisites may even be disregarded. Moreover, in determining the presence or absence of an employer-employee relationship, the Court has consistently looked for the following incidents, to wit: (a) the selection and engagement of the employee; (b) the payment of wages; (c) the power of dismissal; and (d) the employers power to control the employee on the means and methods by which the work is accomplished. The last element, the so-called control test, is the most important element. 6[24]Hereunder are some of the circumstances and incidents occurring while petitioner was supposedly employed by BCC that debunked his claim against respondents. It can be deduced from the March 1996 affidavit of petitioner that respondents challenged his authority to deliver some 158 checks to SFC. Considering that he contested respondents challenge by pointing to the existing arrangements between BCC and SFC, it should be clear that respondents did not exercise the power of control over him, because he thereby acted for the benefit and in the interest of SFC more than of BCC. In addition, petitioner presented no document setting forth the terms of his employment by BCC. The failure to present such agreement on terms of employment may be understandable and expected if he was a common or ordinary laborer who would not jeopardize his employment by demanding such document from the employer, but may not square well with his actual status as a highly educated professional.Petitioners admission that he did not receive his salary for the three months of his employment by BCC, as his complaint for illegal dismissal and non-payment of wages 7[25] and the criminal case for estafa he later filed against the respondents for non-payment of wages 8[26] indicated, further raised grave doubts about his assertion of employment by BCC. If the assertion was true, we are puzzled how he could have remained in BCCs employ in that period of time despite not being paid the first salary of P20,000.00/month. Moreover, his name did not appear in the payroll of BCC despite him having approved the payroll as comptroller.Lastly, the confusion about the date of his alleged illegal dismissal provides another indicium of the insincerity of petitioners assertion of employment by BCC. In the petition for review on certiorari, he averred that he had been barred from entering the premises of BCC on October 19, 1995,9[27] and thus was illegally dismissed. Yet, his complaint for illegal dismissal stated that he had been illegally dismissed on December 12, 1995 when respondents security guards barred him from entering the premises of BCC,10[28] causing him to bring his complaint only on December 29, 1995, and after BCC had already filed the criminal complaint against him. The wide gap between October 19, 1995 and December 12, 1995 cannot be dismissed as a trivial inconsistency considering that the several incidents affecting the veracity of his assertion of employment by BCC earlier noted herein transpired in that interval. With all the grave doubts thus raised against petitioners claim, we need not dwell at length on the other proofs he presented, like the affidavits of some of the employees of BCC, the ID, and the signed checks, bills and receipts. Suffice it to be stated that such other proofs were easily explainable by respondents and by the aforestated circumstances showing him to be the employee of SFC, not of BCC.

6 7 8 9 10

Page 2 of 95

Bernarte vs PBA Facts: Complainants (Jose Mel Bernarte and Renato Guevarra) aver that they were invited to join the PBA as referees. During the leadership of Commissioner Emilio Bernardino, they were made to sign contracts on a year-to-year basis. During the term of Commissioner Eala, however, changes were made on the terms of their employment. On January 15, 2004, Bernarte received a letter from the Office of the Commissioner advising him that his contract would not be renewed citing his unsatisfactory performance on and off the court. It was a total shock for Bernarte who was awarded Referee of the year in 2003. He felt that the dismissal was caused by his refusal to fix a game upon order of Ernie De Leon. On the other hand, complainant Guevarra alleges that he was invited to join the PBA pool of referees in February 2001. On March 1, 2001, he signed a contract as trainee. Beginning 2002, he signed a yearly contract as Regular Class C referee. On May 6, 2003, respondent Martinez issued a memorandum to Guevarra expressing dissatisfaction over his questioning on the assignment of referees officiating out-of-town games. Beginning February 2004, he was no longer made to sign a contract. Issue: WON employer-employee relationship exists between the referees and PBA Ruling: No employer-employee relationship. (referees are independent contractor) The existence of an employer-employee relationship is ultimately a question of fact. As a general rule, factual issues are beyond the province of this Court. However, this rule admits of exceptions, one of which is where there are conflicting findings of fact between the Court of Appeals, on one hand, and the NLRC and Labor Arbiter, on the other, such as in the present case.18To determine the existence of an employer-employee relationship, case law has consistently applied the four-fold test, to wit: (a) the selection and engagement of the employee; (b) the payment of wages; (c) the power of dismissal; and (d) the employers power to control the employee on the means and methods by which the work is accomplished. The so-called control test is the most important indicator of the presence or absence of an employeremployee relationship.19 In this case, PBA admits repeatedly engaging petitioners services, as shown in the retainer contracts. PBA pays petitioner a retainer fee, exclusive of per diem or allowances, as stipulated in the retainer contract. PBA can terminate the retainer contract for petitioners violation of its terms and conditions. However, respondents argue that the all-important element of control is lacking in this case, making petitioner an independent contractor and not an employee of respondents.Petitioner contends otherwise. Petitioner asserts that he is an employee of respondents since the latter exercise control over the performance of his work. Petitioner cites the following stipulations in the retainer contract which evidence control: (1) respondents classify or rate a referee; (2) respondents require referees to attend all basketball games organized or authorized by the PBA, at least one hour before the start of the first game of each day; (3) respondents assign petitioner to officiate ballgames, or to act as alternate referee or substitute; (4) referee agrees to observe and comply with all the requirements of the PBA governing the conduct of the referees whether on or off the court; (5) referee agrees (a) to keep himself in good physical, mental, and emotional condition during the life of the contract; (b) to give always his best effort and service, and loyalty to the PBA, and not to officiate as referee in any basketball game outside of the PBA, without written prior consent of the Commissioner; (c) always to conduct himself on and off the court according to the highest standards of honesty or morality; and (6) imposition of various sanctions for violation of the terms and conditions of the contract.

The foregoing stipulations hardly demonstrate control over the means and methods by which petitioner performs his work as a referee officiating a PBA basketball game. The contractual stipulations do not pertain to, much less dictate, how and

Page 3 of 95

when petitioner will blow the whistle and make calls. On the contrary, they merely serve as rules of conduct or guidelines in order to maintain the integrity of the professional basketball league. As correctly observed by the Court of Appeals, how could a skilled referee perform his job without blowing a whistle and making calls? x x x [H]ow can the PBA control the performance of work of a referee without controlling his acts of blowing the whistle and making calls?20 We agree with respondents that once in the playing court, the referees exercise their own independent judgment, based on the rules of the game, as to when and how a call or decision is to be made. The referees decide whether an infraction was committed, and the PBA cannot overrule them once the decision is made on the playing court. The referees are the only, absolute, and final authority on the playing court. Respondents or any of the PBA officers cannot and do not determine which calls to make or not to make and cannot control the referee when he blows the whistle because such authority exclusively belongs to the referees. The very nature of petitioners job of officiating a professional basketball game undoubtedly calls for freedom of control by respondents. Moreover, the following circumstances indicate that petitioner is an independent contractor: (1) the referees are required to report for work only when PBA games are scheduled, which is three times a week spread over an average of only 105 playing days a year, and they officiate games at an average of two hours per game; and (2) the only deductions from the fees received by the referees are withholding taxes. In other words, unlike regular employees who ordinarily report for work eight hours per day for five days a week, petitioner is required to report for work only when PBA games are scheduled or three times a week at two hours per game. In addition, there are no deductions for contributions to the Social Security System, Philhealth or Pag-Ibig, which are the usual deductions from employees salaries. These undisputed circumstances buttress the fact that petitioner is an independent contractor, and not an employee of respondents. Furthermore, the applicable foreign case law declares that a referee is an independent contractor, whose special skills and independent judgment are required specifically for such position and cannot possibly be controlled by the hiring party. In addition, the fact that PBA repeatedly hired petitioner does not by itself prove that petitioner is an employee of the former. For a hired party to be considered an employee, the hiring party must have control over the means and methods by which the hired party is to perform his work, which is absent in this case. The continuous rehiring by PBA of petitioner simply signifies the renewal of the contract between PBA and petitioner, and highlights the satisfactory services rendered by petitioner warranting such contract renewal. Conversely, if PBA decides to discontinue petitioners services at the end of the term fixed in the contract, whether for unsatisfactory services, or violation of the terms and conditions of the contract, or for whatever other reason, the same merely results in the non-renewal of the contract, as in the present case. The non-renewal of the contract between the parties does not constitute illegal dismissal of petitioner by respondents. MARTICIO SEMBLANTE and DUBRICK Petitioner- versus -COURT OF APPEALS, 19 TH DIVISION, now SPECIAL FORMER 19 TH DIVISION, GALLERA DE MANDAUEmSPOUSES VICENTE and MARIA LUISA LOOT, Respondents. Facts: For their services as masiador and sentenciador, Semblante receives PhP 2,000 per week or a total of PhP 8,000 per month, while Pilar gets PhP 3,500 a week or PhP 14,000 per month. They work every Tuesday, Wednesday, Saturday, and Sunday every week, excluding monthly derbies and cockfights held on special holidays. Their working days start at 1:00 p.m. and last until 12:00 midnight, or until the early hours of the morning depending on the needs of the cockpit. Petitioners had both been issued employees identification cards11[5] that they wear every time they report for duty. They alleged never having incurred any infraction and/or violation of the cockpit rules and regulations.

11

Page 4 of 95

On November 14, 2003, however, petitioners were denied entry into the cockpit upon the instructions of respondents, and were informed of the termination of their services effective that date. This prompted petitioners to file a complaint for illegal dismissal against respondents. Issue: WON masiador and sentenciador are employees of the Gellera de mandaue Ruling: No employer employee relationship. Petitioners are NOT employees of respondents, since their relationship fails to pass muster the four-fold test of employment We have repeatedly mentioned in countless decisions: (1) the selection and engagement of the employee; (2) the payment of wages; (3) the power of dismissal; and (4) the power to control the employees conduct, which is the most important element.12[18] As found by both the NLRC and the CA, respondents had no part in petitioners selection and management; 13[19] petitioners compensation was paid out of the arriba (which is a percentage deducted from the total bets), not by petitioners;14[20] and petitioners performed their functions as masiador and sentenciador free from the direction and control of respondents .15[21] In the conduct of their work, petitioners relied mainly on their expertise that is characteristic of the cockfight gambling, 16[22] and were never given by respondents any tool needed for the performance of their work.17[23] Respondents, not being petitioners employers, could never have dismissed, legally or illegally, petitioners, since respondents were without power or prerogative to do so in the first place. The rule on the posting of an appeal bond cannot defeat the substantive rights of respondents to be free from an unwarranted burden of answering for an illegal dismissal for which they were never responsible. CRC AGRICULTURAL TRADING and ROLANDO B. CATINDIG , Petitioners- versus LABORRELATIONS COMMISSION and ROBERTO OBI Respondents. Facts: In his Sinumpaang Salaysay,18[6] the respondent alleged that the petitioners employed him as a driver sometime in 1985. The respondent worked for the petitioners until he met an accident in 1989, after which the petitioners no longer allowed him to work. After six years, or in February 1995, the petitioners again hired the respondent as a driver and offered him to stay inside the companys premises. The petitioners gave him a P3,000.00 loan to help him build a hut for his family. -NATIONAL

12 13 14 15 16 17 18

Page 5 of 95

Sometime in March 2003, the petitioners ordered respondent to have the alternator of one of its vehicles repaired. The respondent brought the vehicle to a repair shop and subsequently gave the petitioners two receipts issued by the repair shop. The latter suspected that the receipts were falsified and stopped talking to him and giving him work assignments. The petitioners, however, still paid him P700.00 and P500.00 on April 15 and 30, 2004, respectively, but no longer gave him any salary after that. As a result, the respondent and his family moved out of the petitioners compound and relocated to a nearby place. The respondent claimed that the petitioners paid him a daily wage of P175.00, but did not give him service incentive leave, holiday pay, rest day pay, and overtime pay. He also alleged that the petitioners did not send him a notice of termination. Issue: WON employer-employee relationship existed Ruling: Employer-employee relationship existed between the petitioners and respondent. The elements to determine the existence of an employment relationship are: (1) the selection and engagement of the employee; (2) the payment of wages; (3) the power of dismissal; and (4) the employers power to control the employees conduct. The most important element is the employers control of the employees conduct, not only as to the result of the work to be done, but also as to the means and methods to accomplish it. All the four elements are present in this case.19[10]

First, the petitioners engaged the services of the respondent in 1995. Second, the petitioners paid the respondent a daily wage of P175.00, with allowances ranging from P140.00 to P200.00 per day. The fact the respondent was paid under a no work no pay scheme, assuming this claim to be true, is not significant. The no work no pay scheme is merely a method of computing compensation, not a basis for determining the existence or absence of employer-employee relationship. Third, the petitioners power to dismiss the respondent was inherent in the fact that they engaged the services of the respondent as a driver. Finally, a careful review of the record shows that the respondent performed his work as driver under the petitioners supervision and control. Petitioners determined how, where, and when the respondent performed his task. They, in fact, requested the respondent to live inside their compound so he (respondent) could be readily available when the petitioners needed his services. Undoubtedly, the petitioners exercised control over the means and methods by which the respondent accomplished his work as a driver.
We conclude from all these that an employer-employee relationship existed between the petitioners and respondent. DEALCO FARMS, INC., Petitioner, vs.NATIONAL LABOR CHIQUITO BASTIDA, and ALBERT CABAN, Respondents. Facts: Petitioner is a corporation engaged in the business of importation, production, fattening and distribution of live cattle for sale to meat dealers, meat traders, meat processors, canned good manufacturers and other dealers in Mindanao and in Metro Manila. Petitioner imports cattle by the boatload from Australia into the ports of General Santos City, Subic, Batangas, or Manila. In turn, these imported cattle are transported to, and housed in, petitioners farms in Polomolok, South Cotabato, or in Magalang, Pampanga, for fattening until the cattle individually reach the market weight of 430 to 450 kilograms.Respondents Albert Caban and Chiquito Bastida were hired by petitioner on June 25, 1993 and October 29, 1994, respectively, as escorts or "comboys" for the transit of live cattle from General Santos City to Manila. Respondents work entailed tending to the cattle during transportation. It included feeding and frequently showering the cattle to prevent dehydration and to develop heat resistance. On the whole, respondents ensured that the cattle would be safe from harm or death caused by a cattle fight or any such similar incident. Upon arrival in Manila, the cattle are turned over RELATIONS COMMISSION (5th DIVISION),

19

Page 6 of 95

to and received by the duly acknowledged buyers or customers of petitioner, at which point, respondents work ceases. For every round trip travel which lasted an average of 12 days, respondents were each paid P1,500.00. The 12-day period is occasionally extended when petitioners customers are delayed in receiving the cattle. In a month, respondents usually made two trips. On October 15, 1999, respondents Bastida and Caban, together with Ramon Maquinsay and Roland Parrocha, filed a Complaint for illegal dismissal with claims for separation pay with full backwages, salary differentials, service incentive leave pay, 13th month pay, damages, and attorneys fees against petitioner. Issue: WON employer-employee existed Ruling: There is employer-employee relationship existed. After a judicious review of the records of this case, we found no cogent reason to disturb the findings of the branch. The presence of the four (4) elements in the determination of an employer-employee relationship has been clearly established by the facts and evidence on record, starting with the admissions of [petitioner] who acknowledged the engagement of [respondents] as escorts of their cattles shipped from General Santos to Manila, and the compensation of the latter at a fee of P1,500.00 per trip. The dates claimed by [respondents] that they were engaged remain not disputed by [petitioner] as observed by the branch. The element of control, jurisprudentially considered the most essential element of the four, has not been demolished by any evidence to the contrary. The branch has noticed that the preparation of the shipment of cattle, manning and feeding them while in transit, and making a report upon their return to General Santos that the cattle shipped and which reached Manila actually tallied were all indicators of instructions, supervision and control by [petitioner] on [respondents] performance of work as escorts for which they were hired. This we agree on all four[s]. The livestock shipment would cost thousands of pesos and the certainty of it reaching its destination would be the only thing any operator would consider at all [time] and under all circumstances. Nothing more, nothing less. It is illogical for [petitioner] to argue that the shipment was not necessary [or] desirable to their business, as their business was mainly livestock production, because they were undeniably the owners of the cattle escorted by [respondents]. Should losses of a shipment occur due to [respondents] neglect these would still be [petitioners] loss, and nobody elses. At this point, we emphasize the fact that even on appeal [petitioner] declines to refute, by way of evidence, the finding of the branch that they failed to prove the payment of [respondents] services by any of the supposed traders, or that said traders actually shipped livestock. This is the point where the case of NFL v. Bibiana Farms cited by [petitioner] differs from the instant case in that bills of lading issued to, thus, in the name of the hog shippers were submitted as proof that said shippers engaged, compensated and supervised the escorts or convoys in their work, and not the hog raisers.8 Undoubtedly, respondents were regular employees of petitioner with respect to the escort or "comboy" activity for which they had been engaged since 1993 and 1994, respectively, without regard to continuity or brokenness of the service. Lastly, considering that we have sustained the Labor Arbiters and the NLRCs finding of an employer-employee relationship between the parties, we likewise sustain the administrative bodies finding of respondents illegal dismissal. WPP MARKETING COMMUNICATIONS, INC.,JOHN STEEDMAN, LANSANG, Petitioners, - versus -JOCELYN M. GALERA, Respondent Facts: MARK WEBSTER, and NOMINADA

Page 7 of 95

Petitioner is Jocelyn Galera (GALERA), a [sic] American citizen who was recruited from the United States of America by private respondent John Steedman, Chairman-WPP Worldwide and Chief Executive Officer of Mindshare, Co., a corporation based in Hong Kong, China, to work in the Philippines for private respondent WPP Marketing Communications, Inc. (WPP), a corporation registered and operating under the laws of Philippines. GALERA accepted the offer and she signed an Employment Contract entitled Confirmation of Appointment and Statement of Terms and Conditions. Employment of GALERA with private respondent WPP became effective on September 1, 1999 solely on the instruction of the CEO and upon signing of the contract, without any further action from the Board of Directors of private respondent WPP. Four months had passed when private respondent WPP filed before the Bureau of Immigration an application for petitioner GALERA to receive a working visa, wherein she was designated as Vice President of WPP. Petitioner alleged that she was constrained to sign the application in order that she could remain in the Philippines and retain her employment. Then, on December 14, 2000 , petitioner GALERA alleged she was verbally notified by private respondent STEEDMAN that her services had been terminated from private respondent WPP. A termination letter followed the next day.20[4] On 3 January 2001, Galera filed a complaint for illegal dismissal. Issue: WON employer-employee relationship existed (Whether Galera is an Employee or a Corporate Officer) Ruling: Galera is an employee of WPP. Galera, on the belief that she is an employee, filed her complaint before the Labor Arbiter. On the other hand, WPP, Steedman, Webster and Lansang contend that Galera is a corporate officer; hence, any controversy regarding her dismissal is under the jurisdiction of the Regional Trial Court. We agree with Galera. Corporate officers are given such character either by the Corporation Code or by the corporations by-laws. Under Section 25 of the Corporation Code, the corporate officers are the president, secretary, treasurer and such other officers as may be provided in the by-laws.21[19] Other officers are sometimes created by the charter or by-laws of a corporation, or the board of directors may be empowered under the by-laws of a corporation to create additional offices as may be necessary. An examination of WPPs by-laws resulted in a finding that Galeras appointment as a corporate officer (Vice-President with the operational title of Managing Director of Mindshare) during a special meeting of WPPs Board of Directors is an appointment to a non-existent corporate office. WPPs by-laws provided for only one Vice-President. At the time of Galeras appointment on 31 December 1999, WPP already had one Vice-President in the person of Webster. Galera cannot be said to be a director of WPP also because all five directorship positions provided in the by-laws are already occupied. Finally, WPP cannot rely on its Amended By-Laws to support its argument that Galera is a corporate officer. The Amended By-Laws provided for more than one Vice-President and for two additional directors. Even though WPPs stockholders voted for the amendment on 31 May 2000, the SEC approved the amendments only on 16 February 2001. Galera was dismissed on 14 December 2000. WPP, Steedman, Webster, and Lansang did not present any evidence that Galeras dismissal took effect with the action of WPPs Board of Directors.

20 21

Page 8 of 95

The appellate court further justified that Galera was an employee and not a corporate officer by subjecting WPP and Galeras relationship to the four-fold test: (a) the selection and engagement of the employee; (b) the payment of wages; (c) the power of dismissal; and (d) the employers power to control the employee with respect to the means and methods by which the work is to be accomplished. The appellate court found: x x x Sections 1 and 4 of the employment contract mandate where and how often she is to perform her work; sections 3, 5, 6 and 7 show that wages she receives are completely controlled by x x x WPP; and sections 10 and 11 clearly state that she is subject to the regular disciplinary procedures of x x x WPP. Another indicator that she was a regular employee and not a corporate officer is Section 14 of the contract, which clearly states that she is a permanent employee not a Vice-President or a member of the Board of Directors.x x x x Another indication that the Employment Contract was one of regular employment is Section 12, which states that the rights to any invention, discovery, improvement in procedure, trademark, or copyright created or discovered by petitioner GALERA during her employment shall automatically belong to private respondent WPP. Under Republic Act 8293, also known as the Intellectual Property Code, this condition prevails if the creator of the work subject to the laws of patent or copyright is an employee of the one entitled to the patent or copyright. Another convincing indication that she was only a regular employee and not a corporate officer is the disciplinary procedure under Sections 10 and 11 of the Employment Contract, which states that her right of redress is through Mindshares Chief Executive Officer for the Asia-Pacific. This implies that she was not under the disciplinary control of private respondent WPPs Board of Directors (BOD), which should have been the case if in fact she was a corporate officer because only the Board of Directors could appoint and terminate such a corporate officer. Although petitioner GALERA did sign the Alien Employment Permit from the Department of Labor and Employment and the application for a 9(g) visa with the Bureau of Immigration both of which stated that she was private respondents WPP Vice President these should not be considered against her. Assurming arguendo that her appointment as VicePresident was a valid act, it must be noted that these appointments occurred afater she was hired as a regular employee. After her appointments, there was no appreciable change in her duties.22[20] Gomez vs PMDC development corporation Facts: Petitioner Gloria V. Gomez used to work as Manager of the Legal Department of Petron Corporation, then a governmentowned corporation. With Petrons privatization, she availed of the companys early retirement program and left that organization on April 30, 1994. On the following day, May 1, 1994, however, Filoil Refinery Corporation (Filoil), also a government-owned corporation, appointed her its corporate secretary and legal counsel, 23[1] with the same managerial rank, compensation, and benefits that she used to enjoy at Petron.But Filoil was later on also identified for privatization. To facilitate its conversion, the Filoil board of directors created a five-member task force headed by petitioner Gomez who had been designated administrator.24[2] While documenting Filoils assets, she found several properties which were not in the books of the corporation. Consequently, she advised the board to suspend the privatization until all assets have been accounted for.With the privatization temporarily shelved, Filoil underwent reorganization and was renamed Filoil Development Management Corporation (FDMC), which later became the respondent PNOC Development Management Corporation (PDMC). When this happened, Gomezs task force was abolished and its members, including Gomez, were given termination notices on March 5, 1996. 25[3] The matter was then reported to the Department of Labor and

22 23 24 25

Page 9 of 95

Employment on March 7, 1996.26[4]Meantime, petitioner Gomez continued to serve as corporate secretary of respondent PDMC. On September 23, 1996 its president re-hired her as administrator and legal counsel of the company. 27[5] In accordance with company guidelines, it credited her the years she served with the Filoil task force. On May 24, 1998, the next president of PDMC extended her term as administrator beyond her retirement age,28[6] pursuant to his authority under the PDMC Approvals Manual.29[7] She was supposed to serve beyond retirement from August 11, 1998 to August 11, 2004. Meantime, a new board of directors for PDMC took over the company.On March 29, 1999 the new board of directors of respondent PDMC removed petitioner Gomez as corporate secretary. Further, at the boards meeting on October 21, 1999 the board questioned her continued employment as administrator. In answer, she presented the former presidents May 24, 1998 letter that extended her term. Dissatisfied with this, the board sought the advice of its legal department, which expressed the view that Gomezs term extension was an ultra vires act of the former president. It reasoned that, since her position was functionally that of a vice-president or general manager, her term could be extended under the companys by-laws only with the approval of the board. The legal department held that her de facto tenure could be legally put to an end.30[8] Sought for comment, the Office of the Government Corporate Counsel (OGCC) held the view that while respondent PDMCs board did not approve the creation of the position of administrator that Gomez held, such action should be deemed ratified since the board had been aware of it since 1994. But the OGCC ventured that the extension of her term beyond retirement age should have been made with the boards approval.31[9] Petitioner Gomez for her part conceded that as corporate secretary, she served only as a corporate officer. But, when they named her administrator, she became a regular managerial employee. Consequently, the respondent PDMCs board did not have to approve either her appointment as such or the extension of her term in 1998. Pending resolution of the issue, the respondent PDMCs board withheld petitioner Gomezs wages from November 16 to 30, 1999, prompting her to file a complaint for non-payment of wages, damages, and attorneys fees with the Labor Arbiter on December 8, 1999.32[10] She later amended her complaint to include other money claims.33[11] Issue: WON employer-employee relationship existed Ruling: Gomez is an employee of PDMC.

26 27 28 29 30 31 32 33

Page 10 of 95

But the relationship of a person to a corporation, whether as officer or agent or employee, is not determined by the nature of the services he performs but by the incidents of his relationship with the corporation as they actually exist. 34[27] Here, respondent PDMC hired petitioner Gomez as an ordinary employee without board approval as was proper for a corporate officer. When the company got her the first time, it agreed to have her retain the managerial rank that she held with Petron. Her appointment paper said that she would be entitled to all the rights, privileges, and benefits that regular PDMC employees enjoyed.35[28] This is in sharp contrast to what the former PDMC presidents appointment paper stated: he was elected to the position and his compensation depended on the will of the board of directors.36[29] What is more, respondent PDMC enrolled petitioner Gomez with the Social Security System, the Medicare, and the PagIbig Fund. It even issued certifications dated October 10, 2008, 37[30] stating that Gomez was a permanent employee and that the company had remitted combined contributions during her tenure. The company also made her a member of the PDMCs savings and provident plan38[31] and its retirement plan.39[32] It grouped her with the managers covered by the companys group hospitalization insurance.40[33] Likewise, she underwent regular employee performance appraisals, 41 [34] purchased stocks through the employee stock option plan,42[35] and was entitled to vacation and emergency leaves.43[36] PDMC even withheld taxes on her salary and declared her as an employee in the official Bureau of Internal Revenue forms.44[37] These are all indicia of an employer-employee relationship which respondent PDMC failed to refute. Estoppel, an equitable principle rooted on natural justice, prevents a person from rejecting his previous acts and representations to the prejudice of others who have relied on them. 45[38] This principle of law applies to corporations as well. The PDMC in this case is estopped from claiming that despite all the appearances of regular employment that it weaved around petitioner Gomezs position it must have technically hired her only as a corporate officer. The board and its officers made her stay on and work with the company for years under the belief that she held a regular managerial position. That petitioner Gomez served concurrently as corporate secretary for a time is immaterial. A corporation is not prohibited from hiring a corporate officer to perform services under circumstances which will make him an employee.46[39] Indeed, it is possible for one to have a dual role of officer and employee. In Elleccion Vda. De Lecciones v. National Labor

34 35 36 37 38 39 40 41 42 43 44 45 46

Page 11 of 95

Relations Commission,47[40] the Court upheld NLRC jurisdiction over a complaint filed by one who served both as corporate secretary and administrator, finding that the money claims were made as an employee and not as a corporate officer.
Locsin vs PLT Facts: On November 1, 1990, respondent Philippine Long Distance Telephone Company (PLDT) and the Security and Safety Corporation of the Philippines (SSCP) entered into a Security Services Agreement 48[3] (Agreement) whereby SSCP would provide armed security guards to PLDT to be assigned to its various offices. Pursuant to such agreement, petitioners Raul Locsin and Eddie Tomaquin, among other security guards, were posted at a PLDT office. On August 30, 2001, respondent issued a Letter dated August 30, 2001 terminating the Agreement effective October 1, 2001.49[4] Despite the termination of the Agreement, however, petitioners continued to secure the premises of their assigned office. They were allegedly directed to remain at their post by representatives of respondent. In support of their contention, petitioners provided the Labor Arbiter with copies of petitioner Locsins pay slips for the period of January to September 2002.50[5] Then, on September 30, 2002, petitioners services were terminated. Thus, petitioners filed a complaint before the Labor Arbiter for illegal dismissal and recovery of money claims such as overtime pay, holiday pay, premium pay for holiday and rest day, service incentive leave pay, Emergency Cost of Living Allowance, and moral and exemplary damages against PLDT. Issue: Won employer-employee relationship existed. The only issue in this case is whether petitioners became employees of respondent after the Agreement between SSCP and respondent was terminated. Ruling: This must be answered in the affirmative. Notably, respondent does not deny the fact that petitioners remained in the premises of their offices even after the Agreement was terminated. And it is this fact that must be explained.To recapitulate, the CA, in rendering a decision in favor of respondent, found that: (1) petitioners failed to prove that SSCP was a labor-only contractor; and (2) petitioners are employees of SSCP and not of PLDT.In arriving at such conclusions, the CA relied on the provisions of the Agreement, wherein SSCP undertook to supply PLDT with the required security guards, while furnishing PLDT with a performance bond in the amount of PhP 707,000. Moreover, the CA gave weight to the provision in the Agreement that SSCP

47 48 49 50

Page 12 of 95

warranted that it carry on an independent business and has substantial capital or investment in the form of equipment, work premises, and other materials which are necessary in the conduct of its business. Further, in determining that no employer-employee relationship existed between the parties, the CA quoted the express provision of the Agreement, stating that no employer-employee relationship existed between the parties herein. The CA disregarded the pay slips of Locsin considering that they were in fact issued by SSCP and not by PLDT. From the foregoing explanation of the CA, the fact remains that petitioners remained at their post after the termination of the Agreement. Notably, in its Comment dated March 10, 2009, 51[8] respondent never denied that petitioners remained at their post until September 30, 2002. While respondent denies the alleged circumstances stated by petitioners, that they were told to remain at their post by respondents Security Department and that they were informed by SSCP Operations Officer Eduardo Juliano that their salaries would be coursed through SSCP as per arrangement with PLDT, it does not state why they were not made to vacate their posts. Respondent said that it did not know why petitioners remained at their posts. Rule 131, Section 3(y) of the Rules of Court provides: SEC. 3. Disputable presumptions.The following presumptions are satisfactory if uncontradicted, but may be contradicted and overcome by other evidence:x x x x (y) That things have happened according to the ordinary course of nature and the ordinary habits of life. In the ordinary course of things, responsible business owners or managers would not allow security guards of an agency with whom the owners or managers have severed ties with to continue to stay within the business premises. This is because upon the termination of the owners or managers agreement with the security agency, the agencys undertaking of liability for any damage that the security guard would cause has already been terminated. Thus, in the event of an accident or otherwise damage caused by such security guards, it would be the business owners and/or managers who would be liable and not the agency. The business owners or managers would, therefore, be opening themselves up to liability for acts of security guards over whom the owners or managers allegedly have no control. At the very least, responsible business owners or managers would inquire or learn why such security guards were remaining at their posts, and would have a clear understanding of the circumstances of the guards stay. It is but logical that responsible business owners or managers would be aware of the situation in their premises. We point out that with respondents hypothesis, it would seem that SSCP was paying petitioners salaries while securing respondents premises despite the termination of their Agreement. Obviously, it would only be respondent that would benefit from such a situation. And it is seriously doubtful that a security agency that was established for profit would allow its security guards to secure respondents premises when the Agreement was already terminated. From the foregoing circumstances, reason dictates that we conclude that petitioners remained at their post under the instructions of respondent. We can further conclude that respondent dictated upon petitioners that the latter perform their regular duties to secure the premises during operating hours. This, to our mind and under the circumstances, is sufficient to establish the existence of an employer-employee relationship. Certainly, the facts as narrated by petitioners are more believable than the irrational denials made by respondent. Thus, we ruled in Lee Eng Hong v. Court of Appeals:52[9] Evidence, to be believed, must not only proceed from the mouth of a credible witness, but it must be credible in itself such as the common experience and observation of mankind can approve as probable under the circumstances. We have no test of the truth of human testimony, except its conformity to our knowledge, observation and experience. Whatever is

51 52

Page 13 of 95

repugnant to these belongs to the miraculous and is outside judicial cognizance (Castaares v. Court of Appeals, 92 SCRA 568 [1979]). To reiterate, while respondent and SSCP no longer had any legal relationship with the termination of the Agreement, petitioners remained at their post securing the premises of respondent while receiving their salaries, allegedly from SSCP. Clearly, such a situation makes no sense, and the denials proffered by respondent do not shed any light to the situation. It is but reasonable to conclude that, with the behest and, presumably, directive of respondent, petitioners continued with their services. Evidently, such are indicia of control that respondent exercised over petitioners. Such power of control has been explained as the right to control not only the end to be achieved but also the means to be used in reaching such end.53[10] With the conclusion that respondent directed petitioners to remain at their posts and continue with their duties, it is clear that respondent exercised the power of control over them; thus, the existence of an employer-employee relationship. Furthermore, Article 106 of the Labor Code contains a provision on contractors, to wit: Art. 106. Contractor or subcontractor. Whenever an employer enters into a contract with another person for the performance of the formers work, the employees of the contractor and of the latters subcontractor, if any, shall be paid in accordance with the provisions of this Code. In the event that the contractor or subcontractor fails to pay the wages of his employees in accordance with this Code, the employer shall be jointly and severally liable with his contractor or subcontractor to such employees to the extent of the work performed under the contract, in the same manner and extent that he is liable to employees directly employed by him. The Secretary of Labor and Employment may, by appropriate regulations, restrict or prohibit the contracting-out of labor to protect the rights of workers established under this Code. In so prohibiting or restricting, he may make appropriate distinctions between labor-only contracting and job contracting as well as differentiations within these types of contracting and determine who among the parties involved shall be considered the employer for purposes of this Code, to prevent any violation or circumvention of any provision of this Code. There is labor-only contracting where the person supplying workers to an employer does not have substantial capital or investment in the form of tools, equipment, machineries, work premises, among others, and the workers recruited and placed by such person are performing activities which are directly related to the principal business of such employer. In such cases, the person or intermediary shall be considered merely as an agent of the employer who shall be responsible to the workers in the same manner and extent as if the latter were directly employed by him. (Emphasis supplied.) Thus, the Secretary of Labor issued Department Order No. 18-2002, Series of 2002, implementing Art. 106 as follows: Section 5. Prohibition against labor-only contracting.Labor-only contracting is hereby declared prohibited. For this purpose, labor-only contracting shall refer to an arrangement where the contractor or subcontractor merely recruits, supplies or places workers to perform a job, work or service for a principal, and any of the following elements are present: (i) (ii) The contractor or subcontractor does not have substantial capital or investment which relates to the job, work or service to be performed and the employees recruited, supplied or placed by such contractor or subcontractor are performing activities which are directly related to the main business of the principal; or the contractor does not exercise the right to control over the performance of the work of the contractual employee.

53

Page 14 of 95

The foregoing provisions shall be without prejudice to the application of Article 248 (C) of the Labor Code, as amended. Substantial capital or investment refers to capital stocks and subscribed capitalization in the case of corporations, tools, equipment, implements, machineries and work premises, actually and directly used by the contractor or subcontractor in the performance or completion of the job, work or service contracted out. The right to control shall refer to the right reserved to the person for whom the services of the contractual workers are performed, to determine not only the end to be achieved, but also the manner and means to be used in reaching that end. On the other hand, Sec. 7 of the department order contains the consequence of such labor-only contracting: Section 7. Existence of an employer-employee relationship.The contractor or subcontractor shall be considered the employer of the contractual employee for purposes of enforcing the provisions of the Labor Code and other social legislation. The principal, however, shall be solidarily liable with the contractor in the event of any violation of any provision of the Labor Code, including the failure to pay wages. The principal shall be deemed the employer of the contractual employee in any of the following cases as declared by a competent authority: (a) where there is labor-only contracting; or (b) where the contracting arrangement falls within the prohibitions provided in Section 6 (Prohibitions) hereof. (Emphasis supplied.) Evidently, respondent having the power of control over petitioners must be considered as petitioners employerfrom the termination of the Agreement onwardsas this was the only time that any evidence of control was exhibited by respondent over petitioners and in light of our ruling in Abella.54[12] Thus, as aptly declared by the NLRC, petitioners were entitled to the rights and benefits of employees of respondent, including due process requirements in the termination of their services. MASONIC CONTRACTOR, INC. and MELVIN BALAIS/AVELINO REYES Petitioners,- versus MAGDALENA MADJOS, ZENAIDA TIAMZON, and CARMELITA RAPADAS, Respondents. Facts: Respondents Magdalena Madjos, Zenaida Tiamzon and Carmelita Rapadas were employed sometime in 1991 as allaround laborers (driver/sweeper/ taga-libing/grass-cutter) by Masonic Contractor, Inc. (MCI). Each of them received an initial daily wage of P165.00 and were required to report for work from 7:00 a.m. to 4:00 p.m. Three years thereafter, MCI increased their wages by P15.00 per day55[5] but not without earning the ire of Melvin Balais, president of MCI.56[6. Sometime in 2004, Balais told Madjos, Tiamzon and Rapadas, along with nine (9) other employees, to take a two-day leave. When they reported for work two days thereafter, they were barred from entering the work premises and were informed that they had already been replaced by other workers. 57[7] This prompted Madjos and her co-workers to file a

54 55 56 57

Page 15 of 95

complaint against herein petitioners for illegal dismissal and for non-payment of overtime pay, holiday pay, 13 th month pay, and damages. Issue: WON employer-employee relationship existed (whether respondents are employees of MCI) Ruling: We answer in the affirmative. In Brotherhood Labor Unity Movement of the Philippines v. Hon. Zamora, the Court explained: In determining the existence of an employer-employee relationship, the elements that are generally considered are the following: (a) the selection and engagement of the employee; (b) the payment of wages; (c) the power of dismissal; and (d) the employers power to control the employee with respect to the means and methods by which the work is to be accomplished. It is the so-called control test that is the most important element.58[16] The existence of an employer-employee relationship is a question of fact which should be supported by substantial evidence.59[17] Petitioners defense that they merely contracted the services of respondents through Malibiran fails to persuade us. The facts of this case show that respondents have been under the employ of MCI as early as 1991. They were hired not to perform a specific job or undertaking. Instead, they were employed as all-around laborers doing varied and intermittent jobs, such as those of drivers, sweepers, gardeners, and even undertakers or tagalibing, until they were arbitrarily terminated by MCI in 2004. Their wages were paid directly by MCI, as evidenced by the latters payroll summary, 60[18] belying its self-serving and unsupported contention that it paid directly to Malibiran for respondents services. Respondents had identification cards or gate passes issued not by Malibiran, but by MCI, 61[19] and were required to wear uniforms bearing MCIs emblem or logo when they reported for work.62[20] It is common practice for companies to provide identification cards to individuals not only as a security measure, but more importantly to identify the bearers thereof as bona fide employees of the firm or institution that issued them. 63[21] The provision of company-issued identification cards and uniforms to respondents, aside from their inclusion in MCIs summary payroll, indubitably constitutes substantial evidence sufficient to support only one conclusion: that respondents were indeed employees of MCI. Moreover, as correctly observed by the CA, petitioners failed to show that it was Malibiran who exercised control over the means and methods of the work assigned to respondents. Interestingly, Malibirans affidavit is silent on the aspect of control over respondents means and methods of work. Rather than categorically stating that she was the one who directly employed respondents to render work for MCI, Malibiran merely implies that, like respondents, she was just a coworker. Malibirans statement that the work for MCI was merely in the nature of accommodation to help respondents

58 59 60 61 62 63

Page 16 of 95

earn a living, in effect, impliedly admits the fact that she did not have the capacity to engage in the independent jobcontracting business, and that, therefore, she was not respondents employer. With the issue of respondents employment resolved, we then declare that respondents were illegally terminated when petitioners summarily dismissed them from work without any valid reason for doing so and without observing procedural due process. We thus affirm the CAs finding that petitioners are liable for their unwarranted action against respondents. LESLIE OKOL, Petitioner, - versus -SLIMMERS WORLD INTERNATIONAL, BEHAVIOR MODIFICATIONS, INC., and RONALD JOSEPH MOY, Respondents. Facts: Respondent Slimmers World International operating under the name Behavior Modifications, Inc. (Slimmers World) employed petitioner Leslie Okol (Okol) as a management trainee on 15 June 1992. She rose up the ranks to become Head Office Manager and then Director and Vice President from 1996 until her dismissal on 22 September 1999. On 28 July 1999, prior to Okols dismissal, Slimmers World preventively suspended Okol. The suspension arose from the seizure by the Bureau of Customs of seven Precor elliptical machines and seven Precor treadmills belonging to or consigned to Slimmers World. The shipment of the equipment was placed under the names of Okol and two customs brokers for a value less than US$500. For being undervalued, the equipment were seized.On 2 September 1999, Okol received a memorandum that her suspension had been extended from 2 September until 1 October 1999 pending the outcome of the investigation on the Precor equipment importation.On 17 September 1999, Okol received another memorandum from Slimmers World requiring her to explain why no disciplinary action should be taken against her in connection with the equipment seized by the Bureau of Customs. On 19 September 1999, Okol filed her written explanation. However, Slimmers World found Okols explanation to be unsatisfactory. Through a letter dated 22 September 1999 signed by its president Ronald Joseph Moy (Moy), Slimmers World terminated Okols employment. Okol filed a complaint64[3] with the Arbitration branch of the NLRC against Slimmers World, Behavior Modifications, Inc. and Moy (collectively called respondents) for illegal suspension, illegal dismissal, unpaid commissions, damages and attorneys fees, with prayer for reinstatement and payment of backwages. Issue: WON employer-employee relationship existed Ruling: No employer-employee relationship existed. Okol is a corporate officer. NLRC has no jurisdiction. Petitioner insists that the Court of Appeals erred in ruling that she was a corporate officer and that the case is an intracorporate dispute falling within the jurisdiction of the regular courts. Petitioner asserts that even as vice-president, the work that she performed conforms to that of an employee rather than a corporate officer. Mere title or designation in a corporation will not, by itself, determine the existence of an employer-employee relationship. It is the four-fold test, namely (1) the power to hire, (2) the payment of wages, (3) the power to dismiss, and (4) the power to control, which must be applied. Petitioner enumerated the instances that she was under the power and control of Moy, Slimmers Worlds president: (1) petitioner received salary evidenced by pay slips, (2) Moy deducted Medicare and SSS benefits from petitioners salary, and (3) petitioner was dismissed from employment not through a board resolution but by virtue of a letter from Moy.

64

Page 17 of 95

Thus, having shown that an employer-employee relationship exists, the jurisdiction to hear and decide the case is vested with the labor arbiter and the NLRC. Respondents, on the other hand, maintain that petitioner was a corporate officer at the time of her dismissal from Slimmers World as supported by the General Information Sheet and Directors Affidavit attesting that petitioner was an officer. Also, the factors cited by petitioner that she was a mere employee do not prove that she was not an officer of Slimmers World. Even the alleged absence of any resolution of the Board of Directors approving petitioners termination does not constitute proof that petitioner was not an officer. Respondents assert that petitioner was not only an officer but also a stockholder and director; which facts provide further basis that petitioners separation from Slimmers World does not come under the NLRCs jurisdiction. The issue revolves mainly on whether petitioner was an employee or a corporate officer of Slimmers World. Section 25 of the Corporation Code enumerates corporate officers as the president, secretary, treasurer and such other officers as may be provided for in the by-laws. In Tabang v. NLRC,65[12] we held that an office is created by the charter of the corporation and the officer is elected by the directors or stockholders. On the other hand, an employee usually occupies no office and generally is employed not by action of the directors or stockholders but by the managing officer of the corporation who also determines the compensation to be paid to such employee. In the present case, the respondents, in their motion to dismiss filed before the labor arbiter, questioned the jurisdiction of the NLRC in taking cognizance of petitioners complaint. In the motion, respondents attached the General Information Sheet66[13] (GIS) dated 14 April 1998, Minutes 67[14] of the meeting of the Board of Directors dated 14 April 1997 and Secretarys Certificate,68[15] and the Amended By-Laws69[16] dated 1 August 1994 of Slimmers World as submitted to the SEC to show that petitioner was a corporate officer whose rights do not fall within the NLRCs jurisdiction. The GIS and minutes of the meeting of the board of directors indicated that petitioner was a member of the board of directors, holding one subscribed share of the capital stock, and an elected corporate officer. The relevant portions of the Amended By-Laws of Slimmers World which enumerate the power of the board of directors as well as the officers of the corporation state: Article II The Board of Directors 1. Qualifications and Election The general management of the corporation shall be vested in a board of five directors who shall be stockholders and who shall be elected annually by the stockholders and who shall serve until the election and qualification of their successors.x x x Article III Officersx x x 4. Vice-President Like the Chairman of the Board and the President, the Vice-President shall be elected by the Board of Directors from [its] own members.

65 66 67 68 69

Page 18 of 95

The Vice-President shall be vested with all the powers and authority and is required to perform all the duties of the President during the absence of the latter for any cause. The Vice-President will perform such duties as the Board of Directors may impose upon him from time to time.x x x Clearly, from the documents submitted by respondents, petitioner was a director and officer of Slimmers World. The charges of illegal suspension, illegal dismissal, unpaid commissions, reinstatement and back wages imputed by petitioner against respondents fall squarely within the ambit of intra-corporate disputes. In a number of cases, 70[17] we have held that a corporate officers dismissal is always a corporate act, or an intra-corporate controversy which arises between a stockholder and a corporation. The question of remuneration involving a stockholder and officer, not a mere employee, is not a simple labor problem but a matter that comes within the area of corporate affairs and management and is a corporate controversy in contemplation of the Corporation Code.71[18] Prior to its amendment, Section 5(c) of Presidential Decree No. 902-A72[19] (PD 902-A) provided that intra-corporate disputes fall within the jurisdiction of the Securities and Exchange Commission (SEC) Sec. 5. In addition to the regulatory and adjudicative functions of the Securities and Exchange Commission over corporations, partnerships and other forms of associations registered with it as expressly granted under existing laws and decrees, it shall have original and exclusive jurisdiction to hear and decide cases involving: xxx c) Controversies in the election or appointments of directors, trustees, officers or managers of such corporations, partnerships or associations. Subsection 5.2, Section 5 of Republic Act No. 8799, which took effect on 8 August 2000, transferred to regional trial courts the SECs jurisdiction over all cases listed in Section 5 of PD 902-A: 5.2. The Commissions jurisdiction over all cases enumerated under Section 5 of Presidential Decree No. 902-A is hereby transferred to the Courts of general jurisdiction or the appropriate Regional Trial Court.x x x It is a settled rule that jurisdiction over the subject matter is conferred by law. 73[20] The determination of the rights of a director and corporate officer dismissed from his employment as well as the corresponding liability of a corporation, if any, is an intra-corporate dispute subject to the jurisdiction of the regular courts. Thus, the appellate court correctly ruled that it is not the NLRC but the regular courts which have jurisdiction over the present case.

Topic 2. Classification of employees a. Regular vs casual employees

70 71 72 73

Page 19 of 95

LEYTE GEOTHERMAL POWER PROGRESSIVE EMPLOYEES UNION - ALU - TUCP, Petitioner,vs. PHILIPPINE NATIONAL OIL COMPANY - ENERGY DEVELOPMENT CORPORATION, Respondent.

Facts: [Respondent Philippine National Oil Corporation]-Energy Development Corporation [PNOC-EDC] is a government-owned and controlled corporation engaged in exploration, development, utilization, generation and distribution of energy resources like geothermal energy.

Petitioner is a legitimate labor organization, duly registered with the Department of Labor and Employment (DOLE) Regional Office No. VIII, Tacloban City.

Among [respondents] geothermal projects is the Leyte Geothermal Power Project located at the Greater Tongonan Geothermal Reservation in Leyte. The said Project is composed of the Tongonan 1 Geothermal Project (T1GP) and the Leyte Geothermal Production Field Project (LGPF) which provide the power and electricity needed not only in the provinces and cities of Central and Eastern Visayas (Region VII and VIII), but also in the island of Luzon as well. Thus, the [respondent] hired and employed hundreds of employees on a contractual basis, whereby, their employment was only good up to the completion or termination of the project and would automatically expire upon the completion of such project.

On December 28, 1998, the petitioner filed a Notice of Strike with DOLE against the [respondent] on the ground of purported commission by the latter of unfair labor practice for "refusal to bargain collectively, union busting and mass termination." On the same day, the petitioner declared a strike and staged such strike. To avert any work stoppage, then Secretary of Labor Bienvenido E. Laguesma intervened and issued the Order, dated January 4, 1999, certifying the labor dispute to the NLRC for compulsory arbitration. Accordingly, all the striking workers were directed to return to work within twelve (12) hours from receipt of the Order and for the [respondent] to accept them back under the same terms and conditions of employment prior to the strike. Further, the parties were directed to cease and desist from committing any act that would exacerbate the situation. However, despite earnest efforts on the part of the Secretary of Labor and Employment to settle the dispute amicably, the petitioner remained adamant and unreasonable in its position, causing the failure of the negotiation towards a peaceful compromise. In effect, the petitioner did not abide by [the] assumption order issued by the Secretary of Labor. Consequently, on January 15, 1999, the [respondent] filed a Complaint for Strike Illegality, Declaration of Loss of Employment and Damages at the NLRC-RAB VIII in Tacloban City and at the same time, filed a Petition for Cancellation of Petitioners Certificate of Registration with DOLE, Regional Office No. VIII. The two cases were later on consolidated pursuant to the New NLRC Rules of Procedure. The consolidated case was docketed as NLRC Certified Case No. V-02-99 (NCMB-RAB VIII-NS-12-0190-98; RAB Case No. VIII-1-0019-99). The said certified case was indorsed to the NLRC 4th Division in Cebu City on June 21, 1999 for the proper disposition thereof.3

Page 20 of 95

Issue:

Whether the officers and members of petitioner Union are project employees of respondent

Ruling: They are project employees SC held. We cannot subscribe to the view taken by petitioner Union. The distinction between a regular and a project employment is provided in Article 280, paragraph 1, of the Labor Code: ART. 280. Regular and Casual Employment. The provisions of written agreement to the contrary notwithstanding and regardless of the oral agreement of the parties, an employment shall be deemed to be regular where the employee has been engaged to perform activities which are usually necessary or desirable in the usual business or trade of the employer, except where the employment has been fixed for a specific project or undertaking the completion or termination of which has been determined at the time of the engagement of the employee or where the work or service to be performed is seasonal in nature and the employment is for the duration of the season. An employment shall be deemed to be casual if it is not covered by the preceding paragraph: Provided, That, any employee who has rendered at least one year of service, whether such service is continuous or broken, shall be considered a regular employee with respect to the activity in which he is employed and his employment shall continue while such actually exists.7 The foregoing contemplates four (4) kinds of employees: (a) regular employees or those who have been "engaged to perform activities which are usually necessary or desirable in the usual business or trade of the employer"; (b) project employees or those "whose employment has been fixed for a specific project or undertaking[,] the completion or termination of which has been determined at the time of the engagement of the employee"; (c) seasonal employees or those who work or perform services which are seasonal in nature, and the employment is for the duration of the season; 8 and (d) casual employees or those who are not regular, project, or seasonal employees. Jurisprudence has added a fifth kind a fixed-term employee.9 Article 280 of the Labor Code, as worded, establishes that the nature of the employment is determined by law, regardless of any contract expressing otherwise. The supremacy of the law over the nomenclature of the contract and the stipulations contained therein is to bring to life the policy enshrined in the Constitution to "afford full protection to labor."10 Thus, labor contracts are placed on a higher plane than ordinary contracts; these are imbued with public interest and therefore subject to the police power of the State.11 However, notwithstanding the foregoing iterations, project employment contracts which fix the employment for a specific project or undertaking remain valid under the law: x x x By entering into such a contract, an employee is deemed to understand that his employment is coterminous with the project. He may not expect to be employed continuously beyond the completion of the project. It is of judicial notice that project employees engaged for manual services or those for special skills like those of carpenters or masons, are, as a rule, unschooled. However, this fact alone is not a valid reason for bestowing special treatment on them or for invalidating a contract of employment. Project employment contracts are not lopsided agreements in favor of only one party thereto. The employers interest is equally important as that of the employee[s] for theirs is the interest that propels economic activity. While it may be true that it is the employer who drafts project employment contracts with its

Page 21 of 95

business interest as overriding consideration, such contracts do not, of necessity, prejudice the employee. Neither is the employee left helpless by a prejudicial employment contract. After all, under the law, the interest of the worker is paramount.12 In the case at bar, the records reveal that the officers and the members of petitioner Union signed employment contracts indicating the specific project or phase of work for which they were hired, with a fixed period of employment. The NLRC correctly disposed of this issue: A deeper examination also shows that [the individual members of petitioner Union] indeed signed and accepted the [employment contracts] freely and voluntarily. No evidence was presented by [petitioner] Union to prove improper pressure or undue influence when they entered, perfected and consummated [the employment] contracts. In fact, it was clearly established in the course of the trial of this case, as explained by no less than the President of [petitioner] Union, that the contracts of employment were read, comprehended, and voluntarily accepted by them. x x x. xxxx As clearly shown by [petitioner] Unions own admission, both parties had executed the contracts freely and voluntarily without force, duress or acts tending to vitiate the worker[s] consent. Thus, we see no reason not to honor and give effect to the terms and conditions stipulated therein. x x x.13 Thus, we are hard pressed to find cause to disturb the findings of the NLRC which are supported by substantial evidence. It is well-settled in jurisprudence that factual findings of administrative or quasi-judicial bodies, which are deemed to have acquired expertise in matters within their respective jurisdictions, are generally accorded not only respect but even finality, and bind the Court when supported by substantial evidence.14 Rule 133, Section 5 defines substantial evidence as "that amount of relevant evidence which a reasonable mind might accept as adequate to justify a conclusion." Consistent therewith is the doctrine that this Court is not a trier of facts, and this is strictly adhered to in labor cases. 15 We may take cognizance of and resolve factual issues, only when the findings of fact and conclusions of law of the Labor Arbiter or the NLRC are inconsistent with those of the CA.16 In the case at bar, both the NLRC and the CA were one in the conclusion that the officers and the members of petitioner Union were project employees. Nonetheless, petitioner Union insists that they were regular employees since they performed work which was usually necessary or desirable to the usual business or trade of the Construction Department of respondent. The landmark case of ALU-TUCP v. NLRC17 instructs on the two (2) categories of project employees: It is evidently important to become clear about the meaning and scope of the term "project" in the present context. The "project" for the carrying out of which "project employees" are hired would ordinarily have some relationship to the usual business of the employer. Exceptionally, the "project" undertaking might not have an ordinary or normal relationship to the usual business of the employer. In this latter case, the determination of the scope and parameters of the "project" becomes fairly easy. x x x. From the viewpoint, however, of the legal characterization problem here presented to the Court, there should be no difficulty in designating the employees who are retained or hired for the purpose of undertaking fish culture or the production of vegetables as "project employees," as distinguished from ordinary or "regular employees," so long as the duration and scope of the project were determined or specified at the time of engagement of the "project employees." For, as is evident from the provisions of Article 280 of the Labor Code, quoted earlier, the principal test for determining whether particular employees are properly characterized as "project employees" as distinguished from "regular employees," is whether or not the "project employees" were assigned to carry out a "specific project or undertaking," the duration (and scope) of which were specified at the time the employees were engaged for that project.

Page 22 of 95

In the realm of business and industry, we note that "project" could refer to one or the other of at least two (2) distinguishable types of activities. Firstly, a project could refer to a particular job or undertaking that is within the regular or usual business of the employer company, but which is distinct and separate, and identifiable as such, from the other undertakings of the company. Such job or undertaking begins and ends at determined or determinable times. The typical example of this first type of project is a particular construction job or project of a construction company. A construction company ordinarily carries out two or more [distinct] identifiable construction projects: e.g., a twenty-five-storey hotel in Makati; a residential condominium building in Baguio City; and a domestic air terminal in Iloilo City. Employees who are hired for the carrying out of one of these separate projects, the scope and duration of which has been determined and made known to the employees at the time of employment, are properly treated as "project employees," and their services may be lawfully terminated at completion of the project. The term "project" could also refer to, secondly, a particular job or undertaking that is not within the regular business of the corporation. Such a job or undertaking must also be identifiably separate and distinct from the ordinary or regular business operations of the employer. The job or undertaking also begins and ends at determined or determinable times.18 Plainly, the litmus test to determine whether an individual is a project employee lies in setting a fixed period of employment involving a specific undertaking which completion or termination has been determined at the time of the particular employees engagement. In this case, as previously adverted to, the officers and the members of petitioner Union were specifically hired as project employees for respondents Leyte Geothermal Power Project located at the Greater Tongonan Geothermal Reservation in Leyte. Consequently, upon the completion of the project or substantial phase thereof, the officers and the members of petitioner Union could be validly terminated. Petitioner Union is adamant, however, that the lack of interval in the employment contracts of its officer and members negates the latters status as mere project employees. For petitioner Union, the lack of interval further drives home its point that its officers and members are regular employees who performed work which was usually necessary or desirable to the usual business or trade of respondent. We are not persuaded. Petitioner Unions members employment for more than a year does equate to their regular employment with respondent. In this regard, Mercado, Sr. v. NLRC19 illuminates: The first paragraph [of Article 280 of the Labor Code] answers the question of who are regular employees. It states that, regardless of any written or oral agreement to the contrary, an employee is deemed regular where he is engaged in necessary or desirable activities in the usual business or trade of the employer, except for project employees. A project employee has been defined to be one whose employment has been fixed for a specific project or undertaking, the completion or termination of which has been determined at the time of the engagement of the employee, or where the work or service to be performed is seasonal in nature and the employment is for the duration of the season, as in the present case. The second paragraph of Art. 280 demarcates as "casual" employees, all other employees who do not fall under the definition of the preceding paragraph. The proviso, in said second paragraph, deems as regular employees those "casual" employees who have rendered at least one year of service regardless of the fact that such service may be continuous or broken. Petitioners, in effect, contend that the proviso in the second paragraph of Art. 280 is applicable to their case and that the Labor Arbiter should have considered them regular by virtue of said proviso. The contention is without merit. The general rule is that the office of a proviso is to qualify or modify only the phrase immediately preceding it or restrain or limit the generality of the clause that it immediately follows. Thus, it has been held that a proviso is to be construed

Page 23 of 95

with reference to the immediately preceding part of the provision to which it is attached, and not to the statute itself or to other sections thereof. The only exception to this rule is where the clear legislative intent is to restrain or qualify not only the phrase immediately preceding it (the proviso) but also earlier provisions of the statute or even the statute itself as a whole. Policy Instruction No. 12 of the Department of Labor and Employment discloses that the concept of regular and casual employees was designed to put an end to casual employment in regular jobs, which has been abused by many employers to prevent so called casuals from enjoying the benefits of regular employees or to prevent casuals from joining unions. The same instructions show that the proviso in the second paragraph of Art. 280 was not designed to stifle small-scale businesses nor to oppress agricultural land owners to further the interests of laborers, whether agricultural or industrial. What it seeks to eliminate are abuses of employers against their employees and not, as petitioners would have us believe, to prevent small-scale businesses from engaging in legitimate methods to realize profit. Hence, the proviso is applicable only to the employees who are deemed "casuals" but not to the "project" employees nor the regular employees treated in paragraph one of Art. 280. Clearly, therefore, petitioners being project employees, or, to use the correct term, seasonal employees, their employment legally ends upon completion of the project or the [end of the] season. The termination of their employment cannot and should not constitute an illegal dismissal. Considering our holding that the officers and the members of petitioner Union were project employees, its claim of union busting is likewise dismissed. SAN MIGUEL CORPORATION Petitioner,- versus -EDUARDO L. TEODOSIO, Respondent. Facts: On September 5, 1991, respondent Eduardo Teodosio was hired by San Miguel Corporation (SMC) as a casual forklift operator in its Bacolod City Brewery. 74[3] As a forklift operator, respondent was tasked with loading and unloading pallet75[4] of beer cases within the brewery premises. Respondent continuously worked from September 5, 1991 until March 1992, after which he was asked to rest for a while. A month after, or sometime in April 1992, respondent was rehired for the same position, and after serving for about five to six months, he was again asked to rest. After three weeks, he was again rehired as a forklift operator. He continued to work as such until August 1993.76[5] Sometime in August 1993, respondent was made to sign an Employment with a Fixed Period77[6] contract by SMC, wherein it was stipulated, among other things, that respondents employment would be from August 7, 1993 to August 30, 1995, or upon cessation of the instability/fluctuation of the market demand, whichever comes first. Thereafter, respondent worked at the plant without interruption as a forklift operator. On March 20, 1995, respondent was transferred to the plants bottling section as a case piler. In a letter 78[7] dated April 10, 1995, respondent formally informed SMC of his opposition to his transfer to the bottling section. He asserted that he would be more effective as a forklift operator because he had been employed as such for more than three years already. Respondent also requested that he be transferred to his former position as a forklift operator. However, SMC did not answer his letter.

74 75 76 77 78

Page 24 of 95

In an undated letter,79[8] respondent informed SMC that he was applying for the vacant position of bottling crew as he was interested in becoming a regular employee of SMC. On June 1, 1995, SMC notified the respondent that his employment shall be terminated on July 1, 1995 in compliance with the Employment with a Fixed Period contract.80[9] SMC explained that this was due to the reorganization and streamlining of its operations. Issue: WON the respondent is a casual or regular employee Ruling: Respondent is a REGULAR employee. This Court finds the respondent to be a regular employee. Article 280 of the Labor Code, as amended, provides: ART. 280. REGULAR AND CASUAL EMPLOYMENT. - The provisions of written agreement to the contrary notwithstanding and regardless of the oral agreement of the parties, an employment shall be deemed to be regular where the employee has been engaged to perform activities which are usually necessary or desirable in the usual business or trade of the employer, except where the employment has been fixed for a specific project or undertaking, the completion or termination of which has been determined at the time of the engagement of the employee or where the work or services to be performed is seasonal in nature and the employment is for the duration of the season. An employment shall be deemed to be casual if it is not covered by the preceding paragraph: Provided, That, any employee who has rendered at least one year of service, whether such service is continuous or broken, shall be considered a regular employee with respect to the activity in which he is employed and his employment shall continue while such activity exists. Thus, there are two kinds of regular employees, namely: (1) those who are engaged to perform activities which are usually necessary or desirable in the usual business or trade of the employer; and (2) those who have rendered at least one year of service, whether continuous or broken, with respect to the activity in which they are employed.81[23] Simply stated, regular employees are classified into (1) regular employees - by nature of work and (2) regular employees - by years of service. The former refers to those employees who perform a particular activity which is necessary or desirable in the usual business or trade of the employer, regardless of their length of service; while the latter refers to those employees who have been performing the job, regardless of the nature thereof, for at least a year. 82[24] If the employee has been performing the job for at least one year, even if the performance is not continuous or merely intermittent, the law deems the repeated and continuing need for its performance as sufficient evidence of the necessity, if not indispensability, of that activity to the business.83[25] Based on the circumstances surrounding respondents employment by SMC, this Court is convinced that he has attained the status of a regular employee long before he executed the employment contract with a fixed period. Although respondent was initially hired by SMC as a casual employee, respondent has attained the status of a regular employee.

79 80 81 82 83

Page 25 of 95

Respondent was initially hired by SMC on September 5, 1991 until March 1992. He was rehired for the same position in April 1992 which lasted for five to six months. After three weeks, he was again rehired as a forklift operator and he continued to work as such until August 1993. Thus, at the time he signed the Employment with a Fixed Period contract, respondent had already been in the employ of SMC for at least twenty-three (23) months. The Labor Code provides that a casual employee can be considered as a regular employee if said casual employee has rendered at least one year of service regardless of the fact that such service may be continuous or broken. Section 3, Rule V, Book II of the Implementing Rules and Regulations of the Labor Code clearly defines the term at least one year of service to mean service within 12 months, whether continuous or broken, reckoned from the date the employee started working, including authorized absences and paid regular holidays, unless the working days in the establishment, as a matter of practice or policy, or as provided in the employment contract, is less than 12 months, in which case said period shall be considered one year. If the employee has been performing the job for at least one year, even if the performance is not continuous or merely intermittent, the law deems the repeated and continuing need for its performance as sufficient evidence of the necessity, if not indispensability, of that activity to the business of the employer.84[26] Moreover, the nature of respondents work is necessary in the business in which SMC is engaged. SMC is primarily engaged in the manufacture and marketing of beer products, for which purpose, it specifically maintains a brewery in Bacolod City.85[27] Respondent, on the other hand, was engaged as a forklift operator tasked to lift and transfer pallets and pile them from the bottling section to the piling area. SMC admitted that it hired respondent as a forklift operator since the third quarter of 1991 when, in the absence of fully automated palletizers, manual transfers of beer cases and empties would be extensive within the brewery and its premises. SMC would have wanted this Court to believe that circumstances have transpired to force it to implement full automation of its brewery and new marketing and distribution systems in its sales offices resulting in the reduction of personnel and termination of employees with a fixed period contract. However, even after the installation of the automated palletizers, SMC did not leave the position of forklift operator vacant. SMC even transferred one of its regular employees to the Bacolod City Brewery to replace respondent who was in turn transferred to the bottling section of the plant. This demonstrates the continuing necessity and indispensability of hiring a forklift operator to the business of SMC. Undoubtedly, respondent is a regular employee of SMC. Consequently, the employment contract with a fixed period which SMC had respondent execute was meant only to circumvent respondents right to security of tenure and is, therefore, invalid. PLT vs Arceo Facts: In May 1990, respondent Rosalina Arceo (Arceo) applied for the position of telephone operator with petitioner Philippine Long Distance Telephone Company, Inc. Tarlac Exchange (PLDT). She, however, failed the pre-employment qualifying examination. Having failed the test, Arceo requested PLDT to allow her to work at the latters office even without pay. PLDT agreed and assigned her to its commercial section where she was made to perform various tasks like photocopying documents, sorting out telephone bills and notices of disconnection, and other minor assignments and activities. After two weeks, PLDT decided to pay her the minimum wage. On February 15, 1991, PLDT saw no further need for Arceos services and decided to fire her but, through the intervention of PLDTs commercial section supervisor, Mrs. Beatriz Mataguihan, she was recommended for an on-the-job training on minor traffic work. When she failed to assimilate traffic procedures, the company transferred her to auxiliary services, a minor facility.

84 85

Page 26 of 95

Subsequently, Arceo took the pre-qualifying exams for the position of telephone operator two more times but again failed in both attempts. Finally, on October 13, 1991, PLDT discharged Arceo from employment. She then filed a case for illegal dismissal before the labor arbiter.86[2] On May 11, 1993, the arbiter ruled in her favor. PLDT was ordered to reinstate Arceo to her former position or to an equivalent position. This decision became final and executory. On June 9, 1993, Arceo was reinstated as casual employee with a minimum wage of P106 per day. She was assigned to photocopy documents and sort out telephone bills. On September 3, 1996 or more than three years after her reinstatement, Arceo filed a complaint for unfair labor practice, underpayment of salary, underpayment of overtime pay, holiday pay, rest day pay and other monetary claims. She alleged in her complaint that, since her reinstatement, she had yet to be regularized and had yet to receive the benefits due to a regular employee. Issue: WON arceo is a regular employee of PLT (is Arceo eligible to become a regular employee of PLDT?) Ruling: Yes. Article 280 of the Labor Code, as amended, provides: Art. 280. Regular and Casual Employment. The provisions of written agreement to the contrary notwithstanding and regardless of the oral agreement of the parties, an employment shall be deemed to be regular where the employee has been engaged to perform activities which are usually necessary or desirable in the usual business or trade of the employer , except where the employment has been fixed for a specific project or undertaking the completion or termination of which has been determined at the time of engagement of the employee or where the work or services to be performed is seasonal in nature and employment is for the duration of the season. An employment shall be deemed to be casual if it employee who has rendered at least one year shall be considered a regular employee with employment shall continue while such activity is not covered by the preceding paragraph. Provided, that, any of service, whether such service is continuous or broken, respect to the activity in which he is employed and his exists . (emphasis ours)

Under the foregoing provision, a regular employee is (1) one who is either engaged to perform activities that are necessary or desirable in the usual trade or business of the employer or (2) a casual employee who has rendered at least one year of service, whether continuous or broken, with respect to the activity in which he is employed. Under the first criterion, respondent is qualified to be a regular employee. Her work, consisting mainly of photocopying documents, sorting out telephone bills and disconnection notices, was certainly necessary or desirable to the business of PLDT. But even if the contrary were true, the uncontested fact is that she rendered service for more than one year as a casual employee. Hence, under the second criterion, she is still eligible to become a regular employee. Petitioners argument that respondents position has been abolished, if indeed true, does not preclude Arceos becoming a regular employee. The order to reinstate her also included the alternative to reinstate her to a position equivalent thereto. Thus, PLDT can still regularize her in an equivalent position.

86

Page 27 of 95

Moreover, PLDTs argument does not hold water in the absence of proof that the activity in which Arceo was engaged (like photocopying of documents and sorting of telephone bills) no longer subsists. Under Article 280, any employee who has rendered at least one year of service shall be considered a regular employee with respect to the activity in which he is employed and his employment shall continue while such activity exists. For PLDTs failure to show that the activity undertaken by Arceo has been discontinued, we are constrained to confirm her regularization in that position. From what date will she be entitled to the benefits of a regular employee? Considering that she has already worked in PLDT for more than one year at the time she was reinstated, she should be entitled to all the benefits of a regular employee from June 9, 1993 the day of her actual reinstatement. PLDTs other contention that the regularization of respondent as telephone operator was not possible since she failed in three qualifying exams for that position is also untenable. It is understood that she will be regularized in the position she held prior to the filing of her complaint with the labor arbiter, or, if that position was already abolished, to an equivalent position. The position of telephone operator was never even considered in any of the assailed decisions of the labor arbiter, the NLRC or the CA. ABS- CBN BROADCASTING CORPORATION, petitioner,vs. MARLYN JENNIFER DEIPARINE, and JOSEPHINE LERASAN, respondents. Facts: Petitioner ABS-CBN Broadcasting Corporation (ABS-CBN) is engaged in the broadcasting business and owns a network of television and radio stations, whose operations revolve around the broadcast, transmission, and relay of telecommunication signals. It sells and deals in or otherwise utilizes the airtime it generates from its radio and television operations. It has a franchise as a broadcasting company, and was likewise issued a license and authority to operate by the National Telecommunications Commission. Petitioner employed respondents Nazareno, Gerzon, Deiparine, and Lerasan as production assistants (PAs) on different dates. They were assigned at the news and public affairs, for various radio programs in the Cebu Broadcasting Station, with a monthly compensation of P4,000. They were issued ABS-CBN employees identification cards and were required to work for a minimum of eight hours a day, including Sundays and holidays. They were made to perform the following tasks and duties: a) Prepare, arrange airing of commercial broadcasting based on the daily operations log and digicart of respondent ABSCBN; b) Coordinate, arrange personalities for air interviews; c) Coordinate, prepare schedule of reporters for scheduled news reporting and lead-in or incoming reports; d) Facilitate, prepare and arrange airtime schedule for public service announcement and complaints; e) Assist, anchor program interview, etc; and f) Record, log clerical reports, man based control radio.4 Issue: WON the respondents are regular employees of ABS CBN Ruling: NAZARENO, MERLOU GERZON,

Page 28 of 95

The respondents are regular employees of ABS CBN. The question of whether respondents are regular or project employees or independent contractors is essentially factual in nature; nonetheless, the Court is constrained to resolve it due to its tremendous effects to the legions of production assistants working in the Philippine broadcasting industry. We agree with respondents contention that where a person has rendered at least one year of service, regardless of the nature of the activity performed, or where the work is continuous or intermittent, the employment is considered regular as long as the activity exists, the reason being that a customary appointment is not indispensable before one may be formally declared as having attained regular status. Article 280 of the Labor Code provides: ART. 280. REGULAR AND CASUAL EMPLOYMENT.The provisions of written agreement to the contrary notwithstanding and regardless of the oral agreement of the parties, an employment shall be deemed to be regular where the employee has been engaged to perform activities which are usually necessary or desirable in the usual business or trade of the employer except where the employment has been fixed for a specific project or undertaking the completion or termination of which has been determined at the time of the engagement of the employee or where the work or services to be performed is seasonal in nature and the employment is for the duration of the season. In Universal Robina Corporation v. Catapang,31 the Court reiterated the test in determining whether one is a regular employee: The primary standard, therefore, of determining regular employment is the reasonable connection between the particular activity performed by the employee in relation to the usual trade or business of the employer. The test is whether the former is usually necessary or desirable in the usual business or trade of the employer. The connection can be determined by considering the nature of work performed and its relation to the scheme of the particular business or trade in its entirety. Also, if the employee has been performing the job for at least a year, even if the performance is not continuous and merely intermittent, the law deems repeated and continuing need for its performance as sufficient evidence of the necessity if not indispensability of that activity to the business. Hence, the employment is considered regular, but only with respect to such activity and while such activity exists.32 As elaborated by this Court in Magsalin v. National Organization of Working Men:33 Even while the language of law might have been more definitive, the clarity of its spirit and intent, i.e., to ensure a "regular" workers security of tenure, however, can hardly be doubted. In determining whether an employment should be considered regular or non-regular, the applicable test is the reasonable connection between the particular activity performed by the employee in relation to the usual business or trade of the employer. The standard, supplied by the law itself, is whether the work undertaken is necessary or desirable in the usual business or trade of the employer, a fact that can be assessed by looking into the nature of the services rendered and its relation to the general scheme under which the business or trade is pursued in the usual course. It is distinguished from a specific undertaking that is divorced from the normal activities required in carrying on the particular business or trade. But, although the work to be performed is only for a specific project or seasonal, where a person thus engaged has been performing the job for at least one year, even if the performance is not continuous or is merely intermittent, the law deems the repeated and continuing need for its performance as being sufficient to indicate the necessity or desirability of that activity to the business or trade of the employer. The employment of such person is also then deemed to be regular with respect to such activity and while such activity exists.34 Not considered regular employees are "project employees," the completion or termination of which is more or less determinable at the time of employment, such as those employed in connection with a particular construction project, and "seasonal employees" whose employment by its nature is only desirable for a limited period of time. Even then, any employee who has rendered at least one year of service, whether continuous or intermittent, is deemed regular with respect to the activity performed and while such activity actually exists.

Page 29 of 95

It is of no moment that petitioner hired respondents as "talents." The fact that respondents received pre-agreed "talent fees" instead of salaries, that they did not observe the required office hours, and that they were permitted to join other productions during their free time are not conclusive of the nature of their employment. Respondents cannot be considered "talents" because they are not actors or actresses or radio specialists or mere clerks or utility employees. They are regular employees who perform several different duties under the control and direction of ABS-CBN executives and supervisors. Thus, there are two kinds of regular employees under the law: (1) those engaged to perform activities which are necessary or desirable in the usual business or trade of the employer; and (2) those casual employees who have rendered at least one year of service, whether continuous or broken, with respect to the activities in which they are employed.35 The law overrides such conditions which are prejudicial to the interest of the worker whose weak bargaining situation necessitates the succor of the State. What determines whether a certain employment is regular or otherwise is not the will or word of the employer, to which the worker oftentimes acquiesces, much less the procedure of hiring the employee or the manner of paying the salary or the actual time spent at work. It is the character of the activities performed in relation to the particular trade or business taking into account all the circumstances, and in some cases the length of time of its performance and its continued existence.36 It is obvious that one year after they were employed by petitioner, respondents became regular employees by operation of law.37 Additionally, respondents cannot be considered as project or program employees because no evidence was presented to show that the duration and scope of the project were determined or specified at the time of their engagement. Under existing jurisprudence, project could refer to two distinguishable types of activities. First, a project may refer to a particular job or undertaking that is within the regular or usual business of the employer, but which is distinct and separate, and identifiable as such, from the other undertakings of the company. Such job or undertaking begins and ends at determined or determinable times. Second, the term project may also refer to a particular job or undertaking that is not within the regular business of the employer. Such a job or undertaking must also be identifiably separate and distinct from the ordinary or regular business operations of the employer. The job or undertaking also begins and ends at determined or determinable times.38 The principal test is whether or not the project employees were assigned to carry out a specific project or undertaking, the duration and scope of which were specified at the time the employees were engaged for that project.39 In this case, it is undisputed that respondents had continuously performed the same activities for an average of five years. Their assigned tasks are necessary or desirable in the usual business or trade of the petitioner. The persisting need for their services is sufficient evidence of the necessity and indispensability of such services to petitioners business or trade.40 While length of time may not be a sole controlling test for project employment, it can be a strong factor to determine whether the employee was hired for a specific undertaking or in fact tasked to perform functions which are vital, necessary and indispensable to the usual trade or business of the employer. 41 We note further that petitioner did not report the termination of respondents employment in the particular "project" to the Department of Labor and Employment Regional Office having jurisdiction over the workplace within 30 days following the date of their separation from work, using the prescribed form on employees termination/ dismissals/suspensions.42 As gleaned from the records of this case, petitioner itself is not certain how to categorize respondents. In its earlier pleadings, petitioner classified respondents as program employees, and in later pleadings, independent contractors. Program employees, or project employees, are different from independent contractors because in the case of the latter, no employer-employee relationship exists. POSEIDON FISHING/TERRY DE JESUS, petitioners, vs.NATIONAL LABOR RELATIONS COMMISSION and JIMMY S. ESTOQUIA, Respondents. Facts:

Page 30 of 95

Petitioner Poseidon Fishing is a fishing company engaged in the deep-sea fishing industry. Its various vessels catch fish in the outlying islands of the Philippines, which are traded and sold at the Navotas Fish Port. One of its boat crew was private respondent Jimmy S. Estoquia.3 Petitioner Terry de Jesus is the manager of petitioner company. Private respondent was employed by Poseidon Fishing in January 1988 as Chief Mate. After five years, he was promoted to Boat Captain. In 1999, petitioners, without reason, demoted respondent from Boat Captain to Radio Operator of petitioner Poseidon.4 As a Radio Operator, he monitored the daily activities in their office and recorded in the duty logbook the names of the callers and time of their calls.5 On 3 July 2000, private respondent failed to record a 7:25 a.m. call in one of the logbooks. However, he was able to record the same in the other logbook. Consequently, when he reviewed the two logbooks, he noticed that he was not able to record the said call in one of the logbooks so he immediately recorded the 7:25 a.m. call after the 7:30 a.m. entry.6 Around 9:00 oclock in the morning of 4 July 2000, petitioner Terry de Jesus detected the error in the entry in the logbook. Subsequently, she asked private respondent to prepare an incident report to explain the reason for the said oversight.7 At around 2:00 oclock in the afternoon of that same day, petitioner Poseidons secretary, namely Nenita Laderas, summoned private respondent to get his separation pay amounting to Fifty-Five Thousand Pesos (P55,000.00). However, he refused to accept the amount as he believed that he did nothing illegal to warrant his immediate discharge from work.8 Rising to the occasion, private respondent filed a complaint for illegal dismissal on 11 July 2000 with the Labor Arbiter. Asserting their right to terminate the contract with private respondent per the "Kasunduan" with him, petitioners pointed to the provision thereof stating that he was being employed only on a por viaje basis and that his employment would be terminated at the end of the trip for which he was being hired, to wit: NA, kami ay sumasang-ayon na MAGLINGKOD at GUMAWA ng mga gawaing magmula sa pag-alis ng lantsa sa pondohan sa Navotas patungo sa palakayahan; pabalik sa pondohan ng lantsa sa Navotas hanggang sa paghango ng mga kargang isda.13 Petitioners lament that fixed-term employment contracts are recognized as valid under the law notwithstanding the provision of Article 280 of the Labor Code. Petitioners theorize that the Civil Code has always recognized the validity of contracts with a fixed and definite period, and imposes no restraints on the freedom of the parties to fix the duration of the contract, whatever its object, be it species, goods or services, except the general admonition against stipulations contrary to law, morals, good customs, public order and public policy. Quoting Brent School Inc. v. Zamora, 14 petitioners are hamstrung on their reasoning that under the Civil Code, fixed-term employment contracts are not limited, as they are under the present Labor Code, to those that by their nature are seasonal or for specific projects with pre-determined dates of completion as they also include those to which the parties by free choice have assigned a specific date of termination. Hence, persons may enter into such contracts as long as they are capacitated to act, petitioners bemoan. Issue: Was private respondent a regular employee at the time his employment was terminated on 04 July 2000? Ruling: Petitioners reliance in Brent vs Zamora is misplaced. Respondent is indeed a regular employee of the petitioner when he was terminated on July 4, 2000.

Page 31 of 95

Petitioners construal of Brent School, Inc. v. Zamora, has certainly gone astray. The subject of scrutiny in the Brent case was the employment contract inked between the school and one engaged as its Athletic Director. The contract fixed a specific term of five years from the date of execution of the agreement. This Court upheld the validity of the contract between therein petitioner and private respondent, fixing the latters period of employment. This Court laid down the following criteria for judging the validity of such fixed-term contracts, to wit: Accordingly, and since the entire purpose behind the development of legislation culminating in the present Article 280 of the Labor Code clearly appears to have been, as already observed, to prevent circumvention of the employees right to be secure in his tenure, the clause in said article indiscriminately and completely ruling out all written or oral agreements conflicting with the concept of regular employment as defined therein should be construed to refer to the substantive evil that the Code itself has singled out: agreements entered into precisely to circumvent security of tenure. It should have no application to instances where a fixed period of employment was agreed upon knowingly and voluntarily by the parties, without any force, duress or improper pressure being brought to bear upon the employee and absent any other circumstances vitiating his consent, or where it satisfactorily appears that the employer and employee dealt with each other on more or less equal terms with no moral dominance whatever being exercised by the former over the latter. Unless thus limited in its purview, the law would be made to apply to purposes other than those explicitly stated by its framers; it thus becomes pointless and arbitrary, unjust in its effects and apt to lead to absurd and unintended consequences.15 (Emphasis supplied.) Brent cited some familiar examples of employment contracts which may neither be for seasonal work nor for specific projects, but to which a fixed term is an essential and natural appurtenance, i.e., overseas employment contracts, appointments to the positions of dean, assistant dean, college secretary, principal, and other administrative offices in educational institutions, which are by practice or tradition rotated among the faculty members, and where fixed terms are a necessity without which no reasonable rotation would be possible.16 Thus, in Brent, the acid test in considering fixedterm contracts as valid is: if from the circumstances it is apparent that periods have been imposed to preclude acquisition of tenurial security by the employee, they should be disregarded for being contrary to public policy. On the same tack as Brent, the Court in Pakistan International Airlines Corporation v. Ople,17 ruled in this wise: It is apparent from Brent School that the critical consideration is the presence or absence of a substantial indication that the period specified in an employment agreement was designed to circumvent the security of tenure of regular employees which is provided for in Articles 280 and 281 of the Labor Code. This indication must ordinarily rest upon some aspect of the agreement other than the mere specification of a fixed term of the employment agreement, or upon evidence aliunde of the intent to evade. Consistent with the pronouncements in these two earlier cases, the Court, in Cielo v. National Labor Relations Commission,18 did not hesitate to nullify employment contracts stipulating a fixed term after finding that "the purpose behind these individual contracts was to evade the application of the labor laws." In the case under consideration, the agreement has such an objective - to frustrate the security of tenure of private respondent- and fittingly, must be nullified. In this case, petitioners intent to evade the application of Article 280 of the Labor Code is unmistakable. In a span of 12 years, private respondent worked for petitioner company first as a Chief Mate, then Boat Captain, and later as Radio Operator. His job was directly related to the deep-sea fishing business of petitioner Poseidon. His work was, therefore, necessary and important to the business of his employer. Such being the scenario involved, private respondent is considered a regular employee of petitioner under Article 280 of the Labor Code, the law in point, which provides: Art. 280. Regular and Casual Employment. The provisions of written agreement to the contrary notwithstanding and regardless of the oral agreement of the parties, an employment shall be deemed to be regular where the employee has been engaged to perform activities which are usually necessary or desirable in the usual business or trade of the employer, except where the employment has been fixed for a specific project or undertaking the completion or termination of which has been determined at the time of the engagement of the employee or where the work or services to be performed is seasonal in nature and the employment is for the duration of the season.

Page 32 of 95

An employment shall be deemed to be casual if it is not covered by the preceding paragraph: Provided, That any employee who has rendered at least one year of service, whether such service is continuous or broken, shall be considered a regular employee with respect to the activity in which he is employed and his employment shall continue while such actually exists. (Emphasis supplied.) Moreover, unlike in the Brent case where the period of the contract was fixed and clearly stated, note that in the case at bar, the terms of employment of private respondent as provided in the Kasunduan was not only vague, it also failed to provide an actual or specific date or period for the contract. As adroitly observed by the Labor Arbiter: There is nothing in the contract that says complainant, who happened to be the captain of said vessel, is a casual, seasonal or a project worker. The date July 1 to 31, 1998 under the heading "Pagdating" had been placed there merely to indicate the possible date of arrival of the vessel and is not an indication of the status of employment of the crew of the vessel. Actually, the exception under Article 280 of the Labor Code in which the respondents have taken refuge to justify its position does not apply in the instant case. The proviso, "Except where the employment has been fixed for a specific project or undertaking the completion or determination of which has been determined at the time of the engagement of the employee or where the work or services to be performed is seasonal in nature and the employment is for the duration of the season." (Article 280 Labor Code), is inapplicable because the very contract adduced by respondents is unclear and uncertain. The kasunduan does not specify the duration that complainant had been hired x x x.19 (Emphasis supplied.) Furthermore, as petitioners themselves admitted in their petition before this Court, private respondent was repeatedly hired as part of the boats crew and he acted in various capacities onboard the vessel. In Integrated Contractor and Plumbing Works, Inc. v. National Labor Relations Commission, 20 we held that the test to determine whether employment is regular or not is the reasonable connection between the particular activity performed by the employee in relation to the usual business or trade of the employer. And, if the employee has been performing the job for at least one year, even if the performance is not continuous or merely intermittent, the law deems the repeated and continuing need for its performance as sufficient evidence of the necessity, if not indispensability of that activity to the business. 21 In Bustamante v. National Labor Relations Commission,22 the Court expounded on what are regular employees under Article 280 of the Labor Code, viz: It is undisputed that petitioners were illegally dismissed from employment. Article 280 of the Labor Code, states: ART. 280. Regular and Casual Employment. - The provisions of written agreement to the contrary notwithstanding and regardless of the oral agreement of the parties, an employment shall be deemed to be regular where the employee has been engaged to perform activities which are usually necessary or desirable in the usual business or trade of the employer, except where the employment has been fixed for a specific project or undertaking the completion or termination of which has been determined at the time of the engagement of the employee or where the work or services to be performed is seasonal in nature and the employment is for the duration of the season. An employment shall be deemed to be casual if it is not covered by the preceding paragraph: Provided, that, any employee who has rendered at least one year of service, whether such service is continuous or broken, shall be considered a regular employee with respect to the activity in which he is employed and his employment shall continue while such activity exists. This provision draws a line between regular and casual employment, a distinction however often abused by employers. The provision enumerates two (2) kinds of employees, the regular employees and the casual employees. The regular employees consist of the following: 1) those engaged to perform activities which are usually necessary or desirable in the usual business or trade of the employer; and

Page 33 of 95

2) those who have rendered at least one year of service whether such service is continuous or broken.23 Ostensibly, in the case at bar, at different times, private respondent occupied the position of Chief Mate, Boat Captain, and Radio Operator. In petitioners interpretation, however, this act of hiring and re-hiring actually highlight private respondents contractual status saying that for every engagement, a fresh contract was entered into by the parties at the outset as the conditions of employment changed when the private respondent filled in a different position. But to this Court, the act of hiring and re-hiring in various capacities is a mere gambit employed by petitioner to thwart the tenurial protection of private respondent. Such pattern of re-hiring and the recurring need for his services are testament to the necessity and indispensability of such services to petitioners business or trade.24 Petitioners would brush off private respondents length of service by stating that he had worked for the company merely for several years25 and that in those times, his services were not exclusive to petitioners. On the other hand, to prove his claim that he had continuously worked for petitioners from 1988 to 2000, private respondent submitted a copy of his payroll26 from 30 May 1988 to October 1988 and a copy of his SSS Employees Contributions27 as of the year 2000. These documents were submitted by private respondent in order to benchmark his claim of 12 years of service. Petitioners, however, failed to submit the pertinent employee files, payrolls, records, remittances and other similar documents which would show that private respondents work was not continuous and for less than 12 years. Inasmuch as these documents are not in private respondents possession but in the custody and absolute control of petitioners, their failure to refute private respondents evidence or even categorically deny private respondents allegations lead us to no other conclusion than that private respondent was hired in 1988 and had been continuously in its employ since then. Indeed, petitioners failure to submit the necessary documents, which as employers are in their possession, gives rise to the presumption that their presentation is prejudicial to its cause.28 To recapitulate, it was after 12 long years of having private respondent under its wings when petitioners, possibly sensing a brewing brush with the law as far as private respondents employment is concerned, finally found a loophole to kick private respondent out when the latter failed to properly record a 7:25 a.m. call. Capitalizing on this faux pas, petitioner summarily dismissed private respondent. On this note, we disagree with the finding of the NLRC that private respondent was negligent on account of his failure to properly record a call in the log book. A review of the records would ineluctably show that there is no basis to deduct six months worth of salary from the total separation pay that private respondent is entitled to. We note further that the NLRCs finding clashes with that of the Labor Arbiter which found no such negligence and that such inadvertence on the part of private respondent, at best, constitutes simple negligence punishable only with admonition or suspension for a day or two. We are not won over. As correctly pointed out by the Court of Appeals, the "activity of catching fish is a continuous process and could hardly be considered as seasonal in nature."33 In Philex Mining Corp. v. National Labor Relations Commission,34 we defined project employees as those workers hired (1) for a specific project or undertaking, and (2) the completion or termination of such project has been determined at the time of the engagement of the employee. The principal test for determining whether particular employees are "project employees" as distinguished from "regular employees," is whether or not the "project employees" were assigned to carry out a "specific project or undertaking," the duration and scope of which were specified at the time the employees were engaged for that project. In this case, petitioners have not shown that private respondent was informed that he will be assigned to a "specific project or undertaking." As earlier noted, neither has it been established that he was informed of the duration and scope of such project or undertaking at the time of their engagement. More to the point, in Maraguinot, Jr. v. National Labor Relations Commission ,35 we ruled that once a project or work pool employee has been: (1) continuously, as opposed to intermittently, re-hired by the same employer for the same tasks or nature of tasks; and (2) these tasks are vital, necessary and indispensable to the usual business or trade of the employer, then the employee must be deemed a regular employee. In fine, inasmuch as private respondents functions as described above are no doubt "usually necessary or desirable in the usual business or trade" of petitioner fishing company and he was hired continuously for 12 years for the same nature

Page 34 of 95

of tasks, we are constrained to say that he belongs to the ilk of regular employee. Being one, private respondents dismissal without valid cause was illegal. And, where illegal dismissal is proven, the worker is entitled to back wages and other similar benefits without deductions or conditions.36 Indeed, it behooves this Court to be ever vigilant in checking the unscrupulous efforts of some of our entrepreneurs, primarily aimed at maximizing their return on investments at the expense of the lowly workingman.37 BIG AA MANUFACTURER, Petitioner, vs.EUTIQUIO ANTONIO, JAY ANTONIO, FELICISIMO ANTONIO, and LEONARDO ANTONIO, SR., * Respondents. Facts: Petitioner is a sole proprietorship registered in the name of its proprietor, Enrico E. Alejo, 5with office address at 311 Barrio Santol, Balagtas, Bulacan. On January 13, 2000, herein respondents Eutiquio Antonio,6Jay Antonio, Felicisimo Antonio, Leonardo Antonio, Sr. and Roberto Fabian filed a complaint for illegal lay-off and illegal deductions before the NLRCs Regional Arbitration Branch No. III. They claimed that they were dismissed on January 11, 2000 and sought separation pay from petitioner. When amicable settlement during the mandatory conference failed, the parties were required to file their position papers. The Labor Arbiter did not dismiss the complaint with respect to Roberto Fabian, despite his failure to file a position paper. Neither did the Labor Arbiters decision concern Roberto Fabian. Hence, this petition shall apply only to Eutiquio, Jay, Felicisimo, and Leonardo, Sr., all surnamed Antonio, the respondents herein. In respondents position paper,7they alleged that as regular employees, they worked from 8:00 a.m. to 5:00 p.m. at petitioners premises using petitioners tools and equipment and they received P250 per day. Eutiquio was employed as carpenter-foreman from 1991-1999; Jay as carpenter from 1993-1999; Felicisimo as carpenter from 1994-1999; and Leonardo, Sr. also as carpenter from 1997-1999. According to respondents, they were dismissed without just cause and due process; hence, their prayer for reinstatement and full backwages. They also impleaded one Hermie Alejo, a relative of the petitioners owner, as co-respondent in their complaint. Issue: WON the respondents are regular employees of the petitioner Ruling: The respondents are regular employees of the petitioner. Considering the submission of the parties, we are constrained to agree with the unanimous ruling of the Court of Appeals, NLRC and Labor Arbiter that respondents are petitioners regular employees. Respondents were employed for more than one year and their work as carpenters was necessary or desirable in petitioners usual trade or business of manufacturing office furniture. Under Article 280 of the Labor Code, the applicable test to determine whether an employment should be considered regular or non-regular is the reasonable connection between the particular activity performed by the employee in relation to the usual business or trade of the employer.20 True, certain forms of employment require the performance of usual or desirable functions and exceed one year but do not necessarily result to regular employment under Article 280 of the Labor Code.21Some specific exceptions include project or seasonal employment. Yet, in this case, respondents cannot be considered project employees. Petitioner had neither shown that respondents were hired for a specific project the duration of which was determined at the time of their hiring nor identified the specific project or phase thereof for which respondents were hired.

Page 35 of 95

We also agree that Eutiquio was not an independent contractor for he does not carry a distinct and independent business, and he does not possess substantial capital or investment in tools, equipment, machinery or work premises.22He works within petitioners premises using the latters tools and materials, as admitted by petitioner. Eutiquio is also under petitioners control and supervision. Attesting to this is petitioners admission that it allowed respondents to use its facilities for the "proper implementation" of job orders. Moreover, the Implementing Guidelines regulating attendance, overtime, deadlines, penalties; providing petitioners right to fire employees or "contractors"; requiring the carpentry division to join petitioners exercise program; and providing rules on machine maintenance, all reflect control and supervision over respondents. Petitioner likewise alleges that it did not dismiss respondents as they were not its regular employees; that respondents failed to sufficiently establish the fact of illegal dismissal; and that respondents abandoned the work after it issued the Implementing Guidelines.23 EFREN P. PAGUIO, petitioner, vs.NATIONAL LABOR RELATIONS COMMISSION, METROMEDIA TIMES CORPORATION, ROBINA Y. GOKONGWEI, LIBERATO GOMEZ, JR., YOLANDA E. ARAGON, FREDERICK D. GO and ALDA IGLESIA, respondents. Facts: On 22 June 1992, respondent Metromedia Times Corporation entered, for the fifth time, into an agreement with petitioner Efren P. Paguio, appointing the latter to be an account executive of the firm. 1 Again, petitioner was to solicit advertisements for "The Manila Times," a newspaper of general circulation, published by respondent company. Petitioner, for his efforts, was to receive compensation consisting of a 15% commission on direct advertisements less withholding tax and a 10% commission on agency advertisements based on gross revenues less agency commission and the corresponding withholding tax. The commissions, released every fifteen days of each month, were to be given to petitioner only after the clients would have paid for the advertisements. Apart from commissions, petitioner was also entitled to a monthly allowance of P2,000.00 as long as he met the P30,000.00-monthly quota. Basica lly, the contentious points raised by the parties had something to do with the following stipulations of the agreement; viz:"12. You are not an employee of the Metromedia Times Corporation nor does the company have any obligations towards anyone you may employ, nor any responsibility for your operating expenses or for any liability you may incur. The only rights and obligations between us are those set forth in this agreement. This agreement cannot be amended or modified in any way except with the duly authorized consent in writing of both parties. "13. Either party may terminate this agreement at any time by giving written notice to the other, thirty (30) days prior to effectivity of termination."2 On 15 August 1992, barely two months after the renewal of his contract, petitioner received the following notice from respondent firm "Dear Mr. Paguio, "Please be advised of our decision to terminate your services as Account Executive of Manila Times effective September 30, 1992. "This is in accordance with our contract signed last July 1, 1992."3 Issue: WON Paguio is a regular employee of the respondent (The crux of the matter would entail the determination of the nature of contractual relationship between petitioner and respondent company - was it or was it not one of regular employment?)

Page 36 of 95

Ruling: The petitioner is a regular employee. A "regular employment," whether it is one or not, is aptly gauged from the concurrence, or the non-concurrence, of the following factors - a) the manner of selection and engagement of the putative employee, b) the mode of payment of wages, c) the presence or absence of the power of dismissal; and d) the presence or absence of the power to control the conduct of the putative employee or the power to control the employee with respect to the means or methods by which his work is to be accomplished.8 The "control test" assumes primacy in the overall consideration. Under this test, an employment relation obtains where work is performed or services are rendered under the control and supervision of the party contracting for the service, not only as to the result of the work but also as to the manner and details of the performance desired.9 An indicum of regular employment, rightly taken into account by the labor arbiter, was the reservation by respondent Metromedia Times Corporation not only of the right to control the results to be achieved but likewise the manner and the means used in reaching that end. 10 Metromedia Times Corporation exercised such control by requiring petitioner, among other things, to submit a daily sales activity report and also a monthly sales report as well. Various solicitation letters would indeed show that Robina Gokongwei, company president, Alda Iglesia, the advertising manager, and Frederick Go, the advertising director, directed and monitored the sales activities of petitioner. The Labor Code, in Article 280 thereof, provides: "ART. 280. Regular and Casual Employment. The provisions of written agreement to the contrary notwithstanding and regardless of the oral agreement of the parties, an employment shall be deemed to be regular where the employee has been engaged to perform activities which are usually necessary or desirable in the usual business or trade of the employer, except where the employment has been fixed for a specific project or undertaking the completion or termination of which has been determined at the time of the engagement of the employee or where the work or services to be performed is seasonal in nature and the employment is for the duration of the season. "An employment shall be deemed to be casual if it is not covered by the proceeding paragraph: Provided, That, any employee who has rendered at least one year of service, whether such service is continuous or broken, shall be considered a regular employee with respect to the activity in which he is employed and his employment shall continue while such activity exists." Thus defined, a regular employee is one who is engaged to perform activities which are necessary and desirable in the usual business or trade of the employer as against those which are undertaken for a specific project or are seasonal. Even in these latter cases, where such person has rendered at least one year of service, regardless of the nature of the activity performed or of whether it is continuous or intermittent, the employment is considered regular as long as the activity exists, it not being indispensable that he be first issued a regular appointment or be formally declared as such before acquiring a regular status.11 That petitioner performed activities which were necessary and desirable to the business of the employer, and that the same went on for more than a year, could hardly be denied. Petitioner was an account executive in soliciting advertisements, clearly necessary and desirable, for the survival and continued operation of the business of respondent corporation. Robina Gokongwei, its President, herself admitted that the income generated from paid advertisements was the lifeblood of the newspaper's existence. Implicitly, respondent corporation recognized petitioner's invaluable contribution to the business when it renewed, not just once but five times, its contract with petitioner. Respondent company cannot seek refuge under the terms of the agreement it has entered into with petitioner. The law, in defining their contractual relationship, does so, not necessarily or exclusively upon the terms of their written or oral contract, but also on the basis of the nature of the work petitioner has been called upon to perform. 12 The law affords protection to an employee, and it will not countenance any attempt to subvert its spirit and intent. A stipulation in an agreement can be ignored as and when it is utilized to deprive the employee of his security of tenure. 13 The sheer

Page 37 of 95

inequality that characterizes employer-employee relations, where the scales generally tip against the employee, often scarcely provides him real and better options. B. seasonal HACIENDA FATIMA and/or PATRICIO VILLEGAS, ALFONSO VILLEGAS and CRISTINE SEGURA, petitioners, vs.NATIONAL FEDERATION OF SUGARCANE WORKERS- FOOD AND GENERAL TRADE, respondents. Facts: 'Now therefore, in the interest of both labor and management, parties herein agree as follows: '1. That the list of the names of affected union members hereto attached and made part of this agreement shall be referred to the Hacienda payroll of 1990 and determine whether or not this concerned Union members are hacienda workers; '2. That in addition to the payroll of 1990 as reference, herein parties will use as guide the subjects of a Memorandum of Agreement entered into by and between the parties last January 4, 1990; '3. That herein parties can use other employment references in support of their respective claims whether or not any or all of the listed 36 union members are employees or hacienda workers or not as the case may be; '4. That in case conflict or disagreement arises in the determination of the status of the particular hacienda workers subject of this agreement herein parties further agree to submit the same to voluntary arbitration; '5. To effect the above, a Committee to be chaired by Rose Mengaling is hereby created to be composed of three representatives each and is given five working days starting Jan. 23, 1992 to resolve the status of the subject 36 hacienda workers. (Union representatives: Bernardo Torres, Martin Alas-as, Ariston Arulea Jr.)" "Pursuant thereto, the parties subsequently met and the Minutes of the Conciliation Meeting showed as follows:

1. Luisa Rombo 2. Ramona Rombo 3. Bobong Abrega 4. Boboy Silva

'The meeting started at 10:00 A.M. A list of employees was submitted by Atty. Tayko based on who received their 13th month pay. The following are deemed not considered employees: 'The name Orencio Rombo shall be verified in the 1990 payroll.

'The following employees shall be reinstated immediately upon availability of work: 1. Jose Dagle 2. Rico Dagle 3. Ricardo Dagle 4. Jesus Silva 5. Fernando Silva 6. Ernesto Tejares 7. Alejandro Tejares 8. Gaudioso Rombo 9. Martin Alas-as Jr. 10. Cresensio Abrega 11. Ariston Eruela Sr. 12. Ariston Eruela Jr.'

Page 38 of 95

It is being disputed the nature of the employment done by the laborers in hacienda Fatima. Issue: Won the work of respondents was seasonal in nature Ruling: The respondents are regular employees an their work was not seasonal in nature. The CA did not err when it held that respondents were regular employees. Article 280 of the Labor Code, as amended, states: "Art. 280. Regular and Casual Employment. The provisions of written agreement to the contrary notwithstanding and regardless of the oral agreement of the parties, an employment shall be deemed to be regular where the employee has been engaged to perform activities which are usually necessary or desirable in the usual business or trade of the employer, except where the employment has been fixed for a specific project or undertaking the completion or termination of which has been determined at the time of the engagement of the employee or where the work or services to be performed is seasonal in nature and the employment is for the duration of the season. "An employment shall be deemed to be casual if it is not covered by the preceding paragraph: Provided, That, any employee who has rendered at least one year of service, whether such service is continuous or broken, shall be considered a regular employee with respect to the activity in which he is employed and his employment shall continue while such activity exist." (Italics supplied) For respondents to be excluded from those classified as regular employees, it is not enough that they perform work or services that are seasonal in nature. They must have also been employed only for the duration of one season. The evidence proves the existence of the first, but not of the second, condition. The fact that respondents with the exception of Luisa Rombo, Ramona Rombo, Bobong Abriga and Boboy Silva repeatedly worked as sugarcane workers for petitioners for several years is not denied by the latter. Evidently, petitioners employed respondents for more than one season. Therefore, the general rule of regular employment is applicable. In Abasolo v. National Labor Relations Commission,
13

the Court issued this clarification:

"[T]he test of whether or not an employee is a regular employee has been laid down in De Leon v. NLRC, in which this Court held: "The primary standard, therefore, of determining regular employment is the reasonable connection between the particular activity performed by the employee in relation to the usual trade or business of the employer. The test is whether the former is usually necessary or desirable in the usual trade or business of the employer. The connection can be determined by considering the nature of the work performed and its relation to the scheme of the particular business or trade in its entirety. Also if the employee has been performing the job for at least a year, even if the performance is not continuous and merely intermittent, the law deems repeated and continuing need for its performance as sufficient evidence of the necessity if not indispensability of that activity to the business. Hence, the employment is considered regular, but only with respect to such activity and while such activity exists.xxx xxx xxx ". . . [T]he fact that [respondents] do not work continuously for one whole year but only for the duration of the . . . season does not detract from considering them in regular employment since in a litany of cases this Court has already settled that seasonal workers who are called to work from time to time and are temporarily laid off during off-season are not separated from service in said period, but merely considered on leave until re-employed." 14

Page 39 of 95

The CA did not err when it ruled that Mercado v. NLRC 15 was not applicable to the case at bar. In the earlier case, the workers were required to perform phases of agricultural work for a definite period of time, after which their services would be available to any other farm owner. They were not hired regularly and repeatedly for the same phase/s of agricultural work, but on and off for any single phase thereof. On the other hand, herein respondents, having performed the same tasks for petitioners every season for several years, are considered the latter's regular employees for their respective tasks. Petitioners' eventual refusal to use their services even if they were ready, able and willing to perform their usual duties whenever these were available and hiring of other workers to perform the tasks originally assigned to respondents amounted to illegal dismissal of the latter. The Court finds no reason to disturb the CA's dismissal of what petitioners claim was their valid exercise of a management prerogative. The sudden changes in work assignments reeked of bad faith. These changes were implemented immediately after respondents had organized themselves into a union and started demanding collective bargaining. Those who were union members were effectively deprived of their jobs. Petitioners' move actually amounted to unjustified dismissal of respondents, in violation of the Labor Code. "Where there is no showing of clear, valid and legal cause for the termination of employment, the law considers the matter a case of illegal dismissal and the burden is on the employer to prove that the termination was for a valid and authorized cause." 16 In the case at bar, petitioners failed to prove any such cause for the dismissal of respondents who, as discussed above, are regular employees. PHILIPPINE TOBACCO FLUE- CURING & REDRYING RELATIONS COMMISSION Ligaya Lubat et al. Facts: This case involves two groups of seasonal workers who claimed separation benefits after the closure of petitioner's tobacco processing plant in Balintawak, Metro Manila and the transfer of its tobacco operations to Candon, Ilocos Sur. Petitioner refuses to grant separation pay to the workers belonging to the first batch (referred to as the Lubat group), because they had not been given work during the preceding year and, hence, were no longer in its employ at the time it closed its Balintawak plant. Likewise, it claims exemption from awarding separation pay to the second batch (the Luris group), because the closure of its plant was due to "serious business losses," as defined in Article 283 of the Labor Code. These refer to the consolidated cases for payment of separation pay lodged by [the] Lubat Group, and for illegal dismissal and underpayment of separation pay by [the] Luris group, with prayers for damages and attorney's fees against the above respondents. The record reveals that all complainants in both cases were former workers of respondent with their respective periods of employment and latest wages stated in the parties' pleadings/[a]nnexes. On August 1, 1994, due to supposed serious financial reverses and losses suffered by respondent and its desire to prevent further losses, a notice of permanent closure of its red[r]ying operations at Balintawak, Quezon City and transfer [of] the same to Candon, Ilocos Sur was served to the DOLE. On August 3, 1994, complainants were also notified of the said decision to close and transfer. On August 16, 1994, their separation benefits were given to them but allegedly [based on] wrong computation when management did not consider 3/4 of their length of service as claimed by complainants (Luris group). Issue: WON the respondents are seasonal employees CORPORATION, petitioner, vs.NATIONAL LABOR

Page 40 of 95

Ruling: Respondents are seasonal workers who are called to work from time to time ripened into regular employment. Petitioner relies upon our ruling in Mercado v. NLRC 11 that the "employment [of seasonal employees] legally ends upon completion of the . . . season," a statement which was subsequently reiterated in Magcalas v. NLRC. 12 Thus, petitioner argues that it was not obliged to rehire the members of the Lubat group for the 1994 season, because their employment had been terminated at the end of the 1993 season. Since they were not employed for the 1994 season when the Balintawak plant was closed, it follows that petitioner has no obligation to award them separation pay due to the said closure. We are not persuaded. From the facts, we are convinced that petitioner illegally dismissed the members of the Lubat group when it refused to allow them to work during the 1994 season. This Court has previously ruled in Manila Hotel Company v. CIR 13 that seasonal workers who are called to work from time to time and are temporarily laid of during off-season are not separated from service in said period, but are merely considered on leave until reemployed, viz.: The nature of their relationship . . . is such that during off season they are temporarily laid off but during summer season they are re-employed, or when their services may be needed. They are not strictly speaking separated from the service but are merely considered as on leave of absence without pay until they are re-employed. The above doctrine was echoed by this Court in Industrial-Commercial-Agricultural Workers' Organization (ICAWO) v. CIR 14 and Visayan Stevedore Transportation Company v. CIR. 15 Petitioner claims that the aforecited ruling has been superseded by Article 280 of the Labor Code, which took effect on November 1, 1974. We disagree. There is no clear conflict between the above doctrine and Article 280 of the Labor Code. In fact, the same doctrine was reiterated by this Court in Tacloban Sagkahan Rice and Corn Mills Co. v. NLRC 16 in 1990, which was promulgated after the labor Code took effect. Furthermore, in Bacolod-Murcia Milling Co, Inc. v. NLRC, 17 this Court considered a seasonal workers "in regular employment" in cases involving the determination of an employeremployee relationship and security of tenure. The Court ruled: While under prevailing jurisprudence, Canete may be considered as in regular employment even during those years when she was merely a seasonal worker, that legal conclusion will hold true only in cases involving the determination of an employer-employee relationship or security of tenure. Again in Gaco v. NLRC, petitioner therein was a seasonal worker employed and repeatedly rehired in a business enterprise similar to that of petitioner herein. Finding that he was in regular employment and thus entitled to separation pay for having been constructively dismissed, the Court stated: I may appear that the work in private respondent Orient Leaf Tobacco Corporation is seasonal, however, the records reveal that petitioner Zenaida Gaco was repeatedly re-hired, sufficiently evidencing the necessity and indispensability of her services to the former's business or trade. Furthermore, she has been employed since 1974 up to the end of the season in 1989. Owing to her length of service, she became a regular employee, by operation of law, one year after she was employed. 18 From the foregoing, it follows that the employer-employee relationship between herein petitioner and members of the Lubat group was not terminated at the end of the 1993 season. From the end of the 1993 season until the beginning of the 1994 season, they were considered only on leave but nevertheless still in the employ of petitioner. The facts in the above-mentioned cases are different from those in Mercado v. NLRC 19 and in Magcalas v. NLRC . 20 In Mercado, although respondent constantly availed herself of petitioners' services from year to year, it was clear from the facts therein that they were not in her regular employ. Petitioners therein performed different phases of agricultural work

Page 41 of 95

in a given year. However, during that period that period, they were free to work for other farm owners, and in fact they did. In other words, they worked for respondent, but were nevertheless free to contract their services with other farm owners. The Court was thus emphatic when it ruled that petitioners were mere project employees, who could be hired by other farm owners. As such, their employment would naturally end upon the completion of each project or each phase of farm work which has been contracted. In Magcalas v. NLRC, the Court merely cited the aforequoted ruling to explain the difference among regular, project and seasonal employees. In fact, it concluded that the employees therein were regular and not project employees. From the peculiar facts of Mercado and Magcalas, it is clear that the ruling therein is not inconsistent with Manila Hotel, Gaco and other cases. It is noteworthy that the ponente in Mercado concurred in the Court's ruling in Gaco awarding to the seasonal employee separation pay for every year of service. Prescinding from the above, we hold that petitioner is liable for illegal dismissal and should be responsible for the reinstatement of the Lubat group and the payment of their back wages. However, since reinstatement is no longer possible as petitioner has already closed its Balintawak plant, respondent members of the said group should instead be awarded normal separation pay (in lieu of reinstatement) equivalent to at least one month pay, or one month pay for every year of service, whichever is higher. It must be stressed that the separation pay being awarded to the Lubat group is due to illegal dismissal; hence, it is different from the amount of separation pay provided for in Article 283 in case of retrenchment to prevent losses or in case of closure or cessation of the employer's business, in either of which the separation pay is equivalent to at least one (1) month or one-half (1/2) month pay for every year of service, whichever is higher. PHILIPPINE FRUIT & VEGETABLE INDUSTRIES, INC. and its President and General Manager, MR. PEDRO CASTILLO, petitioners,vs. NATIONAL LABOR RELATIONS COMMISSION, and Philippine Fruit and Vegetable Workers Union-Tupas Local Chapter, respondents. Facts: Petitioner Philippine Fruit and Vegetable Industries, Inc. (PFVII, for brevity) is a government-owned and controlled corporation engaged in the manufacture and processing of fruit and vegetable purees for export. Petitioner Pedro Castillo is the former President and General Manager of petitioner PFVII. On September 5, 1988 herein private respondent Philippine Fruit and Vegetable Workers Union-Tupas Local Chapter, for and in behalf of 127 of its members, filed a complaint for unfair labor practice and/or illegal dismissal with damages against petitioner corporation. Private respondent alleged that many of its complaining members started working for San Carlos Fruits Corporation which later incorporated into PFVII in January or February 1983 until their dismissal on different dates in 1985, 1986, 1987 and 1988. They further alleged that the dismissals were due to complainants' involvement in union activities and were without just cause. Issue: WON the respondents are seasonal employees Ruling: They are NOT seasonal employees but regular employees. Art. 280 of the Labor Code provides:

Regular and Casual Employment. The provisions of written agreement to the contrary notwithstanding and regardless of the oral agreement of the parties, an employment shall be deemed to be regular where the employee has been engaged to perform activities which are usually necessary or desirable in the usual business or trade of the employers, except where the employment has been fixed for a specific project. . . .

Page 42 of 95

An employment shall be deemed to be casual if it is not covered by the preceeding paragraph; provided, that, any employee who has rendered at least one year of service whether such service is continuous or broken, shall be considered a regular employee with respect to the activity in which he is employed and his employment shall continue while such actually exists. Under the above provision, an employment shall be deemed regular where the employee: a) has been engaged to perform activities which are usually necessary or desirable in the usual business or trade of the employer; or b) has rendered at least one year of service, whether such service is continuous or broken, with respect to the activity in which he is employed. 12 In the case at bar, the work of complainants as seeders, operators, sorters, slicers, janitors, drivers, truck helpers, mechanics and office personnel is without doubt necessary in the usual business of a food processing company like petitioner PFVII. It should be noted that complainants' employment has not been fixed for a specific project or undertaking the completion or termination of which has been determined at the time of their appointment or hiring. 13 Neither is their employment seasonal in nature. While it may be true that some phases of petitioner company's processing operations is dependent on the supply of fruits for a particular season, the other equally important aspects of its business, such as manufacturing and marketing are not seasonal. The fact is that large-scale food processing companies such as petitioner company continue to operate and do business throughout the year even if the availability of fruits and vegetables is seasonal. Having determined that private respondents are regular employees under the first paragraph, we need not dwell on the question of whether or not they had rendered one year of service. This Court has clearly stated in Mercado, Sr. vs. NLRC, 14 that: The second paragraph of Article 280 demarcates as "casual" employees, all other employees who do not fall under the definition of the preceding paragraph. The proviso, in said second paragraph, deems as regular employees those "casual" employees who have rendered at least one year of service regardless of the fact that such service may be continuous or broken.1wphi1.nt . . . Hence, the proviso is applicable only to the employees who are deemed "casuals" but not to the "project" employees nor the regular employees treated in paragraph one of Art. 280. As correctly noted by the Office of the Solicitor General; private respondents in this case are deemed regular employees by virtue of the fact that they performed functions which are necessary and desirable in the usual business of PFVII as provided under the first paragraph of Art. 280 of the Labor Code. C. Project Employees WILFREDO ARO, RONILO TIROL, JOSE PACALDO, PRIMITIVO CASQUEJO and MARCIAL ABGO , Petitioners, versus NATIONAL LABOR RELATIONS COMMISSION, FOURTH DIVISION and BENTHEL DEVELOPMENT CORPORATION ,Respondents. Facts: Several employees of private respondent Benthel Development Corporation, including the petitioners, filed a Complaint for illegal dismissal with various money claims and prayer for damages against the latter, in the NLRC Arbitration Branch No. VII in Cebu City and docketed as RAB Case No. 07-09-1222-97/12-1609-97. Thereafter, Labor Arbiter Ernesto F. Carreon rendered a decision finding private respondent guilty of illegal dismissal and ordering it to pay its thirty-six (36) employees P446,940.00 as separation pay. Issue:

Page 43 of 95

Won the petitioners are project employees Ruling: The petitioners are project employees. Court agrees with the findings of the CA that petitioners were project employees. According to the CA, petitioners are project employees as found by Labor Arbiter Ernesto Carreon in his Decision dated May 28, 1998, because they were hired for the construction of the Cordova Reef Village Resort in Cordova, Cebu, which was later on affirmed by the NLRC in its January 12, 1999 decision. The only discrepancy is the Order of the NLRC that petitioners are entitled to backwages up to the finality of its decision, when as project employees, private respondents are only entitled to payment of backwages until the date of the completion of the project. In a later resolution on private respondent's motion for reconsideration of its January 12, 1999 decision, the NLRC changed its findings by ruling that petitioners herein were regular employees and, therefore, entitled to full backwages, until finality of the decision, citing that petitioners repeated rehiring over a long span of time made them regular employees. Article 280 of the Labor Code distinguishes a "project employee" from a "regular employee," thus: Article 280. Regular and Casual Employment The provisions of written agreement to the contrary notwithstanding and regardless of the oral agreement of the parties, an employment shall be deemed to be regular where the employee has been engaged to perform activities which are usually necessary or desirable in the usual business or trade of the employer, except where the employment has been fixed for a specific project or undertaking the completion or termination of which has been determined at the time of the engagement of the employee or where the work or service to be performed is seasonal in nature and the employment is for the duration of the season. An employment shall be deemed to be casual if it is not covered by the preceding paragraph: Provided, That, any employee who has rendered at least one year service, whether such service is continuous or broken, shall be considered a regular employee with respect to the activity in which he is employed and his employment shall continue while such activity exists. In Hanjin Heavy Industries and Construction Co. Ltd. v. Ibaez, 87[10] this Court extensively discussed the above distinction, thus: x x x [T]he principal test for determining whether particular employees are properly characterized as "project employees" as distinguished from "regular employees" is whether or not the project employees were assigned to carry out a "specific project or undertaking," the duration and scope of which were specified at the time the employees were engaged for that project.88[11] In a number of cases,89[12] the Court has held that the length of service or the re-hiring of construction workers on a project-to-project basis does not confer upon them regular employment status, since their re-hiring is only a natural consequence of the fact that experienced construction workers are preferred. Employees who are hired for carrying out a separate job, distinct from the other undertakings of the company, the scope and duration of which has been determined and made known to the employees at the time of the employment , are properly treated as project employees and their services may be lawfully terminated upon the completion of a project.90[13] Should the terms of their employment fail to comply with this standard, they cannot be considered project employees.

87 88 89 90

Page 44 of 95

In Abesco Construction and Development Corporation v. Ramirez,91[14] which also involved a construction company and its workers, this Court considered it crucial that the employees were informed of their status as project employees: The principal test for determining whether employees are "project employees" or "regular employees" is whether they are assigned to carry out a specific project or undertaking, the duration and scope of which are specified at the time they are engaged for that project. Such duration, as well as the particular work/service to be performed, is defined in an employment agreement and is made clear to the employees at the time of hiring. In this case, petitioners did not have that kind of agreement with respondents. Neither did they inform respondents of the nature of the latters work at the time of hiring. Hence, for failure of petitioners to substantiate their claim that respondents were project employees, we are constrained to declare them as regular employees. In Caramol v. National Labor Relations Commission,92[15] and later reiterated in Salinas, Jr. v. National Labor Relations Commission,93[16] the Court markedly stressed the importance of the employees' knowing consent to being engaged as project employees when it clarified that "there is no question that stipulation on employment contract providing for a fixed period of employment such as project-to-project contract is valid provided the period was agreed upon knowingly and voluntarily by the parties, without any force, duress or improper pressure being brought to bear upon the employee and absent any other circumstances vitiating his consent x x x." It is not disputed that petitioners were hired for the construction of the Cordova Reef Village Resort in Cordova, Cebu. By the nature of the contract alone, it is clear that petitioners' employment was to carry out a specific project. Hence, the CA did not commit grave abuse of discretion when it affirmed the findings of the Labor Arbiter. The CA correctly ruled: A review of the facts and the evidence in this case readily shows that a finding had been made by Labor Arbiter Ernesto Carreon, in his decision dated May 28, 1998, that complainants, including private respondents, are project employees. They were hired for the construction of the Cordova Reef Village Resort in Cordova, Cebu. We note that no appeal had been made by the complainants, including herein private respondents, from the said finding. Thus, that private respondents are project employees has already been effectively established. Likewise, a review of the public respondent's January 12, 1999 decision shows that it affirmed the labor arbiter's finding of the private respondents' being project employees. We therefore cannot fathom how the public respondent could have ordered backwages up to the finality of its decision when, as project employees, private respondents are only entitled to payment of the same until the date of the completion of the project. It is settled that, without a valid cause, the employment of project employees cannot be terminated prior to expiration. Otherwise, they shall be entitled to reinstatement with full backwages. However, if the project or work is completed during the pendency of the ensuing suit for illegal dismissal, the employees shall be entitled only to full backwages from the date of the termination of their employment until the actual completion of the work. While it may be true that in the proceedings below the date of completion of the project for which the private respondents were hired had not been clearly established, it constitutes grave abuse of discretion on the part of the public respondent for not determining for itself the date of said completion instead of merely ordering payment of backwages until finality of its decision.x x x x

91 92 93

Page 45 of 95

The decision of the labor arbiter, as affirmed by the public respondent in its January 12, 1999 decision, clearly established that private respondents were project employees. Because there was no showing then that the project for which their services were engaged had already been completed, the public respondent likewise found that private respondents were illegally dismissed and thus entitled to backwages. However, in utter disregard of the law and prevailing jurisprudence, the public respondents capriciously and arbitrarily ordered that the said backwages be computed until the finality of its decision instead of only until the date of the project completion. In grave abuse of its discretion, the public respondent refused to consider the evidence presented before it as to the date of completion of the Cordova Reef Village Resort project. The records show that affidavits have been executed by the petitioner's manager, corporate architect and project engineer as to the fact of the completion of the project in October 1996. As these evidences [sic] were already a matter of record, the public respondent should not have closed its eyes and should have endeavored to render a correct and just judgment.x x x x Furthermore, as earlier noted, private respondents did not appeal from the Labor Arbiter's findings that they were indubitably project employees. However, they were entitled to the payment of separation pay only for the reason that the date of the completion of the project for which they were hired had not been clearly established. Thus, in affirming the labor arbiter's decision, the public respondent in effect sustained the finding that private respondents are project employees. The statement, therefore, contained in the resolution of the petitioner's motion for reconsideration of its January 12, 1999 decision that repeated rehiring makes the worker a regular employee, is at best an obiter, especially considering that such conclusion had not been shown to apply to the circumstances then obtaining with the private respondents' employment with the petitioner.94[17] Therefore, being project employees, petitioners are only entitled to full backwages, computed from the date of the termination of their employment until the actual completion of the work. Illegally dismissed workers are entitled to the payment of their salaries corresponding to the unexpired portion of their employment where the employment is for a definite period.95[18] In this case, as found by the CA, the Cordova Reef Village Resort project had been completed in October 1996 and private respondent herein had signified its willingness, by way of concession to petitioners, to set the date of completion of the project as March 18, 1997; hence, the latter date should be considered as the date of completion of the project for purposes of computing the full backwages of petitioners. D.M. CONSUNJI, INC. and/or DAVID M. CONSUNJI, Petitioners,- versus -ESTELITO L. JAMIN Respondent. Facts: On December 17, 1968, petitioner D.M. Consunji, Inc. (DMCI), a construction company, hired respondent Estelito L. Jamin as a laborer. Sometime in 1975, Jamin became a helper carpenter. Since his initial hiring, Jamins employment contract had been renewed a number of times.96[4] On March 20, 1999, his work at DMCI was terminated due to the completion of the SM Manila project. This termination marked the end of his employment with DMCI as he was not rehired again. On April 5, 1999, Jamin filed a complaint 97[5] for illegal dismissal, with several money claims (including attorneys fees), against DMCI and its President/General Manager, David M. Consunji. Jamin alleged that DMCI terminated his employment without a just and authorized cause at a time when he was already 55 years old and had no independent source of livelihood. He claimed that he rendered service to DMCI continuously for almost 31 years. In addition to the schedule of projects (where he was assigned) submitted by DMCI to the labor arbiter,98[6] he alleged that he worked for three other DMCI projects: Twin Towers, Ritz Towers, from July 29, 1980 to June 12, 1982; New Istana Project, B.S.B.

94 95 96 97

Page 46 of 95

Brunei, from June 23, 1982 to February 16, 1984; and New Istana Project, B.S.B. Brunei, from January 24, 1986 to May 25, 1986. DMCI denied liability. It argued that it hired Jamin on a project-to-project basis, from the start of his engagement in 1968 until the completion of its SM Manila project on March 20, 1999 where Jamin last worked. With the completion of the project, it terminated Jamins employment. It alleged that it submitted a report to the Department of Labor and Employment (DOLE) everytime it terminated Jamins services. Issue: WON Jamin is a project employee Ruling: Due to repeated rehiring, Jamin as project employee from the very start deeme to be regular employee. We agree with the CA . In Liganza v. RBL Shipyard Corporation,99[37] the Court held that [a]ssuming, without granting[,] that [the] petitioner was initially hired for specific projects or undertakings, the repeated re-hiring and continuing need for his services for over eight (8) years have undeniably made him a regular employee . We find the Liganza ruling squarely applicable to this case, considering that for almost 31 years, DMCI had repeatedly, continuously and successively engaged Jamins services since he was hired on December 17, 1968 or for a total of 38 times 35 as shown by the schedule of projects submitted by DMCI to the labor arbiter 100[38] and three more projects or engagements added by Jamin, which he claimed DMCI intentionally did not include in its schedule so as to make it appear that there were wide gaps in his engagements. One of the three projects was local, the Ritz Towers,101[39] from July 29, 1980 to June 12, 1982, while the other two were overseas the New Istana Project in Brunei, Darussalam, from June 23, 1982 to February 16, 1984;102[40] and again, the New Istana Project, from January 24, 1986 to May 25, 1986.103[41] We reviewed Jamins employment contracts as the CA did and we noted that while the contracts indeed show that Jamin had been engaged as a project employee, there was an almost unbroken string of Jamins rehiring from December 17, 1968 up to the termination of his employment on March 20, 1999. While the history of Jamins employment (schedule of projects)104[42] relied upon by DMCI shows a gap of almost four years in his employment for the period between July 28, 1980 (the supposed completion date of the Midtown Plaza project) and June 13, 1984 (the start of the IRRI Dorm IV project), the gap was caused by the companys omission of the three projects above mentioned. For not disclosing that there had been other projects where DMCI engaged his services, Jamin accuses the company of suppressing vital evidence that supports his contention that he rendered service in the companys construction projects continuously and repeatedly for more than three decades. The non-disclosure might not have constituted suppression of evidence it could just have been overlooked by the company but the oversight is unfair to Jamin as the noninclusion of the three projects gives the impression that there were substantial gaps not only of several months but years in his employment with DMCI.

98 99 100 101 102 103 104

Page 47 of 95

Thus, as Jamin explains, the Ritz Tower Project (July 29, 1980 to June 12, 1982) and the New Istana Project (June 23, 1982 to February 16, 1984) would explain the gap between the Midtown Plaza project (September 3, 1979 to July 28, 1980) and the IRRI Dorm IV project (June 13, 1984 to March 12, 1985) and the other New Istana Project (January 24, 1986 to May 25, 1986) would explain the gap between P. 516 Hanger (September 13, 1985 to January 23, 1986) and P. 516 Maint (May 26, 1986 to November 18, 1987). To reiterate, Jamins employment history with DMCI stands out for his continuous, repeated and successive rehiring in the companys construction projects. In all the 38 projects where DMCI engaged Jamins services, the tasks he performed as a carpenter were indisputably necessary and desirable in DMCIs construction business. He might not have been a member of a work pool as DMCI insisted that it does not maintain a work pool, but his continuous rehiring and the nature of his work unmistakably made him a regular employee. In Maraguinot, Jr. v. NLRC,105[43] the Court held that once a project or work pool employee has been: (1) continuously, as opposed to intermittently, rehired by the same employer for the same tasks or nature of tasks; and (2) these tasks are vital, necessary and indispensable to the usual business or trade of the employer, then the employee must be deemed a regular employee . Further, as we stressed in Liganza,106[44] [r]espondent capitalizes on our ruling in D.M. Consunji, Inc. v. NLRC which reiterates the rule that the length of service of a project employee is not the controlling test of employment tenure but whether or not the employment has been fixed for a specific project or undertaking the completion or termination of which has been determined at the time of the engagement of the employee. Surely, length of time is not the controlling test for project employment. Nevertheless, it is vital in determining if the employee was hired for a specific undertaking or tasked to perform functions vital, necessary and indispensable to the usual business or trade of the employer. Here, [private] respondent had been a project employee several times over. His employment ceased to be coterminous with specific projects when he was repeatedly re-hired due to the demands of petitioners business.107[45] Without doubt, Jamins case fits squarely into the employment situation just quoted. With our ruling that Jamin had been a regular employee, the issue of whether DMCI submitted termination of employment reports, pursuant to Policy Instructions No. 20 (Undated 108[46]), as superseded by DOLE Department Order No. 19 (series of 1993), has become academic. DOLE Policy Instructions No. 20 provides in part: Project employees are not entitled to termination pay if they are terminated as a result of the completion of the project or any phase thereof in which they are employed, regardless of the number of projects in which they have been employed by a particular construction company. Moreover, the company is not required to obtain a clearance from the Secretary of Labor in connection with such termination. What is required of the company is a report to the nearest Public Employment Office for statistical purposes.109[47] To set the records straight, DMCI indeed submitted reports to the DOLE but as pointed out by Jamin, the submissions started only in 1992.110[48] DMCI explained that it submitted the earlier reports (1982), but it lost and never recovered the reports. It reconstituted the lost reports and submitted them to the DOLE in October 1992; thus, the dates appearing in the reports.111[49] LEYTE GEOTHERMAL POWER PROGRESSIVE EMPLOYEES UNION - ALU - TUCP, Petitioner, vs.PHILIPPINE NATIONAL OIL COMPANY - ENERGY DEVELOPMENT CORPORATION, Respondent. Facts:

105 106 107 108 109 110 111

Page 48 of 95

[Respondent Philippine National Oil Corporation]-Energy Development Corporation [PNOC-EDC] is a government-owned and controlled corporation engaged in exploration, development, utilization, generation and distribution of energy resources like geothermal energy.

Petitioner is a legitimate labor organization, duly registered with the Department of Labor and Employment (DOLE) Regional Office No. VIII, Tacloban City.

Among [respondents] geothermal projects is the Leyte Geothermal Power Project located at the Greater Tongonan Geothermal Reservation in Leyte. The said Project is composed of the Tongonan 1 Geothermal Project (T1GP) and the Leyte Geothermal Production Field Project (LGPF) which provide the power and electricity needed not only in the provinces and cities of Central and Eastern Visayas (Region VII and VIII), but also in the island of Luzon as well. Thus, the [respondent] hired and employed hundreds of employees on a contractual basis, whereby, their employment was only good up to the completion or termination of the project and would automatically expire upon the completion of such project.

On December 28, 1998, the petitioner filed a Notice of Strike with DOLE against the [respondent] on the ground of purported commission by the latter of unfair labor practice for "refusal to bargain collectively, union busting and mass termination." On the same day, the petitioner declared a strike and staged such strike. To avert any work stoppage, then Secretary of Labor Bienvenido E. Laguesma intervened and issued the Order, dated January 4, 1999, certifying the labor dispute to the NLRC for compulsory arbitration. Accordingly, all the striking workers were directed to return to work within twelve (12) hours from receipt of the Order and for the [respondent] to accept them back under the same terms and conditions of employment prior to the strike. Further, the parties were directed to cease and desist from committing any act that would exacerbate the situation. However, despite earnest efforts on the part of the Secretary of Labor and Employment to settle the dispute amicably, the petitioner remained adamant and unreasonable in its position, causing the failure of the negotiation towards a peaceful compromise. In effect, the petitioner did not abide by [the] assumption order issued by the Secretary of Labor. Consequently, on January 15, 1999, the [respondent] filed a Complaint for Strike Illegality, Declaration of Loss of Employment and Damages at the NLRC-RAB VIII in Tacloban City and at the same time, filed a Petition for Cancellation of Petitioners Certificate of Registration with DOLE, Regional Office No. VIII. The two cases were later on consolidated pursuant to the New NLRC Rules of Procedure. The consolidated case was docketed as NLRC Certified Case No. V-02-99 (NCMB-RAB VIII-NS-12-0190-98; RAB Case No. VIII-1-0019-99). The said certified case was indorsed to the NLRC 4th Division in Cebu City on June 21, 1999 for the proper disposition thereof.3 Issue: Won the petitioners are project employees Ruling: They are project employees SC held. However, notwithstanding the foregoing iterations, project employment contracts which fix the employment for a specific project or undertaking remain valid under the law:

Page 49 of 95

x x x By entering into such a contract, an employee is deemed to understand that his employment is coterminous with the project. He may not expect to be employed continuously beyond the completion of the project. It is of judicial notice that project employees engaged for manual services or those for special skills like those of carpenters or masons, are, as a rule, unschooled. However, this fact alone is not a valid reason for bestowing special treatment on them or for invalidating a contract of employment. Project employment contracts are not lopsided agreements in favor of only one party thereto. The employers interest is equally important as that of the employee[s] for theirs is the interest that propels economic activity. While it may be true that it is the employer who drafts project employment contracts with its business interest as overriding consideration, such contracts do not, of necessity, prejudice the employee. Neither is the employee left helpless by a prejudicial employment contract. After all, under the law, the interest of the worker is paramount.12 In the case at bar, the records reveal that the officers and the members of petitioner Union signed employment contracts indicating the specific project or phase of work for which they were hired, with a fixed period of employment. The NLRC correctly disposed of this issue: A deeper examination also shows that [the individual members of petitioner Union] indeed signed and accepted the [employment contracts] freely and voluntarily. No evidence was presented by [petitioner] Union to prove improper pressure or undue influence when they entered, perfected and consummated [the employment] contracts. In fact, it was clearly established in the course of the trial of this case, as explained by no less than the President of [petitioner] Union, that the contracts of employment were read, comprehended, and voluntarily accepted by them. x x x. xxxx As clearly shown by [petitioner] Unions own admission, both parties had executed the contracts freely and voluntarily without force, duress or acts tending to vitiate the worker[s] consent. Thus, we see no reason not to honor and give effect to the terms and conditions stipulated therein. x x x.13 Thus, we are hard pressed to find cause to disturb the findings of the NLRC which are supported by substantial evidence. It is well-settled in jurisprudence that factual findings of administrative or quasi-judicial bodies, which are deemed to have acquired expertise in matters within their respective jurisdictions, are generally accorded not only respect but even finality, and bind the Court when supported by substantial evidence.14 Rule 133, Section 5 defines substantial evidence as "that amount of relevant evidence which a reasonable mind might accept as adequate to justify a conclusion." Consistent therewith is the doctrine that this Court is not a trier of facts, and this is strictly adhered to in labor cases. 15 We may take cognizance of and resolve factual issues, only when the findings of fact and conclusions of law of the Labor Arbiter or the NLRC are inconsistent with those of the CA.16 In the case at bar, both the NLRC and the CA were one in the conclusion that the officers and the members of petitioner Union were project employees. Nonetheless, petitioner Union insists that they were regular employees since they performed work which was usually necessary or desirable to the usual business or trade of the Construction Department of respondent. The landmark case of ALU-TUCP v. NLRC17 instructs on the two (2) categories of project employees: It is evidently important to become clear about the meaning and scope of the term "project" in the present context. The "project" for the carrying out of which "project employees" are hired would ordinarily have some relationship to the usual business of the employer. Exceptionally, the "project" undertaking might not have an ordinary or normal relationship to the usual business of the employer. In this latter case, the determination of the scope and parameters of the "project" becomes fairly easy. x x x. From the viewpoint, however, of the legal characterization problem here presented to the Court, there should be no difficulty in designating the employees who are retained or hired for the purpose of undertaking fish culture or the production of vegetables as "project employees," as distinguished from ordinary or "regular employees," so long as the duration and scope of the project were determined or specified at the time of engagement of

Page 50 of 95

the "project employees." For, as is evident from the provisions of Article 280 of the Labor Code, quoted earlier, the principal test for determining whether particular employees are properly characterized as "project employees" as distinguished from "regular employees," is whether or not the "project employees" were assigned to carry out a "specific project or undertaking," the duration (and scope) of which were specified at the time the employees were engaged for that project. In the realm of business and industry, we note that "project" could refer to one or the other of at least two (2) distinguishable types of activities. Firstly, a project could refer to a particular job or undertaking that is within the regular or usual business of the employer company, but which is distinct and separate, and identifiable as such, from the other undertakings of the company. Such job or undertaking begins and ends at determined or determinable times. The typical example of this first type of project is a particular construction job or project of a construction company. A construction company ordinarily carries out two or more [distinct] identifiable construction projects: e.g., a twenty-five-storey hotel in Makati; a residential condominium building in Baguio City; and a domestic air terminal in Iloilo City. Employees who are hired for the carrying out of one of these separate projects, the scope and duration of which has been determined and made known to the employees at the time of employment, are properly treated as "project employees," and their services may be lawfully terminated at completion of the project. The term "project" could also refer to, secondly, a particular job or undertaking that is not within the regular business of the corporation. Such a job or undertaking must also be identifiably separate and distinct from the ordinary or regular business operations of the employer. The job or undertaking also begins and ends at determined or determinable times.18 Plainly, the litmus test to determine whether an individual is a project employee lies in setting a fixed period of employment involving a specific undertaking which completion or termination has been determined at the time of the particular employees engagement. In this case, as previously adverted to, the officers and the members of petitioner Union were specifically hired as project employees for respondents Leyte Geothermal Power Project located at the Greater Tongonan Geothermal Reservation in Leyte. Consequently, upon the completion of the project or substantial phase thereof, the officers and the members of petitioner Union could be validly terminated. MILLENNIUM ERECTORS CORPORATION, Petitioner, - versus -VIRGILIO MAGALLANES, Respondent. Facts: In its Position Paper,112[2] petitioner claimed that respondent was a project employee whom it hired for a building project in Libis on January 30, 2003, to prove which it submitted the employment contract113[3] signed by him; that on August 3, 2004, respondents services were terminated as the project was nearing completion; and he was given financial assistance114[4] in the amount of P2,000, for which he signed a quitclaim and waiver.115[5] Petitioner likewise submitted a termination report to the Department of Labor and Employment (DOLE) dated August 17, 2004. Rebutting respondents claim that he was employed since 1988, petitioner contended that it was incorporated only in February 2000, and Kenneth Construction Corporation which was established in 1989 and dissolved in 2000, was a separate and distinct entity. Issue:

112 113 114 115

Page 51 of 95

Won the respondents are project employees Ruling: On the merits of the case, the Court finds that, indeed, respondent was a regular, not a project employee.

Saberola v. Suarez116[14] reiterates the well-settled definition of project employee, viz:


A project employee is one whose "employment has been fixed for a specific project or undertaking, the completion or termination of which has been determined at the time of the engagement of the employee or where the work or service to be performed is seasonal in nature and the employment is for the duration of the season." (emphasis and underscoring supplied) And Equipment Technical Services v. Court of Appeals117[15] emphasizes the difference between a regular employee and a project employee: As the Court has consistently held, the service of project employees are coterminus [ sic ] with the project and may be terminated upon the end or completion of that project or project phase for which they were hired. Regular employees , in contrast, enjoy security of tenure and are entitled to hold on to their work or position until their services are terminated by any of the modes recognized under the Labor Code. (emphasis and underscoring supplied) Petitioners various payrolls dating as early as 2001 show that respondent had been employed by it. As aptly observed by the appellate court, these documents, rather than sustaining petitioners argument, only serve to support respondents contention that he had been employed in various projects, if not for 16 years, at the very least two years prior to his dismissal. Assuming arguendo that petitioner hired respondent initially on a per project basis, his continued rehiring, as shown by the sample payrolls converted his status to that of a regular employee. Following Cocomangas Beach Hotel Resort v. Visca,118[16] the repeated and continuing need for respondents services is sufficient evidence of the necessity, if not indispensability, of his services to petitioner's business and, as a regular employee, he could only be dismissed from employment for a just or authorized cause. Petitioner having failed to discharge its burden of proving that it terminated the services of respondent for cause and with due process, the challenged decision must remain. Sorreda vs Cambridge electronic corp Facts: On May 8, 1999, petitioner was hired by respondent as a technician for a period of 5 months at minimum wage. 119[6] Five weeks into the job (on June 15, 1999), petitioner met an accident in which his left arm was crushed by a machine and had to be amputated.120[7]

116 117 118 119

Page 52 of 95

Petitioner claimed that, shortly after his release from the hospital, officers of respondent company called him to a meeting with his common-law wife, father and cousin. There he was assured a place in the company as a regular employee for as long as the company existed and as soon as he fully recovered from his injury. In September 1999, after he recovered from his injury, petitioner reported for work. Instead of giving him employment, they made him sign a memorandum of resignation to formalize his separation from the company in the light of the expiration of his five-month contract. Issue: Won Sorreda is a project employee Ruling: He is a project employee. SC affirmed the NLRC an CA findings. The NLRC agreed with respondent.121[14] It found that petitioner was not a regular employee; thus, he was neither illegally dismissed nor entitled to reinstatement and backwages. Petitioner sued for compensatory damages because of the accident that befell him. As the contract for per-project employment had already expired, the issue no longer fell under the jurisdiction of the labor arbiter and NLRC. Moreover, the testimonies of petitioners witnesses were declared self-serving and thus insufficient to prove the contract of perpetual employment. William construction an corp vs Trinidad Facts: On August 1, 2006 respondent Jorge R. Trinidad filed a complaint for illegal dismissal and unpaid benefits against petitioner William Uy Construction Corporation. Trinidad claimed that he had been working with the latter company for 16 years since 1988 as driver of its service vehicle, dump truck, and transit mixer. He had signed several employment contracts with the company that identified him as a project employee although he had always been assigned to work on one project after another with some intervals. Respondent Trinidad further alleged that in December 2004 petitioner company terminated him from work after it shut down operations because of lack of projects. He learned later, however, that although it opened up a project in Batangas, it did not hire him back for that project. Petitioner company countered122[1] that it was in the construction business. By the nature of such business, it had to hire and engage the services of project construction workers, including respondent Trinidad, whose employments had to be co-terminous with the completion of specific company projects. For this reason, every time the company employed Trinidad, he had to execute an employment contract with it, called Appointment as Project Worker. Issue: WON the respondent is a project employee

120 121 122

Page 53 of 95

Ruling: Trinidad is a project employee. The test for distinguishing a project employee from a regular employee is whether or not he has been assigned to carry out a specific project or undertaking, with the duration and scope of his engagement specified at the time his service is contracted.123[5] Here, it is not disputed that petitioner company contracted respondent Trinidads service by specific projects with the duration of his work clearly set out in his employment contracts.124[6] He remained a project employee regardless of the number of years and the various projects he worked for the company.125[7] Generally, length of service provides a fair yardstick for determining when an employee initially hired on a temporary basis becomes a permanent one, entitled to the security and benefits of regularization. But this standard will not be fair, if applied to the construction industry, simply because construction firms cannot guarantee work and funding for its payrolls beyond the life of each project. And getting projects is not a matter of course. Construction companies have no control over the decisions and resources of project proponents or owners. There is no construction company that does not wish it has such control but the reality, understood by construction workers, is that work depended on decisions and developments over which construction companies have no say. For this reason, the Court held in Caseres v. Universal Robina Sugar Milling Corporation126[8] that the repeated and successive rehiring of project employees do not qualify them as regular employees, as length of service is not the controlling determinant of the employment tenure of a project employee, but whether the employment has been fixed for a specific project or undertaking, its completion has been determined at the time of the engagement of the employee. In this case, respondent Trinidads series of employments with petitioner company were co-terminous with its projects. When its Boni Serrano-Katipunan Interchange Project was finished in December 2004, Trinidads employment ended with it. He was not dismissed. His employment contract simply ended with the project for which he had signed up. His employment history belies the claim that he continuously worked for the company. Intervals or gaps separated one contract from another.127[9] LA ruled that Trinidad is a project employee.

The CA noted that DOLE Order 19 required employers to submit a report of termination of employees every completion of construction project. And, since petitioner company submitted at the hearing before the Labor Arbiter only the termination report covering respondent Trinidads last project, it failed to satisfy such requirement.

But respondent Trinidad did not say in his complaint that he had been illegally dismissed after each of the projects for which he had been signed up. His complaint was essentially that he should have been rehired from the last project since he had already acquired the status of a regular employee. Consequently, petitioner company needed only to show the last status of Trinidads employment, namely, that of a project employee under a contract that had ended

123 124 125 126 127

Page 54 of 95

and the companys compliance with the reporting requirement for the termination of that employment. Indeed, both the Labor Arbiter and the NLRC were satisfied that the fact of petitioner companys compliance with DOLE Order 19 had been proved in this case. D.M. CONSUNJI, INC., Petitioner, - versus -ANTONIO GOBRES, MAGELLAN DALISAY, GODOFREDO PARAGSA, EMILIO ALETA and GENEROSO MELO,Respondents. Facts: Respondents Antonio Gobres, Magellan Dalisay, Godofredo Paragsa, Emilio Aleta and Generoso Melo worked as carpenters in the construction projects of petitioner D.M. Consunji, Inc., a construction company, on several occasions and/or at various times. Their termination from employment for each project was reported to the Department of Labor and Employment (DOLE), in accordance with Policy Instruction No. 20, which was later superseded by Department Order No. 19, series of 1993. Respondents last assignment was at Quad 4-Project in Glorietta, Ayala, Makati, where they started working on September 1, 1998. On October 14, 1998, respondents saw their names included in the Notice of Termination posted on the bulletin board at the project premises. Respondents filed a Complaint with the Arbitration Branch of the National Labor Relations Commission (NLRC) against petitioner D.M. Consunji, Inc. and David M. Consunji for illegal dismissal, and non-payment of 13 th month pay, five (5) days service incentive leave pay, damages and attorneys fees. Petitioner D.M. Consunji, Inc. and David M. Consunji countered that respondents, being project employees, are covered by Policy Instruction No. 20, as superseded by Department Order No. 19, series of 1993 with respect to their separation or dismissal. Respondents were employed per project undertaken by petitioner company and within varying estimated periods indicated in their respective project employment contracts. Citing the employment record of each respondent, petitioner and David M. Consuji averred that respondents services were terminated when their phases of work for which their services were engaged were completed or when the projects themselves were completed. Respondents notices of termination were filed with the DOLE, in compliance with Policy Instruction No. 20, 128[2] superseded by Department Order No.19, series of 1993.129[3] With respect to respondent Generoso G. Melo, petitioner and David M. Consuji maintained the same positions they had against the case of Melos co-complainants.130[4] Petitioner contended that since respondents were terminated by reason of the completion of their respective phases of work in the construction project, their termination was warranted and legal. Issue: Won the respondents are project employees (are they entitled to nominal damages?) Ruling: Respondents were found to be project employees by the Labor Arbiter, the NLRC and the Court of Appeals. Their unanimous finding that respondents are project employees is binding on the Court. It must also be pointed out that respondents have not appealed from such finding by the Court of Appeals. It is only the petitioner that appealed from the decision of the Court of Appeals. The main issue is whether or not respondents, as project employees, are entitled to nominal damages for lack of advance notice of their dismissal.

128 129 130

Page 55 of 95

A project employee is defined under Article 280 of the Labor Code as one whose employment has been fixed for a specific project or undertaking the completion or termination of which has been determined at the time of the engagement of the employee or where the work or services to be performed is seasonal in nature and the employment is for the duration of the season.131[19] In this case, the Labor Arbiter, the NLRC and the Court of Appeals all found that respondents, as project employees, were validly terminated due to the completion of the phases of work for which their services were engaged. However, the Court of Appeals held that respondents were entitled to nominal damages, because petitioner failed to give them advance notice of their termination. The appellate court cited the case of Agabon v. NLRC as basis for the award of nominal damages. The Court holds that Agabon v. NLRC is not applicable to this case, because it involved the dismissal of regular employees for abandonment of work, which is a just cause for dismissal under Article 282 of the Labor Code. 132[20] Although the dismissal was for a cause, the employer therein was required to observe the standard of due process for termination of employment based on just causes under Article 282 of the Labor Code, which procedural due process requirements are enumerated in Section 2, Rule 1, Book VI133[21] of the Omnibus Rules Implementing the Labor Code.134[22] Since the employer therein failed to comply with the twin requirements of notice and hearing, the Court ordered the employer to pay the employees involved nominal damages in the amount of P30,000.00 for failure to observe procedural due process. Unlike in Agabon, respondents, in this case, were not terminated for just cause under Article 282 of the Labor Code. Dismissal based on just causes contemplate acts or omissions attributable to the employee. 135[23] Instead, respondents were terminated due to the completion of the phases of work for which their services were engaged. As project employees, respondents termination is governed by Section 1 (c) and Section 2 (III), Rule XXIII (Termination of Employment), Book V of the Omnibus Rules Implementing the Labor Code. Section 1 (c), Rule XXIII, Book V of the Omnibus Rules Implementing the Labor Code states: Section 1. Security of tenure. (a) In cases of regular employment, the employer shall not terminate the services of an employee except for just or authorized causes as provided by law, and subject to the requirements of due process. xxxx (c) In cases of project employment or employment covered by legitimate contracting or sub-contracting arrangements, no employee shall be dismissed prior to the completion of the project or phase thereof for which the employee was engaged, or prior to the expiration of the contract between the principal and contractor, unless the dismissal is for just or authorized cause subject to the requirements of due process or prior notice, or is brought about by the completion of the phase of the project or contract for which the employee was engaged.136[24]

131 132 133 134 135 136

Page 56 of 95

Records show that respondents were dismissed after the expiration of their respective project employment contracts, and due to the completion of the phases of work respondents were engaged for. Hence, the cited provisions requirements of due process or prior notice when an employee is dismissed for just or authorized cause (under Articles 282 and 283 of the Labor Code) prior to the completion of the project or phase thereof for which the employee was engaged do not apply to this case. JUDY O. DACUITAL, 137 [1] EUGENIO L. MONDANO, JR., JOSEPH GALER, 138 [2] MARIANO MORALES, ROBERTO RUANCE, JOSEPH PORCADILLA, RAULITO PALAD, RICARDO DIGAMON, NONITO PRISCO , EULOGIO M. TUTOR, MELVIN PEPITO, HELYTO N. REYES, 139 [3] RANDOLF C. BALUDO, ALBERTO EPONDOL, RODELO A. SUSPER, 140 [4] EVARISTO VIGORI, 141 [5] JONATHAN P. AYAAY, FELIPE ERILLA, ARIS A. GARCIA, ROY A. GARCIA, and RESTITUTO TAPANAN,Petitioners, - versus -L.M. CAMUS ENGINEERING CORPORATION and/or LUIS M. CAMUS,Respondents. Facts: Respondent L.M. Camus Engineering Corporation (LMCEC) is a domestic corporation duly organized and existing under and by virtue of Philippine laws, engaged in construction, engineering, and air-conditioning business; while respondent Luis M. Camus (Camus) is the company president. Petitioners Judy O. Dacuital (Dacuital), Eugenio L. Mondano, Jr., Joseph Galer (Galer), Mariano Morales, Roberto Ruance (Ruance), Joseph Porcadilla, Raulito Palad (Palad), Ricardo Digamon (Digamon), Nonito Prisco, Eulogio M. Tutor, Melvin Pepito, Helyto N. Reyes (Reyes), Randolf C. Baludo (Baludo), Alberto Epondol, Rodelo A. Susper, Evaristo Vigori, Jonathan P. Ayaay, Felipe Erilla, Aris A. Garcia (Aris), Roy A. Garcia (Roy), and Restituto Tapanan (Tapanan) were hired by LMCEC as welder, tinsmith, pipefitter, and mechanical employees.142[8. During the months of January, February and March 2001, petitioners were required by LMCEC to surrender their identification cards and ATM cards and were ordered to execute contracts of employment. Most of the petitioners did not comply with the directive as they believed that it was only respondents strategy to get rid of petitioners regular status since they would become new employees disregarding their length of service. Petitioners were later dismissed from employment.143[9] Hence, the complaint for illegal dismissal and non-payment of monetary benefits filed by petitioners. Issue: Won petitioners are project employees (The issues boil down to whether the CA was correct in concluding that petitioners were project employees and that their dismissal from employment was legal.)

137 138 139 140 141 142 143

Page 57 of 95

Ruling: We answer in the negative. Even if the questions that need to be settled are factual in nature, this Court nevertheless feels obliged to resolve them due to the incongruent findings of the NLRC and the LA and those of the CA.144[34 Article 280 of the Labor Code distinguishes a project employee from a regular employee in this wise: Article 280. Regular and casual employment.The provisions of written agreement to the contrary notwithstanding and regardless of the oral agreement of the parties, an employment shall be deemed to be regular where the employee has been engaged to perform activities which are usually necessary or desirable in the usual business or trade of the employer, except where the employment has been fixed for a specific project or undertaking the completion or termination of which has been determined at the time of the engagement of the employee or where the work or services to be performed is seasonal in nature and the employment is for the duration of the season. An employment shall be deemed to be casual if it is not covered by the preceding paragraph: Provided, That, any employee who has rendered at least one year of service, whether such service is continuous or broken, shall be considered a regular employee with respect to the activity in which he is employed and his employment shall continue while such activity exists.145[35] A project employee is assigned to a project which begins and ends at determined or determinable times. 146[36] Employees who work under different project employment contracts for several years do not automatically become regular employees; they can remain as project employees regardless of the number of years they work. Length of service is not a controlling factor in determining the nature of ones employment.147[37] Their rehiring is only a natural consequence of the fact that experienced construction workers are preferred.148[38] In fact, employees who are members of a work pool from which a company draws workers for deployment to its different projects do not become regular employees by reason of that fact alone. The Court has consistently held that members of a work pool can either be project employees or regular employees.149[39] The principal test used to determine whether employees are project employees is whether or not the employees were assigned to carry out a specific project or undertaking, the duration or scope of which was specified at the time the employees were engaged for that project.150[40] Admittedly, respondents did not present the employment contracts of petitioners except that of Dacuital. They explained that it was no longer necessary to present the other contracts since petitioners were similarly situated. Having presented one contract, respondents believed that they sufficiently established petitioners status as project employees.

144 145 146 147 148 149 150

Page 58 of 95

Even though the absence of a written contract does not by itself grant regular status to petitioners, such a contract is evidence that petitioners were informed of the duration and scope of their work and their status as project employees. 151 [41] In this case, where no other evidence was offered, the absence of the employment contracts raises a serious question of whether the employees were properly informed at the onset of their employment of their status as project employees.152[42] While it is true that respondents presented the employment contract of Dacuital, the contract does not show that he was informed of the nature, as well as the duration of his employment. In fact, the duration of the project for which he was allegedly hired was not specified in the contract. The pertinent provision thereof is quoted hereunder for easy reference: 3. In accordance with Policy No. 20 of the Labor Code of the Philippines, parties agree that the effective date of this employment is 4-5-00 up to the duration of the DUCTWORK/ELECTRICAL/MECHANICAL phase of the project estimated to be finished in the month of _______, 19______ or earlier.153[43] Even if we assume that under the above provision of the contract, Dacuital was informed of the nature of his employment and the duration of the project, that same contract is not sufficient evidence to show that the other employees were so informed. It is undisputed that petitioners had individual employment contracts, yet respondents opted not to present them on the lame excuse that they were similarly situated as Dacuital. The non-presentation of these contracts gives rise to the presumption that the employees were not informed of the nature and duration of their employment. It is doctrinally entrenched that in illegal dismissal cases, the employer has the burden of proving with clear, accurate, consistent, and convincing evidence that the dismissal was valid. Absent any other proof that the project employees were informed of their status as such, it will be presumed that they are regular employees.154[44] Moreover, Department Order No. 19 (as well as the old Policy Instructions No. 20) requires employers to submit a report of an employees termination to the nearest public employment office everytime the employment is terminated due to the completion of a project.155[45] In this case, there was no evidence that there was indeed such a report. LMCECs failure to file termination reports upon the cessation of petitioners employment was an indication that petitioners were not project but regular employees. Well-established is the rule that regular employees enjoy security of tenure and they can only be dismissed for just or valid cause and upon compliance with due process, i.e., after notice and hearing. In cases involving an employees dismissal, the burden is on the employer to prove that the dismissal was legal. 156[46] This burden was not amply discharged by LMCEC in this case. Being regular employees, petitioners were entitled to security of tenure, and their services may not be terminated except for causes provided by law.157[47] Pangilinan et al. vs General Milling corp Facts:

151 152 153 154 155 156 157

Page 59 of 95

The respondent General Milling Corporation is a domestic corporation engaged in the production and sale of livestock and poultry.2 It is, likewise, the distributor of dressed chicken to various restaurants and establishments nationwide. 3 As such, it employs hundreds of employees, some on a regular basis and others on a casual basis, as "emergency workers." The petitioners4 were employed by the respondent on different dates as emergency workers at its poultry plant in Cainta, Rizal, under separate "temporary/casual contracts of employment" for a period of five months .5 Most of them worked as chicken dressers, while the others served as packers or helpers. 6 Upon the expiration of their respective contracts, their services were terminated. They later filed separate complaints for illegal dismissal and non-payment of holiday pay, 13th month pay, night-shift differential and service incentive leave pay against the respondent before the Arbitration Branch of the National Labor Relations Commission, docketed as NLRC Case No. RAB-IV-9-4519-92-RI; NLRC Case No. RAB-IV-9-4520-92-RI; NLRC Case No. RAB-IV-9-4521-92-RI; NLRC Case No. RAB-IV-9-4541-92-RI; NLRC Case No. RAB-IV-10-4552-92-RI; NLRC Case No. RAB-IV-10-4595-92-RI and NLRC Case No. RAB-IV-11-4599-92-RI. 7 The petitioners alleged that their work as chicken dressers was necessary and desirable in the usual business of the respondent, and added that although they worked from 10:00 p.m. to 6:00 a.m., they were not paid night-shift differential.8 They stressed that based on the nature of their work, they were regular employees of the respondent; hence, could not be dismissed from their employment unless for just cause and after due notice. In support thereof, the petitioners cited the decision of the Honorable Labor Arbiter Perlita B. Velasco in NLRC Case No. NCR-6-2168-86, entitled Estelita Jayme, et al. vs. General Milling Corporation ; and NLRC Case No. NCR-9-3726-86, entitled Marilou Carino, et al. vs. General Milling Corporation.9 They asserted that the respondent GMC terminated their contract of employment without just cause and due notice. They further argued that the respondent could not rely on the nomenclature of their employment as "temporary or casual." Issue: Won the petitioners are regular employees or project employees or employees on the fixed period Ruling: The petitioners are employees in a fixed period. We agree that the petitioners were employees with a fixed period, and, as such, were not regular employees. Article 280 of the Labor Code comprehends three kinds of employees: (a) regular employees or those whose work is necessary or desirable to the usual business of the employer; (b) project employees or those whose employment has been fixed for a specific project or undertaking the completion or termination of which has been determined at the time of the engagement of the employee or where the work or services to be performed is seasonal in nature and the employment is for the duration of the season; and, (c) casual employees or those who are neither regular nor project employees.40 A regular employee is one who is engaged to perform activities which are necessary and desirable in the usual business or trade of the employer as against those which are undertaken for a specific project or are seasonal.41 There are two separate instances whereby it can be determined that an employment is regular: (1) if the particular activity performed by the employee is necessary or desirable in the usual business or trade of the employer; and, (2) if the employee has been performing the job for at least a year.42 In the case of St. Theresa's School of Novaliches Foundation vs. NLRC ,43 we held that Article 280 of the Labor Code does not proscribe or prohibit an employment contract with a fixed period. We furthered that it does not necessarily follow that where the duties of the employee consist of activities usually necessary or desirable in the usual business of the employer, the parties are forbidden from agreeing on a period of time for the performance of such activities. There is thus nothing essentially contradictory between a definite period of employment and the nature of the employee's duties.

Page 60 of 95

Indeed, in the leading case of Brent School Inc. v. Zamora,44 we laid down the guideline before a contract of employment may be held as valid, to wit: [S]tipulations in employment contracts providing for term employment or fixed period employment are valid when the period were agreed upon knowingly and voluntarily by the parties without force, duress or improper pressure, being brought to bear upon the employee and absent any other circumstances vitiating his consent, or where it satisfactorily appears that the employer and employee dealt with each other on more or less equal terms with no moral dominance whatever being exercised by the former over the latter .45 An examination of the contracts entered into by the petitioners showed that their employment was limited to a fixed period, usually five or six months, and did not go beyond such period. G.R. No. 156748. September 8, 2004], [G.R. No. 156896. September 8, 2004]ISAAC CIOCO, JR., REBIE A. MERCADO, BENITO V. GALVADORES, CECILIO SOLVER, CARMELO JUANZO, [1] BENJAMIN BAYSA, and RODRIGO NAPOLES, petitioners , vs . C. E. CONSTRUCTION CORPORATION and/or JOHNNY TAN, respondents .C. E. CONSTRUCTION CORPORATION, petitioner , vs . ISAAC CIOCO, JR., REBIE A. MERCADO, BENITO V. GALVADORES, CECILIO SOLVER, CARMELO JUANZO, BENJAMIN BAYSA, and RODRIGO NAPOLES, respondents. Facts: Isaac Cioco, Jr., Rebie A. Mercado, Benito V. Galvadores, Cecilio Solver, Carmelo Juanzo, Benjamin Baysa, and Rodrigo Napoles (WORKERS) were hired by C.E. Construction Corporation (COMPANY), a domestic corporation engaged in the construction business and managed by its owner-president, Mr. Johnny Tan. They were hired as carpenters and laborers in various construction projects from 1990 to 1999, the latest of which was the GTI Tower in Makati. Prior to the start of every project, the WORKERS signed individual employment contracts which uniformly read[5] I hereby apply as carpenter/laborer on [the] GTI Tower project. It is understood that if accepted, the period of employment shall be co-terminus with the completion of the project, unless sooner terminated by you prior to the completion of the project. I agree to comply with such rules and regulations and instructions which you may issue from time to time and to accomplish such forms and papers pertinent to my employment. In the event my service is terminated prior to completion of the project or upon its completion as above fixed, I hereby release and hold you harmless and free from any claim, demand or cause of actions. Sometime in May and June 1999, the WORKERS, along with sixty-six (66) others, were terminated by the COMPANY on the ground of completion of the phases of the GTI Tower project for which they had been hired. Alleging that they were regular employees, the WORKERS filed complaints for illegal dismissal with the Arbitration Branch of the NLRC. Claims for underpaid wages and unpaid overtime pay, premium for holiday and rest days, service incentive leave pay, night shift differential, and 13th month pay were likewise demanded. Issue: Won the workers are project employees or regular employees Ruling: The issue of whether the WORKERS were regular or project employees of the COMPANY is a question of fact which shall no longer be dealt with in this petition for review, the Courts jurisdiction being limited to questions of law. The Labor

Page 61 of 95

Arbiter, the NLRC, and the CA, unanimously found that the WORKERS were project employees of the COMPANY. This finding is binding on this Court. We again hold that the fact that the WORKERS have been employed with the COMPANY for several years on various projects, the longest being nine (9) years, did not automatically make them regular employees considering that the definition of regular employment in Article 280[11] of the Labor Code, makes specific exception with respect to project employment. The re-hiring of petitioners on a project-to-project basis did not confer upon them regular employment status. The practice was dictated by the practical consideration that experienced construction workers are more preferred.[12] It did not change their status as project employees. ABESCO CONSTRUCTION AND DEVELOPMENT CORPORATION and MR. OSCAR BANZON, General Manager, Petitioners,vs. ALBERTO RAMIREZ, BERNARDO DIWA, MANUEL LOYOLA, REYNALDO P. ACODESIN, ALEXANDER BAUTISTA, EDGAR TAJONERA and GARY DISON, * Respondents. Facts: Petitioner company was engaged in a construction business where respondents were hired on different dates from 1976 to 1992 either as laborers, road roller operators, painters or drivers. In 1997, respondents filed two separate complaints1 for illegal dismissal against the company and its General Manager, Oscar Banzon, before the Labor Arbiter (LA). Petitioners allegedly dismissed them without a valid reason and without due process of law. The complaints also included claims for non-payment of the 13th month pay, five days' service incentive leave pay, premium pay for holidays and rest days, and moral and exemplary damages. The LA later on ordered the consolidation of the two complaints.2 Petitioners denied liability to respondents and countered that respondents were "project employees" since their services were necessary only when the company had projects to be completed. Petitioners argued that, being project employees, respondents' employment was coterminous with the project to which they were assigned. They were not regular employees who enjoyed security of tenure and entitlement to separation pay upon termination from work. After trial, the LA declared respondents as regular employees because they belonged to a "work pool" from which the company drew workers for assignment to different projects, at its discretion. He ruled that respondents were hired and re-hired over a period of 18 years, hence, they were deemed to be regular employees. He likewise found that their employment was terminated without just cause. Issue: WON the respondents are work pool project employees Ruling: We rule that respondents were regular employees. However, we take exception to the reasons cited by the LA (which both the NLRC and the CA affirmed) in considering respondents as regular employees and not as project employees. Contrary to the disquisitions of the LA, employees (like respondents) who work under different project employment contracts for several years do not automatically become regular employees; they can remain as project employees regardless of the number of years they work.7 Length of service is not a controlling factor in determining the nature of one's employment.8 Moreover, employees who are members of a "work pool" from which a company (like petitioner corporation) draws workers for deployment to its different projects do not become regular employees by reason of that fact alone. The Court has enunciated in some cases 9 that members of a "work pool" can either be project employees or regular employees.

Page 62 of 95

The principal test for determining whether employees are "project employees" or "regular employees" is whether they are assigned to carry out a specific project or undertaking, the duration and scope of which are specified at the time they are engaged for that project.10 Such duration, as well as the particular work/service to be performed, is defined in an employment agreement and is made clear to the employees at the time of hiring.11
In this case, petitioners did not have that kind of agreement with respondents. Neither did they inform respondents of the nature of the latter's work at the time of hiring. Hence, for failure of petitioners to substantiate their claim that respondents were project employees, we are constrained to declare them as regular employees. Furthermore, petitioners cannot belatedly argue that respondents continue to be their employees (so as to escape liability for illegal dismissal). Before the LA, petitioners staunchly postured that respondents were only "project employees" whose employment tenure was coterminous with the projects they were assigned to. However, before the CA, they took a different stance by insisting that respondents continued to be their employees. Petitioners' inconsistent and conflicting positions on their true relation with respondents make it all the more evident that the latter were indeed their regular employees. ISMAEL SAMSON, petitioner, vs.NATIONAL LABOR RELATIONS COMMISSION and ATLANTIC GULF AND PACIFIC CO., MANILA, INC., respondents. Facts: Petitioner has been employed with private respondent Atlantic Gulf and Pacific Co., Manila, Inc. (AG & P) in the latter's various construction projects since April, 1965, in the course of which employment he worked essentially as a rigger, from laborer to rigger foreman. From 1977 up to 1985, he was assigned to overseas projects of AG & P, particularly in Kuwait and Saudi Arabia. On November 5, 1989, petitioner filed a complaint for the conversion of his employment status from project employee to regular employee, which complaint was later amended to include claims for underpayment, non-payment of premium pay for holiday and rest day, refund of reserve fund, and 10% thereof as attorney's fees. Petitioner alleged therein that on the basis of his considerable and continuous length of service with AG & P, he should already be considered a regular employee and, therefore, entitled to the benefits and privileges appurtenant thereto. The labor arbiter, in a decision dated June 30, 1993,2 declared that petitioner should be considered a regular employee on the ground that it has not been shown that AG & P had made the corresponding report to the nearest Public Employment Office every time a project wherein petitioner was assigned had been completed and his employment contract terminated, as required under DOLE Policy Instruction No. 20. Furthermore, pursuant to the same policy instruction, the labor arbiter found that since petitioner was not free to leave anytime and to offer his services to other employers, he should be considered an employee for an indefinite period because he is a member of a work pool from which AG & P draws its project employees and is considered an employee thereof during his membership therein, hence the completion of the project does not mean termination of the employer-employee relationship. Issue: Won the petitioner is a project employee Ruling: Petitioner is a regular employee. Reasons First, the factual findings of respondent commission, which is supported by substantial evidence, is already conclusive and binding and, therefore, entitled to respect by this Court. Second, Department Order No. 19 amended Policy Instruction No. 20 by doing away with the required notice of termination upon completion of the project. Hence, non-compliance with the required report, which is only one of the "indicators" for project employment, no longer affixes a prescription of

Page 63 of 95

regular employment, by reason of which the doctrine laid down in the Caramol case no longer applies to the case at bar. In addition, Department Order No. 19 allows the re-hiring of employees without making them regular employees, aside from the fact that the word "rehiring" connotes new employment. Third, on the basis of petitioner's project employment contracts, his services were engaged for a fixed and determinable period which thus makes each employment for every project separate and distinct from one another. Consequently, the labor arbiter supposedly erred in taking into account petitioner's various employments in the past in determining his length of service, considering that upon completion of a project, the services of the project employee are deemed terminated, his employment being coterminous with each project or phase of the project to which he is assigned. Finally, so it is claimed, petitioner should be considered a project employee since he falls under the exception provided for in Article 280 of the Labor Code to the effect that "the provisions of written agreement to the contrary notwithstanding and regardless of the oral agreement of the parties, an employment shall be deemed to be regular where the employee has been engaged to perform activities which are usually necessary or desirable in the usual business or trade of the employer, except where the employment has been fixed for a specific project or undertaking the completion or termination of which has been determined at the time of the engagement of the employee . . . ." The bulk of the problem appears to hinge on the determination of whether or not Department Order No. 19 should be given retroactive effect in order that the notice of termination requirement may be dispensed with in this case for a correlative ruling on the presumption of regularity of employment which normally arises in case of non-compliance therewith. Both the petitioner and the Solicitor General submit that said order can only have prospective application. Private respondent believes otherwise. We find for petitioner. When the present action for regularization was filed on November 5, 1989 6 and during the entire period of petitioner's employment with private respondent prior to said date, the rule in force then was Policy Instruction No. 20 which, in the fourth paragraph thereof, required the employer company to report to the nearest Public Employment Office the fact of termination of a project employee as a result of the completion of the project or any phase thereof in which he is employed. Furthermore, contrary to private respondent's asseveration, Department Order No. 19, which was issued on April 1, 1993, did not totally dispense with the notice requirement but, instead, made provisions therefor and considered it as one of the "indicators" that a worker is a project employee. This is evident in Section 2.2 thereof which provides that: 2.2 Indicators of project employment. Either one or more of the following circumstances, among others, may be considered as indicators that an employee is a project employee. (a) The duration of the specific/identified undertaking for which the worker is engaged is reasonably determinable. (b) Such duration, as well as the specific work/service to be performed, is defined in an employment agreement and is made clear to the employee at the time of hiring. (c) The work/service performed by the employee is in connection with the particular project/undertaking for which he is engaged. (d) The employee, while not employed and awaiting engagement, is free to offer his services to any other employer. (e) The termination of his employment in the particular project/undertaking is reported to the Department of Labor and Employment (DOLE) Regional Office having jurisdiction over the workplace within 30 days following the date of his separation from work, using the prescribed form on employees' terminations dismissals suspensions.

Page 64 of 95

(f) An undertaking in the employment contract by the employer to pay completion bonus to the project employee as practiced by most construction companies. (Emphasis supplied) More importantly, it must be emphasized that the notice of termination requirement has been retained by express provision of Department Order No. 19 under Section 6.1 thereof, to wit: 6.1. Requirements of labor and social legislations. (a) The construction company and the general contractor and/or subcontractor referred to in Sec. 2.5 shall be responsible for the workers in its employ on matters of compliance with the requirements of existing laws and regulations on hours of work, wages, wage related benefits, health, safety and social welfare benefits, including submission to the DOLE-Regional Office of Work Accident/Illness Report, Monthly Report on Employees' Terminations/Dismissals/Suspensions and other reports. . . . (Emphasis ours.) Perforce, we agree with the labor arbiter that private respondent's failure to report the termination of petitioner's services to the nearest Public Employment Office, after completion of every project or a phase thereof to which he is assigned, is a clear indication that petitioner was not and is not a project employee. On the bases of the foregoing, the retroactivity or prospectivity of Department Order No. 19 would normally be of no moment. At any rate, even if the new issuance has expressly superseded Policy Instruction No. 20, the same cannot be given retroactive effect as such an application would be prejudicial to the employees and would run counter to the constitutional mandate on social justice and protection to labor. Furthermore, this view that we take is more in accord with the avowed purpose of Department Order No. 19 "to ensure the protection and welfare of workers employed" in the construction industry, and which interpretation may likewise be inferred from a reading of Section 7 thereof, applied corollarily to this case, which provides that "nothing herein shall be construed to authorize the diminution or reduction of benefits being enjoyed by employees at the time of issuance hereof." It is a basic and irrefragable rule that in carrying out and interpreting the provisions of the Labor Code and its implementing regulations, the workingman's welfare should be the primordial and paramount consideration. The interpretation herein handed down gives meaning and substance to the liberal and compassionate spirit of the law enunciated in Article 4 of the Labor Code that "all doubts in the implementation and interpretation of the provisions of the Labor Code including its implementing rules and regulations shall be resolved in favor of labor."7 The mandate in Article 281 of the Labor Code, which pertinently prescribes that "the provisions of written agreement to the contrary notwithstanding and regardless of the oral agreements of the parties, an employment shall be deemed to be regular where the employee has been engaged to perform activities which are usually necessary or desirable in the usual business or trade of the employer" and that "any employee who has rendered at least one year of service, whether such service is continuous or broken shall be considered a regular employee with respect to the activity in which he is employed and his employment shall continue while such actually exists," should apply in the case of herein petitioner. It is not disputed that petitioner had been working for private respondent for approximately twenty-eight (28) years as of the adjudication of his plaint by respondent NLRC, and that his "project-to-project" employment was renewed several times. With the successive contracts of employment wherein petitioner continued to perform virtually the same kind of work, i.e., as rigger, throughout his period of employment, it is manifest that petitioner's assigned tasks were usually necessary or desirable in the usual business or trade of private respondent. 8 The repeated re-hiring and continuing need for his services are sufficient evidence of the necessity and indispensability of such services to private respondent's business or trade.9 Where from the circumstances it is apparent that periods have been imposed to preclude the acquisition of tenurial security by the employee, they should be struck down as contrary to public policy, morals, good customs or public order. 10 As observed by the Solicitor General, the record of this case discloses, as part of petitioner's position paper, a certification 11 duly issued by private respondent clearly showing that the former's services were engaged by private respondent on a continuing basis since 1965. The certification indubitably indicates that after a particular project has been accomplished, petitioner would be re-hired immediately the following day save for a gap of one (1) day to one (1)

Page 65 of 95

week from the last project to the succeeding one. 12 There can, therefore, be no escape from the conclusion that petitioner is a regular employee of private respondent. ABS CBN vs Nareno Facts: Petitioner ABS-CBN Broadcasting Corporation (ABS-CBN) is engaged in the broadcasting business and owns a network of television and radio stations, whose operations revolve around the broadcast, transmission, and relay of telecommunication signals. It sells and deals in or otherwise utilizes the airtime it generates from its radio and television operations. It has a franchise as a broadcasting company, and was likewise issued a license and authority to operate by the National Telecommunications Commission.

Petitioner employed respondents Nazareno, Gerzon, Deiparine, and Lerasan as production assistants (PAs) on different dates. They were assigned at the news and public affairs, for various radio programs in the Cebu Broadcasting Station, with a monthly compensation of P4,000. They were issued ABS-CBN employees identification cards and were required to work for a minimum of eight hours a day, including Sundays and holidays. They were made to perform the following tasks and duties: a) Prepare, arrange airing of commercial broadcasting based on the daily operations log and digicart of respondent ABS-CBN; b) c) d) e) f) Coordinate, arrange personalities for air interviews; Coordinate, prepare schedule of reporters for scheduled news reporting and lead-in or incoming reports; Facilitate, prepare and arrange airtime schedule for public service announcement and complaints; Assist, anchor program interview, etc; and Record, log clerical reports, man based control radio.158[4]

nazareno was terminated. Issue: Won nazareno is a project or regular employee of ABS CBN Ruling: Nazareno is a regular employee. Not considered regular employees are project employees, the completion or termination of which is more or less determinable at the time of employment, such as those employed in connection with a particular construction project, and seasonal employees whose employment by its nature is only desirable for a limited period of time. Even then, any employee who has rendered at least one year of service, whether continuous or intermittent, is deemed regular with respect to the activity performed and while such activity actually exists.

158

Page 66 of 95

It is of no moment that petitioner hired respondents as talents. The fact that respondents received pre-agreed talent fees instead of salaries, that they did not observe the required office hours, and that they were permitted to join other productions during their free time are not conclusive of the nature of their employment. Respondents cannot be considered talents because they are not actors or actresses or radio specialists or mere clerks or utility employees. They are regular employees who perform several different duties under the control and direction of ABS-CBN executives and supervisors. Thus, there are two kinds of regular employees under the law: (1) those engaged to perform activities which are necessary or desirable in the usual business or trade of the employer; and (2) those casual employees who have rendered at least one year of service , whether continuous or broken, with respect to the activities in which they are employed.159[35 The law overrides such conditions which are prejudicial to the interest of the worker whose weak bargaining situation necessitates the succor of the State. What determines whether a certain employment is regular or otherwise is not the will or word of the employer, to which the worker oftentimes acquiesces, much less the procedure of hiring the employee or the manner of paying the salary or the actual time spent at work. It is the character of the activities performed in relation to the particular trade or business taking into account all the circumstances, and in some cases the length of time of its performance and its continued existence .160[36] It is obvious that one year after they were employed by petitioner, respondents became regular employees by operation of law.161[37] POSEIDON FISHING/TERRY DE JESUS, petitioners, vs.NATIONAL LABOR RELATIONS COMMISSION and JIMMY S. ESTOQUIA, Respondents . Facts: Petitioner Poseidon Fishing is a fishing company engaged in the deep-sea fishing industry. Its various vessels catch fish in the outlying islands of the Philippines, which are traded and sold at the Navotas Fish Port. One of its boat crew was private respondent Jimmy S. Estoquia.3 Petitioner Terry de Jesus is the manager of petitioner company. Private respondent was employed by Poseidon Fishing in January 1988 as Chief Mate. After five years, he was promoted to Boat Captain. In 1999, petitioners, without reason, demoted respondent from Boat Captain to Radio Operator of petitioner Poseidon.4 As a Radio Operator, he monitored the daily activities in their office and recorded in the duty logbook the names of the callers and time of their calls.5 On 3 July 2000, private respondent failed to record a 7:25 a.m. call in one of the logbooks. However, he was able to record the same in the other logbook. Consequently, when he reviewed the two logbooks, he noticed that he was not able to record the said call in one of the logbooks so he immediately recorded the 7:25 a.m. call after the 7:30 a.m. entry.6 Around 9:00 oclock in the morning of 4 July 2000, petitioner Terry de Jesus detected the error in the entry in the logbook. Subsequently, she asked private respondent to prepare an incident report to explain the reason for the said oversight.7 At around 2:00 oclock in the afternoon of that same day, petitioner Poseidons secretary, namely Nenita Laderas, summoned private respondent to get his separation pay amounting to Fifty-Five Thousand Pesos (P55,000.00). However, he refused to accept the amount as he believed that he did nothing illegal to warrant his immediate discharge from work.8

159 160 161

Page 67 of 95

Rising to the occasion, private respondent filed a complaint for illegal dismissal on 11 July 2000 with the Labor Arbiter, alleging nonpayment of wages with prayer for back wages, damages, attorneys fees, and other monetary benefits. Issue: Won the respondents are project employees Ruling: Respondents are regular employees. Petitioners next assert that deep-sea fishing is a seasonal industry because catching of fish could only be undertaken for a limited duration or seasonal within a given year. Thus, according to petitioners, private respondent was a seasonal or project employee. We are not won over. As correctly pointed out by the Court of Appeals, the "activity of catching fish is a continuous process and could hardly be considered as seasonal in nature."33 In Philex Mining Corp. v. National Labor Relations Commission,34 we defined project employees as those workers hired (1) for a specific project or undertaking, and (2) the completion or termination of such project has been determined at the time of the engagement of the employee. The principal test for determining whether particular employees are "project employees" as distinguished from "regular employees," is whether or not the "project employees" were assigned to carry out a "specific project or undertaking," the duration and scope of which were specified at the time the employees were engaged for that project. In this case, petitioners have not shown that private respondent was informed that he will be assigned to a "specific project or undertaking." As earlier noted, neither has it been established that he was informed of the duration and scope of such project or undertaking at the time of their engagement. More to the point, in Maraguinot, Jr. v. National Labor Relations Commission ,35 we ruled that once a project or work pool employee has been: (1) continuously, as opposed to intermittently, re-hired by the same employer for the same tasks or nature of tasks; and (2) these tasks are vital, necessary and indispensable to the usual business or trade of the employer, then the employee must be deemed a regular employee. In fine, inasmuch as private respondents functions as described above are no doubt "usually necessary or desirable in the usual business or trade" of petitioner fishing company and he was hired continuously for 12 years for the same nature of tasks, we are constrained to say that he belongs to the ilk of regular employee. Being one, private respondents dismissal without valid cause was illegal. And, where illegal dismissal is proven, the worker is entitled to back wages and other similar benefits without deductions or conditions.36 Indeed, it behooves this Court to be ever vigilant in checking the unscrupulous efforts of some of our entrepreneurs, primarily aimed at maximizing their return on investments at the expense of the lowly workingman.37 PLDT vs Ylagan Facts: In November 1992, respondent Mayflor T. Ylagan was hired as an accounting clerk in the Cost Accounting Department (CAD) of petitioner Philippine Long Distance Telephone Company, Inc. (PLDT). In January 1994, she was transferred to the Revenue Auditing Department. Later, on July 3, 1995, she was brought back to the CAD to perform the same accounting duties.

Page 68 of 95

Respondent claims that in May 1996, PLDT refused to renew her employment unless she signed up with an employment agency known as Corporate Executive Search, Inc. (CESI). She was allegedly constrained to sign an employment contract with the agency in order to keep her job with PLDT. But on February 5, 1997, PLDT allegedly refused to allow her to report for work since her employment contract with CESI had already expired. PLDT, however, maintains that respondent was hired as a project employee assigned to the Employment Payroll System Project from the onset of her employment. The project allegedly started on September 21, 1992. It was discontinued in March 1997 when a new system was developed to replace it. PLDT asserts that respondents project employment was covered by contracts for the period of July 3, 1995 to October 2, 1995 and October 3, 1995 to January 2, 1996. Hence, respondent was not dismissed from her work; her employment contract merely expired as of January 2, 1996. PLDT, however, did not explain why respondent had to sign up with CESI in May 1996. Issue: Won the respondents are project employees Ruling: Various indicators convince us that respondent was not a project employee but a regular employee who was illegally dismissed. A project employee is assigned to carry out a specific project or undertaking the duration and scope of which are specified at the time the employee is engaged in the project. 162[9] A project is a job or undertaking which is distinct, separate and identifiable from the undertakings of the company. 163[10] A project employee is assigned to a project which begins and ends at determined or determinable times.164[11] Various indicators convince us that respondent was not a project employee but a regular employee who was illegally dismissed. First, respondent worked continuously for PLDT from November 1992 to July 1995 without any mention of a project to which she was specifically assigned. She was hired to perform accounting duties which were not shown as distinct, separate and identifiable from the usual undertakings of the company. Although essentially a telephone company, PLDT maintains its own accounting department to which respondent was assigned. Second, aside from its statement that respondent was hired as a project employee for the Employment Payroll System Project which began in 1992, PLDT did not provide evidence of the project employment contracts covering the period from November 1992 (when respondent was hired) to July 1995. PLDT mentioned only two contracts but these pertained to her employment period from July 1995 to January 1996. Third, despite the supposed expiration of respondents project employment contract on January 2, 1996, respondent continued to work for PLDT until May 2, 1996 when respondent was required to sign up with CESI. 165[12] Respondent worked for PLDT, under contract with CESI, until February 3, 1997. PLDT explained that it no longer allowed respondent to report for work by then since the project was already done. But the project was only completed in March 1997.

162 163 164 165

Page 69 of 95

Most important of all, based on the records, PLDT did not report the termination of respondents supposed project employment to the Department of Labor and Employment as project employee. Department Order No. 19 (as well as the old Policy Instructions No. 20) required employers to submit a report of an employees termination to the nearest public employment office every time his employment was terminated due to a completion of a project. 166[13] PLDTs failure to file termination reports was an indication that the respondent was not a project employee but a regular employee.167[14] The test to determine whether employment is regular or not is the reasonable connection between the particular activity performed by the employee in relation to the usual business or trade of the employer. Also, if the employee has been performing the job for at least one year, even if the performance is not continuous or merely intermittent, the law deems the repeated and continuing need for its performance as sufficient evidence of the necessity, if not indispensability of that activity to the business. Thus, we held that where the employment of project employees is extended long after the supposed project has been finished, the employees are removed from the scope of project employees and are considered regular employees.168[15] (emphasis ours) While length of time may not be the controlling test for project employment, it is crucial in determining if the employee is hired for a specific undertaking to perform functions vital, necessary and indispensable to the usual business of the company.169[16] Here, respondent worked in PLDTs accounting department from November 1992 to July 1995 without any project employment contract. Her employment was continuous until she was constrained to sign up with CESI in May 1996. It was almost a year later when she was no longer allowed to report for work in PLDT due to the alleged expiration of her contract with CESI. PLDT asserts that even if respondent rendered continuous service for a year or so, she could not be deemed a regular employee because the services performed were not necessary or desirable to the usual trade or business of the company. On this, we agree with the CA: It is absurd to argue that services rendered by the [respondent] as an accounting clerk to the accounting and auditing department of PLDT in relation to its PEPS project (computerization of employees[] payroll system) is not necessary or desirable to the companys business. There wont be any business without any workforce xxx. Employees render their services for a certain payment or compensation. Thus, [respondents] job pertaining to effective payroll system is part and parcel [of] the usual business of PLDT. Further, [the CA] is reminded of the jurisprudence that the character of employment is determined not by stipulations in the contract, but by the nature of the work performed and that an employee is regular because of the nature of the work and the length of service, not because of the mode or even the reason for hiring them. 170[17] Thus, the scheme of PLDT in adopting the Project Employment Agreement where the [respondent] was made to sign and the design of referring [respondent] to an employment agency undoubtedly bolster its intention of stripping from [respondent] the privileges earned by the status of her [regular] employment.171[18] (citations omitted)

166 167 168 169 170 171

Page 70 of 95

Even assuming that respondent was hired as a project employee from the onset, we have ruled that once such an employee has been: (1) continuously, as opposed to intermittently, re-hired by the same employer for the same tasks or nature of tasks and (2) these tasks are vital, necessary and indispensable to the usual business or trade of the employer, then the employee must be deemed a regular employee.172[19] From the foregoing, the duration (of at least one year) and necessity of respondents employment have been established. She was therefore a regular employee of PLDT. As such, respondent was entitled to all the privileges and benefits attached to that status. BAHIA SHIPPING SERVICES, INC., petitioner, vs.REYNALDO CHUA, respondent. Facts: Private respondent Reynaldo Chua was hired by the petitioner shipping company, Bahia Shipping Services, Inc., as a restaurant waiter on board a luxury cruise ship liner M/S Black Watch pursuant to a Philippine Overseas Employment Administration (POEA) approved employment contract dated October 9, 1996 for a period of nine (9) months from October 18, 1996 to July 17, 1997. On October 18, 1996, the private respondent left Manila for Heathrow, England to board the said sea vessel where he will be assigned to work. On February 15, 1997, the private respondent reported for his working station one and one-half (1) hours late. On February 17, 1997, the master of the vessel served to the private respondent an official warning-termination form pertaining to the said incident. On March 8, 1997, the vessel's master, ship captain Thor Fleten conducted an inquisitorial hearing to investigate the said incident. Thereafter, on March 9, 1997, private respondent was dismissed from the service on the strength of an unsigned and undated notice of dismissal. An alleged record or minutes of the said investigation was attached to the said dismissal notice. On March 24, 1997, the private respondent filed a complaint for illegal dismissal and other monetary claims, which case was assigned to Labor Arbiter Manuel M. Manansala. The LA declared the dismissal of respondent illegal for the reason that the infraction he committed of being tardy by 1 hour should not have been penalized by petitioner with the ultimate punishment of termination; rather, the commensurate penalty for such single tardiness would have been suspension for one or two weeks. The LA further noted that petitioner meted out on respondent the penalty of dismissal hastily and summarily in that it merely went through the motions of notifying respondent and hearing his side when, all along, it had already decided to dismiss him.9 The NLRC sustained the foregoing findings of the LA, noting that the claim of petitioner that respondent's tardiness was not infrequent but habitual is not supported by evidence. 10 However, the NLRC held that, although the penalty of dismissal on respondent was properly lifted, a penalty of deduction of one day's salary, the same to be subtracted from his monetary award, should be imposed on the latter for the tardiness he incurred. 11The CA held that the NLRC and LA did not commit any grave abuse of discretion in arriving at the factual assessments which are all supported by substantial evidence.12 Issue: Won the respondent is a project employee Ruling: No discussion about project employees but this talks about the contract/fixed term contract entered into by the seafarers.

172

Page 71 of 95

The Court had occasion to rule on a similar issue in Stolt-Nielsen Marine Services (Phils.), Inc. v. National Labor Relations Commission,27 where the NLRC was questioned for awarding to an illegally dismissed overseas worker fixed overtime pay equivalent to the unexpired portion of the latter's contract. In resolving the question, the Court, citing Cagampan v. National Labor Relations Commission,28 held that although an overseas employment contract may guarantee the right to overtime pay, entitlement to such benefit must first be established, otherwise the same cannot be allowed. Hence, it being improbable that respondent rendered overtime work during the unexpired term of his contract, the inclusion of his "guaranteed overtime" pay into his monthly salary as basis in the computation of his salaries for the entire unexpired period of his contract has no factual or legal basis and the same should have been disallowed. VIRGILIO SAPIO, PETITIONER, VS. UNDALOC CONSTRUCTION AND/OR ENGR. CIRILO UNDALOC, RESPONDENTS. Facts: The controversy started with a complaint filed by petitioner against Undaloc Construction and/or Engineer Cirilo Undaloc for illegal dismissal, underpayment of wages and nonpayment of statutory benefits. Respondent Undaloc Construction, a single proprietorship owned by Cirilo Undaloc, is engaged in road construction business in Cebu City. Petitioner had been employed as watchman from 1 May 1995 to 30 May 1998 when he was terminated on the ground that the project he was assigned to was already finished, he being allegedly a project employee. Petitioner asserted he was a regular employee having been engaged to perform works which are "usually necessary or desirable" in respondents' business. He claimed that from 1 May to 31 August 1995 and from 1 September to 31 December 1995, his daily wage rate was only P80.00 and P90.00, respectively, instead of P121.87 as mandated by Wage Order No. ROVII-03. From 1 March 1996 to 30 May 1998, his daily rate was P105.00. He further alleged that he was made to sign two payroll sheets, the first bearing the actual amount he received wherein his signature was affixed to the last column opposite his name, and the second containing only his name and signature. To buttress this allegation, petitioner presented the payroll sheet covering the period from 4 to 10 December 1995 in which the entries were written in pencil. He also averred that his salary from 18 to 30 May 1998 was withheld by respondents. [5] For its part, respondent Cirilo Undaloc maintained that petitioner was hired as a project employee on 1 May 1995 and was assigned as watchman from one project to another until the termination of the project on 30 May 1998. [6] Refuting the claim of underpayment, respondent presented the payroll sheets from 2 September to 8 December 1996, 26 May to 15 June 1997, and 12 January to 31 May 1998. [7] That petitioner was a project employee became a non-issue beginning with the decision of the Labor Arbiter. Contested still is his entitlement to salary differential, apart from attorney's fees. Issue: Won the petitioner is a project employee. Ruling: No discussion about project employee. Absent any indication sufficient enough to support a conclusion, we cannot uphold the findings of the Labor Arbiter and the NLRC.[19] The conclusion of the Labor Arbiter that entries in the December 1995 payroll sheet could have been altered is utterly baseless. The claim that the December 1995 payroll sheet was written in pencil and was thus rendered it prone to alterations or erasures is clearly non sequitur. The same is true with respect to the typewritten payroll sheets. In fact, neither the Labor Arbiter nor the NLRC found any alteration or erasure or traces thereat, whether on the pencil-written or typewritten payroll sheets. Indeed, the most minute examination will not reveal any tampering. Furthermore, if there is any adverse conclusion as regards the December 1995 payroll sheet, it must be confined only to it and cannot be applied

Page 72 of 95

to

the

typewritten

payroll

sheets.

Moreover, absent any evidence to the contrary, good faith must be presumed in this case. Entries in the payroll, being entries in the course of business, enjoy the presumption of regularity under Rule 130, Section 43 of the Rules of Court. Hence, while as a general rule, the burden of proving payment of monetary claims rests on the employer, [20] when fraud is alleged in the preparation of the payroll, the burden of evidence shifts to the employee and it is incumbent upon him to adduce clear and convincing evidence in support of his claim. [21] Unfortunately, petitioner's bare assertions of fraud do not suffice to overcome the disputable presumption of regularity. While we adhere to the position of the appellate court that the "tendency" to alter the entries in the payrolls was not substantiated, we cannot however subscribe to the total deletion of the award of salary differential and attorney's fees, as it so ruled. The Labor Arbiter granted a salary differential of P24,902.88. [22]

The Labor Arbiter erred in his computation. He fixed the daily wage rate actually received by petitioner at P105.00 [23] without taking into consideration the P141.00 rate indicated in the typewritten payroll sheets submitted by respondents. Moreover, the Labor Arbiter misapplied the wage orders[24] when he wrongly categorized respondent as falling within the first category. Based on the stipulated number of employees and audited financial statements, [25] respondents should have been covered by the second category. To avoid further delay in the disposition of this case which is not in consonance with the objective of speedy justice, we have to adjudge the rightful computation of the salary differential based on the applicable wage orders. After all, the supporting records are complete. This Court finds that from 1 January to 30 August 1996 and 1 July 1997 to 31 May 1998, petitioner had received a wage less than the minimum mandated by law. Therefore, he is entitled to a salary differential. d. Piece Rate Workers Labor congress of the Phils. Et al. vs NLRC Facts: It rooted on a chaotic strike due to ULP and charges of underpayment of wages. The LA ruled: Anent the charge that there was underpayment of wages, the evidence points to the contrary. The enumeration of complainants' wages in their consolidated Affidavits of merit and position paper which implies underpayment has no leg to stand on in the light of the fact that complainants' admission that they are piece workers or paid on a pakiao [basis] i.e. a certain amount for every thousand pieces of cheese curls or other products repacked. The only limitation for piece workers or pakiao workers is that they should receive compensation no less than the minimum wage for an eight (8) hour work [sic]. And compliance therewith was satisfactorily explained by respondent Gonzalo Kehyeng in his testimony (TSN, p. 12-30) during the July 31, 1991 hearing. On cross-examination, complainants failed to rebut or deny Gonzalo Kehyeng's testimony that complainants have been even receiving more than the minimum wage for an average workers [sic]. Certainly, a lazy worker earns less than the minimum wage but the same cannot be attributable to respondents but to the lazy workers. NLRC affirmed. Issue: WON the employees as piece rate workers cannot be considered as regular employees Ruling:

Page 73 of 95

That petitioner employees are "pakyao" or piece workers does not imply that they are not regular employees entitled to reinstatement. Private respondent Empire Food Products, Inc. is a food and fruit processing company. In Tabas v. California Manufacturing Co., Inc. (169 SCRA 497), this Honorable Court held that the work of merchandisers of processed food, who coordinate with grocery stores and other outlets for the sale of the processed food is necessary in the day-to-day operation[s] of the company. With more reason, the work of processed food repackers is necessary in the day-to-day operation[s] of respondent Empire Food Products. 10 It may likewise be stressed that the burden of proving the existence of just cause for dismissing an employee, such as abandonment, rests on the employer, 11 a burden private respondents failed to discharge. Private respondents, moreover, in considering petitioners' employment to have been terminated by abandonment, violated their rights to security of tenure and constitutional right to due process in not even serving them with a written notice of such termination. 12 Section 2, Rule XIV, Book V of the Omnibus Rules Implementing the Labor Code provides: Sec. 2. Notice of Dismissal Any employer who seeks to dismiss a worker shall furnish him a written notice stating the particular acts or omission constituting the grounds for his dismissal. In cases of abandonment of work, the notice shall be served at the worker's last known address. Petitioners are therefore entitled to reinstatement with full back wages pursuant to Article 279 of the Labor Code, as amended by R.A. No. 6715. Nevertheless, the records disclose that taking into account the number of employees involved, the length of time that has lapsed since their dismissal, and the perceptible resentment and enmity between petitioners and private respondents which necessarily strained their relationship, reinstatement would be impractical and hardly promotive of the best interests of the parties. In lieu of reinstatement then, separation pay at the rate of one month for every year of service, with a fraction of at least six (6) months of service considered as one (1) year, is in order. 13 That being said, the amount of back wages to which each petitioner is entitled, however, cannot be fully settled at this time. Petitioners, as piece-rate workers having been paid by the piece, 14 there is need to determine the varying degrees of production and days worked by each worker. Clearly, this issue is best left to the National Labor Relations Commission. As to the other benefits, namely, holiday pay, premium pay, 13th month pay and service incentive leave which the labor arbiter failed to rule on but which petitioners prayed for in their complaint, 15 we hold that petitioners are so entitled to these benefits. Three (3) factors lead us to conclude that petitioners, although piece-rate workers, were regular employees of private respondents. First, as to the nature of petitioners' tasks, their job of repacking snack food was necessary or desirable in the usual business of private respondents, who were engaged in the manufacture and selling of such food products; second, petitioners worked for private respondents throughout the year, their employment not having been dependent on a specific project or season; and third, the length of time 16 that petitioners worked for private respondents. Thus, while petitioners' mode of compensation was on a "per piece basis," the status and nature of their employment was that of regular employees. The Rules Implementing the Labor Code exclude certain employees from receiving benefits such as nighttime pay, holiday pay, service incentive leave 17 and 13th month pay, 18 inter alia, "field personnel and other employees whose time and performance is unsupervised by the employer, including those who are engaged on task or contract basis, purely commission basis, or those who are paid a fixed amount for performing work irrespective of the time consumed in the performance thereof." Plainly, petitioners as piece-rate workers do not fall within this group. As mentioned earlier, not only did petitioners labor under the control of private respondents as their employer, likewise did petitioners toil throughout the year with the fulfillment of their quota as supposed basis for compensation. Further, in Section 8 (b), Rule IV, Book III which we quote hereunder, piece workers are specifically mentioned as being entitled to holiday pay. Sec. 8. Holiday pay of certain employees.

Page 74 of 95

(b) Where a covered employee is paid by results or output, such as payment on piece work, his holiday pay shall not be less than his average daily earnings for the last seven (7) actual working days preceding the regular holiday: Provided, however, that in no case shall the holiday pay be less than the applicable statutory minimum wage rate. In addition, the Revised Guidelines on the Implementation of the 13th Month Pay Law, in view of the modifications to P.D. No. 851 19 by Memorandum Order No. 28, clearly exclude the employer of piece rate workers from those exempted from paying 13th month pay, to wit: 2. EXEMPTED EMPLOYERS The following employers are still not covered by P.D. No. 851: d. Employers of those who are paid on purely commission, boundary or task basis, and those who are paid a fixed amount for performing specific work, irrespective of the time consumed in the performance thereof, except where the workers are paid on piece-rate basis in which case the employer shall grant the required 13th month pay to such workers . (emphasis supplied) The Revised Guidelines as well as the Rules and Regulations identify those workers who fall under the piece-rate category as those who are paid a standard amount for every piece or unit of work produced that is more or less regularly replicated, without regard to the time spent in producing the same. 20 As to overtime pay, the rules, however, are different. According to Sec. 2(e), Rule I, Book III of the Implementing Rules, workers who are paid by results including those who are paid on piece-work, takay, pakiao, or task basis, if their output rates are in accordance with the standards prescribed under Sec. 8, Rule VII, Book III, of these regulations, or where such rates have been fixed by the Secretary of Labor in accordance with the aforesaid section, are not entitled to receive overtime pay. Here, private respondents did not allege adherence to the standards set forth in Sec. 8 nor with the rates prescribed by the Secretary of Labor. As such, petitioners are beyond the ambit of exempted persons and are therefore entitled to overtime pay. Once more, the National Labor Relations Commission would be in a better position to determine the exact amounts owed petitioners, if any. AVELINO LAMBO and VICENTE BELOCURA, petitioners, vs. NATIONAL LABOR RELATIONS COMMISSION and J.C. TAILOR SHOP and/or JOHNNY CO, respondents . Facts: Petitioners Avelino Lambo and Vicente Belocura were employed as tailors by private respondents J.C. Tailor Shop and/or Johnny Co on September 10, 1985 and March 3, 1985, respectively. They worked from 8:00 a.m. to 7:00 p.m. daily, including Sundays and holidays. As in the case of the other 100 employees of private respondents, petitioners were paid on a piece-work basis, according to the style of suits they made. Regardless of the number of pieces they finished in a day, they were each given a daily pay of at least P64.00. On January 17, 1989, petitioners filed a complaint against private respondents for illegal dismissal and sought recovery of overtime pay, holiday pay, premium pay on holiday and rest day, service incentive leave pay, separation pay, 13th month pay, and attorneys fees. Issue: Won petitioners are piece rate employees Ruling: Yes piece rate but that wont neglect their status as regular employees.

Page 75 of 95

There is no dispute that petitioners were employees of private respondents although they were paid not on the basis of time spent on the job but according to the quantity and the quality of work produced by them. There are two categories of employees paid by results: (1) those whose time and performance are supervised by the employer. (Here, there is an element of control and supervision over the manner as to how the work is to be performed. A piece-rate worker belongs to this category especially if he performs his work in the company premises.); and (2) those whose time and performance are unsupervised. (Here, the employers control is over the result of the work. Workers on pakyao and takay basis belong to this group.) Both classes of workers are paid per unit accomplished. Piece-rate payment is generally practiced in garment factories where work is done in the company premises, while payment on pakyao and takay basis is commonly observed in the agricultural industry, such as in sugar plantations where the work is performed in bulk or in volumes difficult to quantify.i[4] Petitioners belong to the first category, i.e., supervised employees. In determining the existence of an employer-employee relationship, the following elements must be considered: (1) the selection and engagement of the employee; (2) the payment of wages; (3) the power of dismissal; and (4) the power to control the employees conduct.ii[5] Of these elements, the most important criterion is whether the employer controls or has reserved the right to control the employee not only as to the result of the work but also as to the means and methods by which the result is to be accomplished.iii[6] In this case, private respondents exercised control over the work of petitioners. As tailors, petitioners worked in the companys premises from 8:00 a.m. to 7:00 p.m. daily, including Sundays and holidays. The mere fact that they were paid on a piece-rate basis does not negate their status as regular employees of private respondents. The term wage is broadly defined in Art. 97 of the Labor Code as remuneration or earnings, capable of being expressed in terms of money whether fixed or ascertained on a time, task, piece or commission basis. Payment by the piece is just a method of compensation and does not define the essence of the relations. iv[7] Nor does the fact that petitioners are not covered by the SSS affect the employer-employee relationship. Indeed, the following factors show that petitioners, although piece-rate workers, were regular employees of private respondents: (1) within the contemplation of Art. 280 of the Labor Code, their work as tailors was necessary or desirable in the usual business of private respondents, which is engaged in the tailoring business; (2) petitioners worked for private respondents throughout the year, their employment not being dependent on a specific project or season; and, (3) petitioners worked for private respondents for more than one year.v[8] Additionally, respondents cannot be considered as project or program employees because no evidence was presented to show that the duration and scope of the project were determined or specified at the time of their engagement. Under existing jurisprudence, project could refer to two distinguishable types of activities. First, a project may refer to a particular job or undertaking that is within the regular or usual business of the employer, but which is distinct and separate, and identifiable as such, from the other undertakings of the company. Such job or undertaking begins and ends at determined or determinable times. Second, the term project may also refer to a particular job or undertaking that is not within the regular business of the employer. Such a job or undertaking must also be identifiably separate and distinct from the ordinary or regular business operations of the employer. The job or undertaking also begins and ends at determined or determinable times.173[38] The principal test is whether or not the project employees were assigned to carry out a specific project or undertaking, the duration and scope of which were specified at the time the employees were engaged for that project.174[39] In this case, it is undisputed that respondents had continuously performed the same activities for an average of five years. Their assigned tasks are necessary or desirable in the usual business or trade of the petitioner. The persisting need for their services is sufficient evidence of the necessity and indispensability of such services to petitioners business or trade.175[40] While length of time may not be a sole controlling test for project employment, it can be a strong factor to

173 174 175

Page 76 of 95

determine whether the employee was hired for a specific undertaking or in fact tasked to perform functions which are vital, necessary and indispensable to the usual trade or business of the employer. 176[41] We note further that petitioner did not report the termination of respondents employment in the particular project to the Department of Labor and Employment Regional Office having jurisdiction over the workplace within 30 days following the date of their separation from work, using the prescribed form on employees termination/suspension/dismissals. MARK ROCHE INTERNATIONAL AND/OR EDUARDO DAYOT and SUSAN DAYOT, petitioners, vs. NATIONAL LABOR RELATIONS COMMISSION, MARK ROCHE WORKERS UNION and WILMA PATACAY, EILEEN RUFON, LILIA BRIONES, BEATRIZ MANAGAYTAY, DELIA ARELLANO, ANITA MARCELO, RIO MARIANO, MARISSA SADILI, ESTRELLA MALLARI, DELIA LAROYA, and DIVINA VILLARBA, respondents. Facts: Petitioners Eduardo Dayot and Susan Dayot were President and Vice President, respectively, of their co-petitioner Mark Roche International (MRI), a corporation organized and existing under the laws of the Philippines, engaged in the garments business. Private respondents Eileen Rufon, Lilia Briones, Beatriz Managaytay, Delia Arellano, Anita Marcelo, Rio Mariano, Marissa Sadili, Wilma Patacay, Estella Mallari, Delia Laroya and Divina Villarba were employed as sewers of MRI with lengths of service varying from three (3) to nine (9) years. On different dates private respondents filed separate complaints for underpayment of wages and non-payment of overtime pay against petitioners MRI, Eduardo Dayot and Susan Dayot. Private respondents alleged that they usually worked eleven (11) to twelve (12) hours daily, except on Mondays during which they worked eight (8) hours, and were paid wages on a piece-rate basis amounting to P450.00 to P600.00 per week. They likewise asserted that sometime in 1992 they were unable to avail of their SSS benefits, e.g., salary loan, sickness benefits and maternity benefits because, as they found out, the company did not remit their contributions to the SSS. On 11 October 1992 private respondents sought the assistance of a labor organization which helped them organize the Mark Roche Workers Union (MRWU). On 14 October 1992 they registered the union with the Department of Labor and Employment National Capital Region (DOLE-NCR) and on the same date filed a Petition for Certification Election before the Med-Arbitration Board. On 27 October 1992 petitioners received a notice of hearing of the petition. Apparently irked by the idea of a union within the company, petitioners ordered private respondents to withdraw the petition and further threatened them that should they insist in the organization of a union they would be dismissed. Unfazed, private respondents refused. As expected, on 29 October 1992 they were discharged from work. On 30 October 1992 private respondents amended their earlier complaints to include as additional causes of action their illegal dismissal, unfair labor practice, non-payment of 13th month pay, underpayment for legal holidays, and for damages. Issue: Won the respondents are piece rate workers Ruling: The respondents are piece rate workers. Finally, as correctly held by the NLRC, private respondents as piece-rate employees are not entitled to service incentive leave pay as well as holiday pay even if they are entitled to other benefits like COLA and 13th month pay. Service

176

Page 77 of 95

incentive leave pay shall not apply to employees whose performance is unsupervised by the employer, including those who are paid in a fixed amount for performing work irrespective of the time consumed in the performance thereof.7 WHEREFORE, this Court finds that private respondents Eileen Rufon, Lilia Briones, Beatriz Managaytay, Delia Arellano, Anita Marcelo, Rio Mariano, Marissa Sadili, Wilma Patacay, Estrella Malari, Delia Laroya and Divina Villarba were illegally dismissed not merely illegally constructively dismissed by petitioners Mark Roche International and/or Eduardo Dayot and Susan Dayot, and to this extent, the assailed Decision of public respondent National Labor Relations Commission affirming that of the Labor Arbiter, is MODIFIED. However, it is AFFIRMED insofar as it ordered the reinstatement of private respondents with back wages, salary differentials and 13th month pay. The service incentive leave pay awarded by the Labor Arbier but deleted by the National Labor Relations Commission is likewise DELETED. 1wphi1.nt e. probationary employees CEBU MARINE BEACH RESORT, OFELIA PELAEZ AND TSUYOSHI SASAKI, petitioners, vs.NATIONAL LABOR RELATIONS COMMISSION (FOURTH DIVISION), RIC RODRIGO RODRIGUEZ, MANULITO VILLEGAS and LORNA G. IGOT, respondents. Facts: Cebu Marine Beach Resort (herein petitioner company), a single proprietorship owned by Victor Dualan, commenced its operations sometime in January, 1990 with the recruitment of its employees, including Ric Rodrigo Rodriguez, Manulita Villegas and Lorna G. Igot, respondents. On the last week of March, 1990 when Japanese tourists began arriving at the resort, petitioner company became fully operational.Inasmuch as the beach resort was intended to cater principally to Japanese tourists, respondents had to undergo a special training in Japanese customs, traditions, discipline as well as hotel and resort services. This special training was supervised by Tsuyoshi Sasaki, also a petitioner. During a seminar conducted on May 24, 1990, petitioner Sasaki suddenly scolded respondents and hurled brooms, floor maps, iron trays, fire hoses and other things at them. In protest, respondents staged a walk-out and gathered in front of the resort. Immediately, petitioner Sasaki reacted by shouting at them to go home and never to report back to work. Heeding his directive, respondents left the premises. Eventually, they filed with the Regional Arbitration Branch at Cebu City a complaint for illegal dismissal and other monetary claims against petitioners. On May 28, 1990, petitioner company, through its acting general manager, Ofelia Pelaez, also a petitioner, sent letters to respondents requiring them to explain why they should not be terminated from employment on the grounds of abandonment of work and failure to qualify with the standards for probationary employees. Issue: whether respondents were illegally dismissed from employment by petitioner company (as probationary employees) Ruling: Yes they are illegally dismissed. We hold that the Court of Appeals did not err when it ruled that respondents were illegally dismissed from the service. It is settled that while probationary employees do not enjoy permanent status, they are entitled to the constitutional protection of security of tenure. Their employment may only be terminated for just cause or when they fail to qualify as regular employees in accordance with reasonable standards made known to them by their employer at the time of engagement, and after due process .5

Page 78 of 95

Here, petitioners terminated respondents probationary employment on the grounds of abandonment and failure to qualify for the positions for which they were employed. On this point, we quote with approval the findings of the Court of Appeals, thus: "x x x. It is undisputed that Mr. Sasaki made an utterance to the effect that private respondents should go home and never come back to work for the company again. Such utterance is tantamount to a dismissal. Its meaning is also clear and unmistakable no matter which accent was used by Mr. Sasaki. Considering further that Mr. Sasaki was in charge of the training of the private respondents, his words carry authority and conviction. Even assuming for the sake of argument that Mr. Sasaki was never vested with the power of dismissal, the petitioner company ratified Mr. Sasakis acts. When petitioner company sent a strongly worded memorandum to private respondents asking them to explain why their services should not be terminated for failure to live up to the companys expectations, it showed intention to terminate. x x x"x x x "The subsequent issuances of the memos were, as rightly interpreted by the public respondent, merely an afterthought to escape the legal liability arising from the illegal termination of the private respondents services. That respondents failed to qualify for their positions, suffice it to state that at the time they were dismissed, they were still in a "trial period" or probationary period. Being in the nature of a "trial period," the essence of a probationary period of employment fundamentally lies in the purpose or objective sought to be attained by both the employer and the employee during said period. While the employer observes the fitness, propriety and efficiency of a probationer to ascertain whether he is qualified for permanent employment, the probationer, on the other hand, seeks to prove to the employer that he has the qualifications to meet the reasonable standards for permanent employment which obviously were made known to him.7 To reiterate, in the case at bar, far from allowing the respondents to prove that they possessed the qualifications to meet the reasonable standards for their permanent employment, petitioners peremptorily dismissed them from the service. On another tack, petitioners argument that the Appellate Courts award of full backwages and separation pay in effect unilaterally extended respondents 6-month probationary employment is bereft of merit. In Philippine Manpower Services, Inc. vs. NLRC ,8 we held that "absent the grounds for termination of a probationary employee, he is entitled to continued employment even beyond the probationary period." On a similar note, our ruling in Lopez vs. Javier9 is quite explicit, thus: "x x x, probationary employees who are unjustly dismissed from work during the probationary period shall be entitled to reinstatement and payment of full backwages and other benefits and privileges from the time they were dismissed up to their actual reinstatement , conformably with Article 279 of the Labor Code, as amended by Section 34 of Republic Act No. 6715, which took effect on March 21, 1989: x x x An employee who is unjustly dismissed from work shall be entitled to reinstatement without loss of seniority rights and other privileges and to his full backwages, inclusive of allowances, and to his other benefits or their monetary equivalent computed from the time his compensation was withheld from him up to the time of his actual reinstatement." **** (!!!!!) check mistsubishi case!!!! RADIN C. ALCIRA, petitioner, vs. NATIONAL LABOR RELATIONS COMMISSION, MIDDLEBY PHILIPPINES CORPORATION/FRANK THOMAS, XAVIER G. PEA and TRIFONA F. MAMARADLO, respondents . Facts: (caveat) the recent ruling is mitsubishi motors vs chrysler phils labor union GR No. 148738 June 29, 2004- regarding the computation of the last days of 180 days trial period. Respondent Middleby Philippines Corporation (Middleby) hired petitioner as engineering support services supervisor on a probationary basis for six months. Apparently unhappy with petitioners performance, respondent Middleby terminated

Page 79 of 95

petitioners services. The bone of contention centered on whether the termination occurred before or after the six-month probationary period of employment. The parties, presenting their respective copies of Alciras appointment paper, claimed conflicting starting dates of employment: May 20, 1996 according to petitioner and May 27, 1996 according to respondent. Both documents indicated petitioners employment status as probationary (6 mos.) and a remark that after five months (petitioners) performance shall be evaluated and any adjustment in salary shall depend on (his) work performance.177 [6] Petitioner asserts that, on November 20, 1996, in the presence of his co-workers and subordinates, a senior officer of respondent Middleby in bad faith withheld his time card and did not allow him to work. Considering this as a dismissal after the lapse of his probationary employment, petitioner filed on November 21, 1996 a complaint in the National Labor Relations Commission (NLRC) against respondent Middleby contending that he had already become a regular employee as of the date he was illegally dismissed. Included as respondents in the complaint were the following officers of respondent Middleby: Frank Thomas (General Manager), Xavier Pea (Human Resources Manager) and Trifona Mamaradlo (Engineering Manager). In their defense, respondents claim that, during petitioners probationary employment, he showed poor performance in his assigned tasks, incurred ten absences, was late several times and violated company rules on the wearing of uniform. Since he failed to meet company standards, petitioners application to become a regular employee was disapproved and his employment was terminated. Issue: WON the petitioners dismissal as probationary employee is valid Ruling: Petitioner insists that he already attained the status of a regular employee when he was dismissed on November 20, 1996 because, having started work on May 20, 1996, the six-month probationary period ended on November 16, 1996. According to petitioners computation, since Article 13 of the Civil Code provides that one month is composed of thirty days, six months total one hundred eighty days. As the appointment provided that petitioners status was probationary (6 mos.) without any specific date of termination, the 180th day fell on November 16, 1996. Thus, when he was dismissed on November 20, 1996, he was already a regular employee. Petitioners contention is incorrect. In CALS Poultry Supply Corporation, et. al. vs. Roco, et. al.,178 [11] this Court dealt with the same issue of whether an employment contract from May 16, 1995 to November 15, 1995 was within or outside the six-month probationary period. We ruled that November 15, 1995 was still within the six-month probationary period. We reiterate our ruling in CALS Poultry Supply: (O)ur computation of the 6-month probationary period is reckoned from the date of appointment up to the same calendar date of the 6th month following.(italics supplied) In short, since the number of days in each particular month was irrelevant, petitioner was still a probationary employee when respondent Middleby opted not to regularize him on November 20, 1996. The second issue is whether respondent Middleby informed petitioner of the standards for regularization at the start of his employment.

177 178

Page 80 of 95

Section 6 (d) of Rule 1 of the Implementing Rules of Book VI of the Labor Code (Department Order No. 10, Series of 1997) provides that: xxx xxx xxx

(d) In all cases of probationary employment, the employer shall make known to the employee the standards under which he will qualify as a regular employee at the time of his engagement. Where no standards are made known to the employee at that time, he shall be deemed a regular employee. xxx xxx xxx

We hold that respondent Middleby substantially notified petitioner of the standards to qualify as a regular employee when it apprised him, at the start of his employment, that it would evaluate his supervisory skills after five months. In Orient Express Placement Philippines vs. National Labor Relations Commission,179[12] we ruled that an employer failed to inform an employee of the reasonable standards for becoming a regular employee: Neither private respondent's Agency-Worker Agreement with ORIENT EXPRESS nor his Employment Contract with NADRICO ever mentioned that he must first take and pass a Crane Operator's License Examination in Saudi Arabia before he would be allowed to even touch a crane. Neither did he know that he would be assigned as floorman pending release of the results of the examination or in the event that he failed; more importantly, that he would be subjected to a performance evaluation by his superior one (1) month after his hiring to determine whether the company was amenable to continuing with his employment. Hence, respondent Flores could not be faulted for precisely harboring the impression that he was hired as crane operator for a definite period of one (1) year to commence upon his arrival at the work-site and to terminate at the end of one (1) year. No other condition was laid out except that he was to be on probation for three (3) months.(emphasis supplied) Conversely, an employer is deemed to substantially comply with the rule on notification of standards if he apprises the employee that he will be subjected to a performance evaluation on a particular date after his hiring. We agree with the labor arbiter when he ruled that: In the instant case, petitioner cannot successfully say that he was never informed by private respondent of the standards that he must satisfy in order to be converted into regular status. This rans ( sic) counter to the agreement between the parties that after five months of service the petitioners performance would be evaluated. It is only but natural that the evaluation should be made vis--vis the performance standards for the job. Private respondent Trifona Mamaradlo speaks of such standard in her affidavit referring to the fact that petitioner did not perform well in his assigned work and his attitude was below par compared to the companys standard required of him.180 [13] The third issue for resolution is whether petitioner was illegally dismissed when respondent Middleby opted not to renew his contract on the last day of his probationary employment. It is settled that even if probationary employees do not enjoy permanent status, they are accorded the constitutional protection of security of tenure. This means they may only be terminated for just cause or when they otherwise fail to qualify as regular employees in accordance with reasonable standards made known to them by the employer at the time of their engagement.181 [14] But we have also ruled in Manlimos, et. al. vs. National Labor Relations Commission 182 [15] that this constitutional protection ends on the expiration of the probationary period. On that date, the parties are free to either renew or

179 180 181

Page 81 of 95

terminate their contract of employment. Manlimos concluded that (t)his development has rendered moot the question of whether there was a just cause for the dismissal of the petitioners xxx. 183 [16] In the case at bar, respondent Middleby exercised its option not to renew the contract when it informed petitioner on the last day of his probationary employment that it did not intend to grant him a regular status. Although we can regard petitioners severance from work as dismissal, the same cannot be deemed illegal. As found by the labor arbiter, the NLRC and the Court of Appeals, petitioner (1) incurred ten absences (2) was tardy several times (3) failed to wear the proper uniform many times and (4) showed inferior supervisory skills. Petitioner failed to satisfactorily refute these substantiated allegations. Taking all this in its entirety, respondent Middleby was clearly justified to end its employment relationship with petitioner. MITSUBISHI MOTORS PHILIPPINES CORPORATION, petitioner, vs.CHRYSLER PHILIPPINES LABOR UNION and NELSON PARAS, respondents. (the prevailing ruling) Facts: Mitsubishi Motors Philippines Corporation (MMPC) is a domestic corporation engaged in the assembly and distribution of Mitsubishi motor vehicles. Chrysler Philippines Labor Union (CPLU) is a legitimate labor organization and the duly certified bargaining agent of the hourly-paid regular rank and file employees of MMPC. Nelson Paras was a member of CPLU. His wife, Cecille Paras, was the President of the Chrysler Philippines Salaried Employees Union (CPSU). Nelson Paras was first employed by MMPC as a shuttle bus driver on March 19, 1976. He resigned on June 16, 1982. He applied for and was hired as a diesel mechanic and heavy equipment operator in Saudi Arabia from 1982 to 1993. When he returned to the Philippines, he was re-hired as a welder-fabricator at the MMPC tooling shop from October 3, 1994 to October 31, 1994.2 On October 29, 1994, his contract was renewed from November 1, 1994 up to March 3, 1995.3 Sometime in May of 1996, Paras was re-hired on a probationary basis as a manufacturing trainee at the Plant Engineering Maintenance Department. He and the new and re-hired employees were given an orientation on May 15, 1996 4 by Emma P. Aninipot, respecting the companys history, corporate philosophy, organizational structure, and company rules and regulations, including the company standards for regularization, code of conduct and company-provided benefits.5 Paras started reporting for work on May 27, 1996. He was assigned at the paint ovens, air make-up and conveyors. As part of the MMPCs policy, Paras was evaluated by his immediate supervisors Lito R. Lacambacal 6 and Wilfredo J. Lopez7 after six (6) months, and received an average rating. Later, Lacambacal informed Paras that based on his performance rating, he would be regularized.8 However, the Department and Division Managers, A.C. Velando and H.T. Victoria, 9 including Mr. Dante Ong,10 reviewed the performance evaluation made on Paras. They unanimously agreed, along with Paras immediate supervisors, that the performance of Paras was unsatisfactory.11 As a consequence, Paras was not considered for regularization. On November 26, 1996, he received a Notice of Termination dated November 25, 1996, informing him that his services were terminated effective the said date since he failed to meet the required company standards for regularization.12 Utilizing the grievance machinery in the collective bargaining agreement, the CPLU demanded the settlement of the dispute which arose from Paras termination.13 The dispute was thereafter submitted for voluntary arbitration, as the parties were unable to agree on a mutually acceptable solution. CPLU posited that Paras was dismissed on his one hundred eighty third (183rd) day of employment, or three (3) days after the expiration of the probationary period of six (6) months. It was contended that Paras was already a regular employee on the date of the termination of his "probationary employment."

182 183

Page 82 of 95

Issue: When is the last ay of 180days trial period in probationary employment? Ruling:

Regularization of Employment
Indeed, an employer, in the exercise of its management prerogative, may hire an employee on a probationary basis in order to determine his fitness to perform work.29 Under Article 281 of the Labor Code, the employer must inform the employee of the standards for which his employment may be considered for regularization. Such probationary period, unless covered by an apprenticeship agreement, shall not exceed six (6) months from the date the employee started working. The employees services may be terminated for just cause or for his failure to qualify as a regular employee based on reasonable standards made known to him.30 Respondent Paras was employed as a management trainee on a probationary basis. During the orientation conducted on May 15, 1996, he was apprised of the standards upon which his regularization would be based. He reported for work on May 27, 1996. As per the companys policy, the probationary period was from three (3) months to a maximum of six (6) months. Applying Article 13 of the Civil Code,31 the probationary period of six (6) months consists of one hundred eighty (180) days.32 This is in conformity with paragraph one, Article 13 of the Civil Code, which provides that the months which are not designated by their names shall be understood as consisting of thirty (30) days each. The number of months in the probationary period, six (6), should then be multiplied by the number of days within a month, thirty (30); hence, the period of one hundred eighty (180) days. As clearly provided for in the last paragraph of Article 13, in computing a period, the first day shall be excluded and the last day included. Thus, the one hundred eighty (180) days commenced on May 27, 1996, and ended on November 23, 1996. The termination letter dated November 25, 1996 was served on respondent Paras only at 3:00 a.m. of November 26, 1996. He was, by then, already a regular employee of the petitioner under Article 281 of the Labor Code. DUSIT HOTEL NIKKO, Petitioner, vs.RENATO M. GATBONTON, Respondent Facts: On November 21, 1998,4 respondent Renato M. Gatbonton was hired as Chief Steward in petitioner Dusit Hotel Nikkos Food and Beverage Department. He signed a three-month probationary employment contract until February 21, 1999, 5 with a monthly salary of P25,000. At the start of his employment, the standards by which he would be assessed to qualify for regular employment were explained to him. The hotel alleged that at the end of the probation period, Ingo Rauber, Director of its Food and Beverage Department, observed that Gatbonton failed to meet the qualification standards for Chief Steward, and Rauber recommended a twomonth extension of Gatbontons probationary period, or until April 22, 1999. At the end of the 4th month, on March 24, 1999, Rauber informed Gatbonton that the latter had poor ratings on staff supervision, productivity, quantity of work, and overall efficiency and did not qualify as Chief Steward. Gatbonton requested another month or until April 22, 1999 to improve his performance, to which Rauber agreed but allegedly refused to sign the Performance Evaluation Form. Neither did he sign the Memorandum on the extension. On March 31, 1999, a notice6 of termination of probationary employment effective April 9, 1999, on the above alleged grounds was served on Gatbonton. On April 12, 1999, he filed a complaint for illegal dismissal and non-payment of wages, with prayers for reinstatement, full backwages, and damages, including attorneys fees.

Page 83 of 95

Issue: Won his probationary employment termination is valid Ruling: He is illegally dismissed. ART. 281. Probationary Employment . -- Probationary employment shall not exceed six (6) months from the date the employee started working, unless it is covered by an apprenticeship agreement stipulating a longer period. The services of an employee who has been engaged on a probationary basis may be terminated for a just cause or when he fails to qualify as a regular employee in accordance with reasonable standards made known by the employer to the employee at the time of his engagement. An employee who is allowed to work after a probationary period shall be considered a regular employee. As Article 281 clearly states, a probationary employee can be legally terminated either: (1) for a just cause; or (2) when the employee fails to qualify as a regular employee in accordance with the reasonable standards made known to him by the employer at the start of the employment. 10 Nonetheless, the power of the employer to terminate an employee on probation is not without limitations. First, this power must be exercised in accordance with the specific requirements of the contract. Second, the dissatisfaction on the part of the employer must be real and in good faith, not feigned so as to circumvent the contract or the law; and third, there must be no unlawful discrimination in the dismissal. In termination cases, the burden of proving just or valid cause for dismissing an employee rests on the employer.11 Here, the petitioner did not present proof that the respondent was evaluated from November 21, 1998 to February 21, 1999, nor that his probationary employment was validly extended. The petitioner alleged that at the end of the respondents three-month probationary employment, Rauber recommended that the period be extended for two months since respondent Gatbonton was not yet ready for regular employment. 12 The petitioner presented a Personnel Action Form13 containing the recommendation. We observed, however, that this document was prepared on March 31, 1999, the end of the 4th month of the respondents employment. In fact, the recommended action was termination of probationary employment effective April 9, 1999, and not extension of probation period.14 Upon appeal to the NLRC, the petitioner presented another Personnel Action Form 15 prepared on March 2, 1999, showing that the respondents probationary employment was extended for two months effective February 23, 1999. The Personnel Action Form dated March 2, 1999, contained the following remarks: "subject to undergo extension of probation for two (2) months as per attached memo." Yet, we find this document inconclusive. First, the action form did not contain the results of the respondents evaluation. Without the evaluation, the action form had no basis. Second, the action form spoke of an attached memo which the petitioner identified as Raubers Memorandum, recommending the extension of the respondents probation period for two months. Again, the supposed Memorandum was not presented. Third, the action form did not bear the respondents signature. In the absence of any evaluation or valid extension, we cannot conclude that respondent failed to meet the standards of performance set by the hotel for a chief steward. At the expiration of the three-month period, Gatbonton had become a regular employee. It is an elementary rule in the law on labor relations that a probationary employee engaged to work beyond the probationary period of six months, as provided under Article 281 of the Labor Code, or for any length of time set forth by the employer (in this case, three months), shall be considered a regular employee. 16 This is clear in the last sentence of Article 281. Any circumvention of this provision would put to naught the States avowed protection for labor. Since respondent was not dismissed for a just or authorized cause, his dismissal was illegal, and he is entitled to reinstatement without loss of seniority rights, and other privileges as well as to full backwages, inclusive of allowances, and to other benefits or their monetary equivalent computed from the time his compensation was withheld from him up to the time of his actual reinstatement.

Page 84 of 95

YOLANDA M. MERCADO,CHARITO S. DE LEON, DIANA R. LACHICA, MARGARITO M. ALBA, JR., and FELIX A. TONOG,Petitioners,- versus -AMA COMPUTER COLLEGE- PARAAQUE CITY Respondent. Facts: AMACC is an educational institution engaged in computer-based education in the country. One of AMACCs biggest schools in the country is its branch at Paraaque City. The petitioners were faculty members who started teaching at AMACC on May 25, 1998. The petitioner Mercado was engaged as a Professor 3, while petitioner Tonog was engaged as an Assistant Professor 2. On the other hand, petitioners De Leon, Lachica and Alba, Jr., were all engaged as Instructor 1.184[5] The petitioners executed individual Teachers Contracts for each of the trimesters that they were engaged to teach, with the following common stipulation:185[6]

1. POSITION. The TEACHER has agreed to accept a non-tenured appointment to work in the College of xxx
effective xxx to xxx or for the duration of the last term that the TEACHER is given a teaching load based on the assignment duly approved by the DEAN/SAVP-COO. [Emphasis supplied] For the school year 2000-2001, AMACC implemented new faculty screening guidelines, set forth in its Guidelines on the Implementation of AMACC Faculty Plantilla.186[7] Under the new screening guidelines, teachers were to be hired or maintained based on extensive teaching experience, capability, potential, high academic qualifications and research background. The performance standards under the new screening guidelines were also used to determine the present faculty members entitlement to salary increases. The petitioners failed to obtain a passing rating based on the performance standards; hence AMACC did not give them any salary increase. 187[8] Because of AMACCs action on the salary increases, the petitioners filed a complaint with the Arbitration Branch of the NLRC on July 25, 2000, for underpayment of wages, non-payment of overtime and overload compensation, 13th month pay, and for discriminatory practices.188[9] On September 7, 2000, the petitioners individually received a memorandum from AMACC, through Human Resources Supervisor Mary Grace Beronia, informing them that with the expiration of their contract to teach, their contract would no longer be renewed. Issue: Won the probationary employment was validly terminated Ruling: They are illegally dismissed. a. Rule on Employment on Probationary Status A reality we have to face in the consideration of employment on probationary status of teaching personnel is that they are not governed purely by the Labor Code. The Labor Code is supplemented with respect to the period of probation

184 185 186 187 188

Page 85 of 95

by special rules found in the Manual of Regulations for Private Schools. 189[27] On the matter of probationary period , Section 92 of these regulations provides: Section 92. Probationary Period. Subject in all instances to compliance with the Department and school requirements , the probationary period for academic personnel shall not be more than three (3) consecutive years of satisfactory service for those in the elementary and secondary levels, six (6) consecutive regular semesters of satisfactory service for those in the tertiary level, and nine (9) consecutive trimesters of satisfactory service for those in the tertiary level where collegiate courses are offered on a trimester basis. [Emphasis supplied] The CA pointed this out in its decision (as the NLRC also did), and we confirm the correctness of this conclusion. Other than on the period, the following quoted portion of Article 281 of the Labor Code still fully applies: x x x The services of an employee who has been engaged on a probationary basis may be terminated for a just cause when he fails to qualify as a regular employee in accordance with reasonable standards made known by the employer to the employee at the time of his engagement . An employee who is allowed to work after a probationary period shall be considered a regular employee. [Emphasis supplied] b. Fixed- period Employment The use of employment for fixed periods during the teachers probationary period is likewise an accepted practice in the teaching profession. We mentioned this in passing in Magis Young Achievers Learning Center v. Adelaida P. Manalo,190 [28] albeit a case that involved elementary, not tertiary, education, and hence spoke of a school year rather than a semester or a trimester. We noted in this case: The common practice is for the employer and the teacher to enter into a contract, effective for one school year. At the end of the school year, the employer has the option not to renew the contract, particularly considering the teachers performance. If the contract is not renewed, the employment relationship terminates. If the contract is renewed, usually for another school year, the probationary employment continues. Again, at the end of that period, the parties may opt to renew or not to renew the contract. If renewed, this second renewal of the contract for another school year would then be the last year since it would be the third school year of probationary employment. At the end of this third year, the employer may now decide whether to extend a permanent appointment to the employee, primarily on the basis of the employee having met the reasonable standards of competence and efficiency set by the employer. For the entire duration of this threeyear period, the teacher remains under probation. Upon the expiration of his contract of employment, being simply on probation, he cannot automatically claim security of tenure and compel the employer to renew his employment contract. It is when the yearly contract is renewed for the third time that Section 93 of the Manual becomes operative, and the teacher then is entitled to regular or permanent employment status. It is important that the contract of probationary employment specify the period or term of its effectivity. The failure to stipulate its precise duration could lead to the inference that the contract is binding for the full three-year probationary period. We have long settled the validity of a fixed-term contract in the case Brent School, Inc. v. Zamora191[29] that AMACC cited. Significantly, Brent happened in a school setting. Care should be taken, however, in reading Brent in the context of this case as Brent did not involve any probationary employment issue; it dealt purely and simply with the validity of a fixed-term employment under the terms of the Labor Code, then newly issued and which does not expressly contain a provision on fixed-term employment.

189 190 191

Page 86 of 95

c. Academic and Management Prerogative Last but not the least factor in the academic world, is that a school enjoys academic freedom a guarantee that enjoys protection from the Constitution no less. Section 5(2) Article XIV of the Constitution guarantees all institutions of higher learning academic freedom.192[30] The institutional academic freedom includes the right of the school or college to decide and adopt its aims and objectives, and to determine how these objections can best be attained, free from outside coercion or interference, save possibly when the overriding public welfare calls for some restraint. The essential freedoms subsumed in the term academic freedom encompass the freedom of the school or college to determine for itself: (1) who may teach; (2) who may be taught; (3) how lessons shall be taught; and (4) who may be admitted to study.193[31] AMACCs right to academic freedom is particularly important in the present case, because of the new screening guidelines for AMACC faculty put in place for the school year 2000-2001. We agree with the CA that AMACC has the inherent right to establish high standards of competency and efficiency for its faculty members in order to achieve and maintain academic excellence. The schools prerogative to provide standards for its teachers and to determine whether or not these standards have been met is in accordance with academic freedom that gives the educational institution the right to choose who should teach.194[32] In Pea v. National Labor Relations Commission,195[33] we emphasized: It is the prerogative of the school to set high standards of efficiency for its teachers since quality education is a mandate of the Constitution. As long as the standards fixed are reasonable and not arbitrary, courts are not at liberty to set them aside. Schools cannot be required to adopt standards which barely satisfy criteria set for government recognition. The same academic freedom grants the school the autonomy to decide for itself the terms and conditions for hiring its teacher, subject of course to the overarching limitations under the Labor Code. Academic freedom, too, is not the only legal basis for AMACCs issuance of screening guidelines. The authority to hire is likewise covered and protected by its management prerogative the right of an employer to regulate all aspects of employment, such as hiring, the freedom to prescribe work assignments, working methods, process to be followed, regulation regarding transfer of employees, supervision of their work, lay-off and discipline, and dismissal and recall of workers.196[34] Thus, AMACC has every right to determine for itself that it shall use fixed-term employment contracts as its medium for hiring its teachers. It also acted within the terms of the Manual of Regulations for Private Schools when it recognized the petitioners to be merely on probationary status up to a maximum of nine trimesters.

The Conflict: Probationary Status and Fixed- term Employment


The existence of the term-to-term contracts covering the petitioners employment is not disputed, nor is it disputed that they were on probationary status not permanent or regular status from the time they were employed on May 25, 1998 and until the expiration of their Teaching Contracts on September 7, 2000. As the CA correctly found, their teaching stints only covered a period of at least seven (7) consecutive trimesters or two (2) years and three (3) months of service. This case, however, brings to the fore the essential question of which, between the two factors affecting employment, should prevail given AMACCs position that the teachers contracts expired and it had the right not to renew them . In other words, should the teachers probationary status be disregarded simply because the contracts were fixed-term?

192 193 194 195 196

Page 87 of 95

The provision on employment on probationary status under the Labor Code 197[35] is a primary example of the fine balancing of interests between labor and management that the Code has institutionalized pursuant to the underlying intent of the Constitution.198[36] On the one hand, employment on probationary status affords management the chance to fully scrutinize the true worth of hired personnel before the full force of the security of tenure guarantee of the Constitution comes into play. 199[37] Based on the standards set at the start of the probationary period, management is given the widest opportunity during the probationary period to reject hirees who fail to meet its own adopted but reasonable standards.200[38] These standards, together with the just201[39] and authorized causes202[40] for termination of employment the Labor Code expressly provides, are the grounds available to terminate the employment of a teacher on probationary status. For example, the school may impose reasonably stricter attendance or report compliance records on teachers on probation, and reject a probationary teacher for failing in this regard, although the same attendance or compliance record may not be required for a teacher already on permanent status. At the same time, the same just and authorizes causes for dismissal under the Labor Code apply to probationary teachers, so that they may be the first to be laid-off if the school does not have enough students for a given semester or trimester. Termination of employment on this basis is an authorized cause under the Labor Code.203[41] Labor, for its part, is given the protection during the probationary period of knowing the company standards the new hires have to meet during the probationary period, and to be judged on the basis of these standards , aside from the usual standards applicable to employees after they achieve permanent status. Under the terms of the Labor Code, these standards should be made known to the teachers on probationary status at the start of their probationary period, or at the very least under the circumstances of the present case, at the start of the semester or the trimester during which the probationary standards are to be applied. Of critical importance in invoking a failure to meet the probationary standards, is that the school should show as a matter of due process how these standards have been applied . This is effectively the second notice in a dismissal situation that the law requires as a due process guarantee supporting the security of tenure provision,204[42] and is in furtherance, too, of the basic rule in employee dismissal that the employer carries the burden of justifying a dismissal.205[43] These rules ensure compliance with the limited security of tenure guarantee the law extends to probationary employees.206[44] When fixed-term employment is brought into play under the above probationary period rules, the situation as in the present case may at first blush look muddled as fixed-term employment is in itself a valid employment mode under

197 198 199 200 201 202 203 204 205 206

Page 88 of 95

Philippine law and jurisprudence.207[45] The conflict, however, is more apparent than real when the respective nature of fixed-term employment and of employment on probationary status are closely examined. The fixed-term character of employment essentially refers to the period agreed upon between the employer and the employee; employment exists only for the duration of the term and ends on its own when the term expires. In a sense, employment on probationary status also refers to a period because of the technical meaning probation carries in Philippine labor law a maximum period of six months, or in the academe, a period of three years for those engaged in teaching jobs. Their similarity ends there, however, because of the overriding meaning that being on probation connotes, i.e., a process of testing and observing the character or abilities of a person who is new to a role or job.208[46] Understood in the above sense, the essentially protective character of probationary status for management can readily be appreciated. But this same protective character gives rise to the countervailing but equally protective rule that the probationary period can only last for a specific maximum period and under reasonable, well-laid and properly communicated standards. Otherwise stated, within the period of the probation, any employer move based on the probationary standards and affecting the continuity of the employment must strictly conform to the probationary rules. Under the given facts where the school year is divided into trimesters, the school apparently utilizes its fixed-term contracts as a convenient arrangement dictated by the trimestral system and not because the workplace parties really intended to limit the period of their relationship to any fixed term and to finish this relationship at the end of that term. If we pierce the veil, so to speak, of the parties so-called fixed-term employment contracts, what undeniably comes out at the core is a fixed-term contract conveniently used by the school to define and regulate its relations with its teachers during their probationary period. To be sure, nothing is illegitimate in defining the school-teacher relationship in this manner. The school, however, cannot forget that its system of fixed-term contract is a system that operates during the probationary period and for this reason is subject to the terms of Article 281 of the Labor Code. Unless this reconciliation is made, the requirements of this Article on probationary status would be fully negated as the school may freely choose not to renew contracts simply because their terms have expired. The inevitable effect of course is to wreck the scheme that the Constitution and the Labor Code established to balance relationships between labor and management. Given the clear constitutional and statutory intents, we cannot but conclude that in a situation where the probationary status overlaps with a fixed-term contract not specifically used for the fixed term it offers , Article 281 should assume primacy and the fixed-period character of the contract must give way. This conclusion is immeasurably strengthened by the petitioners and the AMACCs hardly concealed expectation that the employment on probation could lead to permanent status, and that the contracts are renewable unless the petitioners fail to pass the schools standards. To highlight what we mean by a fixed-term contract specifically used for the fixed term it offers, a replacement teacher, for example, may be contracted for a period of one year to temporarily take the place of a permanent teacher on a oneyear study leave. The expiration of the replacement teachers contracted term, under the circumstances, leads to no probationary status implications as she was never employed on probationary basis; her employment is for a specific purpose with particular focus on the term and with every intent to end her teaching relationship with the school upon expiration of this term. If the school were to apply the probationary standards (as in fact it says it did in the present case), these standards must not only be reasonable but must have also been communicated to the teachers at the start of the probationary period, or at the very least, at the start of the period when they were to be applied. These terms, in addition to those expressly provided by the Labor Code, would serve as the just cause for the termination of the probationary contract. As explained above, the details of this finding of just cause must be communicated to the affected teachers as a matter of due process.

207 208

Page 89 of 95

AMACC, by its submissions, admits that it did not renew the petitioners contracts because they failed to pass the Performance Appraisal System for Teachers (PAST) and other requirements for regularization that the school undertakes to maintain its high academic standards.209[47] The evidence is unclear on the exact terms of the standards, although the school also admits that these were standards under the Guidelines on the Implementation of AMACC Faculty Plantilla put in place at the start of school year 2000-2001. While we can grant that the standards were duly communicated to the petitioners and could be applied beginning the 1st trimester of the school year 2000-2001, glaring and very basic gaps in the schools evidence still exist. The exact terms of the standards were never introduced as evidence; neither does the evidence show how these standards were applied to the petitioners.210[48] Without these pieces of evidence (effectively, the finding of just cause for the non-renewal of the petitioners contracts), we have nothing to consider and pass upon as valid or invalid for each of the petitioners. Inevitably, the non-renewal (or effectively, the termination of employment of employees on probationary status) lacks the supporting finding of just cause that the law requires and, hence, is illegal. TAMSONS ENTERPRISES, INC., NELSON LEE, LILIBETH ONG and JOHNSON NG, Petitioners,- versus -COURT OF APPEALS and ROSEMARIE L. SY,Respondents. Facts: This case stemmed from a complaint for illegal dismissal with money claims filed by respondent Rosemarie L. Sy (Sy) before the Arbitration Branch, National Capital Region, NLRC, against petitioners Tamsons Enterprises, Inc. (Tamsons), Nelson Lee (Lee), the company President; and Lilibeth Ong (Ong) and Johnson Ng (Ng), her co-employees. From the records, it appears that on September 1, 2006, Sy was hired by Tamsons as Assistant to the President. Despite the title, she did not act as such because, per instruction of Lee, she was directed to act as payroll officer, though she actually worked as a payroll clerk.211[5] On February 24, 2007, 212[6] four days before she completed her sixth month of working in Tamsons, Ng, the Sales Project Manager, called her to a meeting with him and Lee. During the meeting, they informed Sy that her services would be terminated due to inefficiency. She was asked to sign a letter of resignation and quitclaim. She was told not to report for work anymore because her services were no longer needed. On her last day of work, Ong humiliated her in front of her officemates by shouting at her and preventing her from getting her personal things or any other document from the office. During her pre-employment interview, Lee had nice comments about her good work experience and educational background. She was assured of a long-term employment with benefits. Throughout her employment, she earnestly performed her duties, had a perfect attendance record, worked even during brownouts and typhoons, and would often work overtime just to finish her work. Sy claimed that the remarks of her superiors about her alleged inefficiency were ill-motivated and made without any basis. She had been rendering services for almost six (6) months before she was arbitrarily and summarily dismissed. Her dismissal was highly suspicious as it took place barely four (4) days prior to the completion of her six-month probationary period. The petitioners did not show her any evaluation or appraisal report regarding her alleged inefficient performance. As she was terminated without an evaluation on her performance, she was deprived of the opportunity to be regularly part of the company and to be entitled to the benefits and privileges of a regular employee. Worse, she was deprived of her only means of livelihood. For their part, the petitioners asserted that before Sy was hired, she was apprised that she was being hired as a probationary employee for six months from September 1, 2006 to February 28, 2007, subject to extension as a regular employee conditioned on her meeting the standards of permanent employment set by the company. Her work performance was thereafter monitored and evaluated. On February 1, 2007, she was formally informed that her

209 210 211 212

Page 90 of 95

employment would end on February 28, 2007 because she failed to meet the companys standards. From then on, Sy started threatening the families of the petitioners with bodily harm. They pointed out that the unpredictable attitude of Sy was one of the reasons for her not being considered for regular employment. Issue: Won Sy was able to meet the probationary standards an deemed regular employee of the petitioner Ruling: Sy is a regular employee. Art. 281. Probationary employment. Probationary employment shall not exceed six months from the date the employee started working, unless it is covered by an apprenticeship agreement stipulating a longer period. The services of an employee who has been engaged in a probationary basis may be terminated for a just cause or when he fails to qualify as a regular employee in accordance with reasonable standards made known by the employer to the employee at the time of his engagement. An employee who is allowed to work after a probationary period shall be considered a regular employee. (Underscoring supplied) There is probationary employment where the employee upon his engagement is made to undergo a trial period during which the employer determines his fitness to qualify for regular employment based on reasonable standards made known to him at the time of engagement.213[14] The probationary employment is intended to afford the employer an opportunity to observe the fitness of a probationary employee while at work, and to ascertain whether he will become an efficient and productive employee. While the employer observes the fitness, propriety and efficiency of a probationer to ascertain whether he is qualified for permanent employment, the probationer, on the other hand, seeks to prove to the employer that he has the qualifications to meet the reasonable standards for permanent employment. Thus, the word probationary, as used to describe the period of employment, implies the purpose of the term or period, not its length.214[15] On the basis of the aforequoted provisions and definition, there is no dispute that Sys employment with Tamsons on September 1, 2006 was probationary in character. As a probationary employee, her employment status was only temporary. Although a probationary or temporary employee with a limited tenure, she was still entitled to a security of tenure. It is settled that even if probationary employees do not enjoy permanent status, they are accorded the constitutional protection of security of tenure. This means they may only be terminated for a just cause or when they otherwise fail to qualify as regular employees in accordance with reasonable standards made known to them by the employer at the time of their engagement.215[16] Consistently, in Mercado v. AMA Computer College-Paranaque City, Inc.,216[17] this Court clearly stressed that: Labor, for its part, is given the protection during the probationary period of knowing the company standards the new hires have to meet during the probationary period, and to be judged on the basis of these standards, aside from the usual standards applicable to employees after they achieve permanent status. Under the terms of the Labor Code, these standards should be made known to the [employees] on probationary status at the start of their probationary period, or xxx during which the probationary standards are to be applied. Of critical importance in invoking a failure to meet the probationary standards, is that the [employer] should show as a matter of due

213 214 215 216

Page 91 of 95

process how these standards have been applied . This is effectively the second notice in a dismissal situation that the law requires as a due process guarantee supporting the security of tenure provision, and is in furtherance, too, of the basic rule in employee dismissal that the employer carries the burden of justifying a dismissal. These rules ensure compliance with the limited security of tenure guarantee the law extends to probationary employees. 217[18] [Emphases supplied] In this case, the justification given by the petitioners for Sys dismissal was her alleged failure to qualify by the companys standard. Other than the general allegation that said standards were made known to her at the time of her employment, however, no evidence, documentary or otherwise, was presented to substantiate the same. Neither was there any performance evaluation presented to prove that indeed hers was unsatisfactory. Thus, this Court is in full accord with the ruling of the CA when it wrote that: Private respondents were remiss in showing that petitioner failed to qualify as a regular employee. Except for their allegations that she was apprised of her status as probationary and that she would be accorded regular status once she meets their standards, no evidence was presented of these standards and that petitioner had been apprised of them at the time she was hired as a probationary employee. Neither was it shown that petitioner failed to meet such standards. Petitioner should have been informed as to the basis of private respondents decision not to extend her regular or permanent employment. This case is bereft of any proof like an evaluation or assessment report which would support private respondents claim that she failed to comply with the standards in order to become a regular employee. One of the conditions before an employer can terminate a probationary employee is dissatisfaction on the part of the employer which must be real and in good faith, not feigned so as to circumvent the contract or the law. In the case at bar, absent any proof showing that the work performance of petitioner was unsatisfactory, We cannot conclude that petitioner failed to meet the standards of performance set by private respondents. This absence of proof, in fact, leads Us to infer that their dissatisfaction with her work performance was contrived so as not to regularize her employment. 218 [19] For failure of the petitioners to support their claim of unsatisfactory performance by Sy, this Court shares the view of the CA that Sys employment was unjustly terminated to prevent her from acquiring a regular status in circumvention of the law on security of tenure. As the Court previously stated, this is a common and convenient practice of unscrupulous employers to circumvent the law on security of tenure. Security of tenure, which is a right of paramount value guaranteed by the Constitution, should not be denied to the workers by such a stratagem. The Court can not permit such a subterfuge, if it is to be true to the law and social justice.219[20] In its attempt to justify Sys dismissal, the petitioners relied heavily on the case of Alcira v. NLRC220[21] where the Court stressed that the constitutional protection ends on the expiration of the probationary period when the parties are free to either renew or terminate their contract of employment. Indeed, the Court recognizes the employers power to terminate as an exercise of management prerogative. The petitioners, however, must be reminded that such right is not without limitations. In this connection, it is well to quote the ruling of the Court in the case of Dusit Hotel Nikko v. Gatbonton, 221[22] where it was written:

217 218 219 220 221

Page 92 of 95

As Article 281 clearly states, a probationary employee can be legally terminated either: (1) for a just cause; or (2) when the employee fails to qualify as a regular employee in accordance with the reasonable standards made known to him by the employer at the start of the employment. Nonetheless, the power of the employer to terminate an employee on probation is not without limitations. First, this power must be exercised in accordance with the specific requirements of the contract. Second, the dissatisfaction on the part of the employer must be real and in good faith, not feigned so as to circumvent the contract or the law; and third, there must be no unlawful discrimination in the dismissal. In termination cases, the burden of proving just or valid cause for dismissing an employee rests on the employer. 222[23] [Emphases supplied] Here, the petitioners failed to convey to Sy the standards upon which she should measure up to be considered for regularization and how the standards had been applied in her case. As correctly pointed out by Sy, the dissatisfaction on the part of the petitioners was at best self-serving and dubious as they could not present concrete and competent evidence establishing her alleged incompetence. Failure on the part of the petitioners to discharge the burden of proof is indicative that the dismissal was not justified. The law is clear that in all cases of probationary employment, the employer shall make known to the employee the standards under which he will qualify as a regular employee at the time of his engagement. Where no standards are made known to the employee at that time, he shall be deemed a regular employee. 223[24] The standards under which she would qualify as a regular employee not having been communicated to her at the start of her probationary period, Sy qualified as a regular employee. As held by this Court in the very recent case of Hacienda Primera Development Corporation v. Villegas,:224[25] In this case, petitioner Hacienda fails to specify the reasonable standards by which respondents alleged poor performance was evaluated, much less to prove that such standards were made known to him at the start of his employment. Thus, he is deemed to have been hired from day one as a regular employee . Due process dictates that an employee be apprised beforehand of the condition of his employment and of the terms of advancement therein. [Emphasis supplied] Even on the assumption that Sy indeed failed to meet the standards set by them and made known to the former at the time of her engagement, still, the termination was flawed for failure to give the required notice to Sy. Section 2, Rule I, Book VI of the Implementing Rules provides: Section 2. Security of tenure. (a) In cases of regular employment, the employer shall not terminate the services of an employee except for just or authorized causes as provided by law, and subject to the requirements of due process. (b) The foregoing shall also apply in cases of probationary employment; Provided however, that in such cases, termination of employment due to failure of the employee to qualify in accordance with the standards of the employer made known to the former at the time of engagement may also be a ground for termination of employment.xxx (d) In all cases of termination of employment, the following standards of due process shall be substantially observed:xxx If the termination is brought about by the completion of a contract or phase thereof, or by failure of an employee to meet the standards of the employer in the case of probationary employment, it shall be sufficient that a written notice is served the employee, within a reasonable time from the effective date of termination. [Emphasis and Underscoring supplied]

222 223 224

Page 93 of 95

In this case, the petitioners failed to comply with the requirement of a written notice. Notably, Sy was merely verbally informed that her employment would be terminated on February 28, 2007, as admitted by the petitioners.225[26] Considering that the petitioners failed to observe due process in dismissing her, the dismissal had no legal sanction. It bears stressing that a workers employment is property in the constitutional sense.226[27] Being a regular employee whose termination was illegal.

225 226

Page 94 of 95

i ii iii iv v

You might also like